Вы находитесь на странице: 1из 183

FRCS Tr & Orth

MCQs and Clinical Cases


FRCS Tr & Orth
MCQs and Clinical Cases

Vikas Khanduja MA MSc FRCS (Orth)


Consultant Orthopaedic Surgeon
Addenbrooke’s Hospital, Cambridge University Hospitals NHS Foundation Trust
Cambridge Biomedical Campus, Cambridge, UK

London • Philadelphia • Panama City • New Delhi


© 2014 The British Editorial Society of Bone & Joint Surgery.
Published by JP Medical Ltd
83 Victoria Street, London SW1H 0HW, UK
Tel: +44 (0)20 3170 8910 Fax: +44 (0)20 3008 6180
Email: info@jpmedpub.com Web: www.jpmedpub.com

The rights of Vikas Khanduja to be identified as the editor of this work have been asserted by him
in accordance with the Copyright, Designs and Patents Act 1988.

All rights reserved. No part of this publication may be reproduced, stored or transmitted in any
form or by any means, electronic, mechanical, photocopying, recording or otherwise, except as
permitted by the UK Copyright, Designs and Patents Act 1988, without the prior permission in
writing of the publishers. Permissions may be sought directly from JP Medical Ltd at the address
printed above.

All brand names and product names used in this book are trade names, service marks, trademarks
or registered trademarks of their respective owners. The publisher is not associated with any
product or vendor mentioned in this book.

Medical knowledge and practice change constantly. This book is designed to provide accurate,
authoritative information about the subject matter in question. However, readers are advised
to check the most current information available on procedures included and check information
from the manufacturer of each product to be administered, to verify the recommended dose,
formula, method and duration of administration, adverse effects and contraindications. It is the
responsibility of the practitioner to take all appropriate safety precautions. Neither the publisher
nor the editor assume any liability for any injury and/or damage to persons or property arising
from or related to use of material in this book.

This book is sold on the understanding that the publisher is not engaged in providing professional
medical services. If such advice or services are required, the services of a competent medical
professional should be sought.

Every effort has been made where necessary to contact holders of copyright to obtain permission
to reproduce copyright material. If any have been inadvertently overlooked, the publisher will be
pleased to make the necessary arrangements at the first opportunity.

ISBN: 978-1-907816-93-2

British Library Cataloguing in Publication Data


A catalogue record for this book is available from the British Library

Library of Congress Cataloging in Publication Data


A catalog record for this book is available from the Library of Congress

JP Medical Ltd is a subsidiary of Jaypee Brothers Medical Publishers (P) Ltd, New Delhi, India

Publisher: Richard Furn


Editorial Assistant: Sophie Woolven
Design: Designers Collective Ltd

Indexed, typeset, printed and bound in India.


Preface

The FRCS (Tr & Orth) examination in the UK comprises a written test composed of Multiple Choice
Questions, a clinical component and four 30 minute orals. The clinical component is divided into
cases that focus on the Upper and Lower limb. The orals, or vivas, are divided into four sections
comprising Adult pathology, Children’s orthopaedics and Hands, Trauma and Basic science.
Most other countries, including Australia, follow a similar system of examination wherein can-
didates are assessed on their ability to examine a patient in a clinical setting and then interpret a
clinical case and present a logical management plan in the viva setting.
The goal of this book is to focus specifically on the needs of trainees preparing for the exit exam
in any part of the world and to cater to all the sections of the exam. It is laid out in a format which
is helpful for revision. Chapter 1 includes the MCQs, written in the single best answer (SBA) format
used in the exam, and Chapter 2 provides the answers. The following chapters cover Adult pathol-
ogy, Trauma, Hands, Children’s orthopaedics and Basic science. For each of these themes there are
separate chapters that include, firstly, clinical cases and, secondly, detailed model answers with
references.
A significant amount of work by a number of busy clinicians and educationalists has gone into
the preparation of this book and we have enjoyed the process; we hope you enjoy reading it as
well.

Vikas Khanduja
December 2013

v
Acknowledgements

The editor would like to express his sincere thanks to Matt Henderson, Jennifer Burns and Emma
Vodden from The Bone & Joint Journal for their editorial assistance and continual support in
completion of this project.

vi
Contents

Preface v
Acknowledgements vi
Contributors viii

Chapter 1 SBA Questions 1


Chapter 2 SBA Answers 15
Chapter 3 Viva Questions: Adult pathology 31
Chapter 4 Viva Answers: Adult pathology 41
Chapter 5 Viva Questions: Trauma 61
Chapter 6 Viva Answers: Trauma 71
Chapter 7 Viva Questions: Hands 89
Chapter 8 Viva Answers: Hands 99
Chapter 9 Viva Questions: Children’s orthopaedics 117
Chapter 10 Viva Answers: Children’s orthopaedics 133
Chapter 11 Viva Questions: Basic science 145
Chapter 12 Viva Answers: Basic science 149
Index 169

vii
Contributors

Timothy N Board MD FRCS (Tr&Orth) Ajay Malvyia MS MSc FRCS Ed (Orth)


Consultant Hip and Knee Surgeon Consultant Orthopaedic Surgeon
Wrightington Hospital Wansbeck General Hospital
Wigan, UK Ashington, Northumberland,
UK
Henry Budd MBChB (Hons) FRCS (Tr&Orth)
Specialist Registrar Paul M Robinson MBChB (Hons) BMedSci
Addenbrooke’s Hospital MRCS (Eng)
Cambridge, UK Specialist Registrar
Addenbrooke’s Hospital
Emmet Griffiths MBBS BSc MRCS MSc Cambridge, UK
Specialist Registrar
Addenbrooke’s Hospital Sumedh C Talwalkar MS(Orth) MCh(Orth)
Cambridge, UK FRCS (Tr & Orth)
Consultant Hand and Upper Limb Surgeon
Sim Johal BM (Hons) BSc(Hons) MRCSEng Wrightington Wigan and Leigh NHS Trust
Specialist Registrar Senior Lecturer, Edge Hill University
Addenbrooke’s Hospital Ormskirk, UK
Cambridge, UK
Malin Wijeratna BSc (Hons) MBBS FRCS
Karan Johal BMedSci BMBS (Hons) MRCS MSc (Tr&Orth)
Specialist Registrar Shoulder & Elbow Fellow
Northwest London Hospitals NHS Trust Melbourne Shoulder & Elbow Centre
London, UK Australia

David Jones FRCS FRCSEd (Orth)


Consultant Adviser
The Bone & Joint Journal
London, UK

viii
Chapter 1
SBA Questions

For each question, select the single best answer from the five options listed.

1. Which of the following investigations is the most specific scanning method to


detect infection in a total joint replacement?
A. Bone scintigraphy
B. A combination of white blood cell scan and a technetium bone scan
C. White blood cell scan
D. Sequential gallium-67 citrate scans
E. Sequential technetium bone scans

2. What proportion of patients will develop significant back or ipsilateral knee


pain 25 years after arthrodesis of their hip?
A. 0%
B. 10%
C. 45%
D. 60%
E. 75%

3. With reference to “pulled elbow” injury in children, which of the following


statements is true?
A. Peak incidence is between 3 and 5 years of age
B. It results from traction on the extended and supinated forearm
C. Recurrence is seen in 5% of cases
D. Pain is due to tearing of the lateral collateral ligament complex
E. X-ray is essential for diagnosis

4. During fracture healing, differentiation of the progenitor cells depends on


local oxygen tension and strain. Based on this theory, the following promotes
formation of woven bone during fracture healing:
A. High strain and low oxygen tension
B. Low strain and high oxygen tension
C. Intermediate strain
D. Low oxygen tension
E. Intermediate strain and low oxygen tension
2 Chapter 1  SBA Questions

5. Which of the following statements regarding the effects of glucocorticoids on


bone mineral metabolism is false?
A. Supraphysiologic glucocorticoid levels do not cause secondary
hyperparathyroidism
B. Glucocorticoids inhibit gastrointestinal absorption of calcium
C. Glucocorticoids preferentially affect trabecular bone and the cortical rim of the
vertebral bodies
D. Glucocorticoids inhibit osteoclastic activity
E. Bisphosphonates are successful in the prevention of glucocorticoid-related
osteoporosis

6. The interaction between water and hyaline cartilage is beneficial because:


A. Cartilage repels water
B. Water is absorbed by cartilage
C. Water lubricates the joint
D. Water is imbibed by cartilage and has a passive role in resisting compression
E. Water prevents the degradation of cartilage by matrix metalloproteinases

7. The location of Kaplan’s cardinal line is:


A. Along the ulnar border of the ring finger (parallel to the metacarpophalangeal
joints)
B. From the first web space to the hook of the hamate
C. From the hook of the hamate perpendicular to the proximal transverse palmar
skin crease
D. From the first carpometacarpal joint to the pisiform
E. Along the distal palmar skin crease

8. What type of afferent peripheral nerve fibre is responsible for transmitting


vibration sense to the spinal cord?
A. Aα (A alpha)
B. Aβ (A beta)
C. Aγ (A gamma)
D. Aδ (A delta)
E. C

9. Which of the following investigations best distinguishes osteoporosis from


osteomalacia?
A. Bone biopsy from the iliac crest
B. Serum calcium
C. Serum phosphate
D. Urinary calcium
E. Tetracycline-labelled bone biopsy
SBA Questions 3

10. The nerve supplying to teres minor is a branch of which of these nerves?
A. Suprascapular nerve (C5, C6)
B. Lower subscapular nerve (C5, C6)
C. Upper subscapular nerve (C5, C6)
D. Axillary nerve (C5, C6)
E. Medial supraclavicular nerve (C3, C4)

11. Which of the following clinical findings is least likely to be associated with a
pre-ganglionic brachial plexus injury?
A. Bruising in the anterior triangle of the neck
B. Pain in an insensate hand
C. Loss of sensation above the clavicle
D. Ipsilateral Horner’s syndrome
E. Loss of muscle function of branches direct from the roots of the brachial plexus

12. Following a latissimus dorsi transfer for chronic irreparable tears of the rotator
cuff, which of the following factors has NOT been associated with a poor clinical
outcome?
A. Male gender
B. Poor pre-operative shoulder function
C. Generalised muscle weakness
D. Absence of electrical activity at follow-up
E. Previous failed rotator cuff repair

13. The alpha angle on the ultrasound of an infant’s hip is defined as:
A. The angle between the acetabular roof and the midline of the pelvis
B. The acute angle between the lateral wall of the ilium and the bony acetabular
roof
C. The angle between the centre of the femoral head and the lateral wall of the
ilium
D. The angle of the thigh required to produce subluxation of the hip on the
sonogram
E. The angle between the acetabular roof and the transverse plane

14. Which of these values reflects the normal tibiofemoral axis for a child aged three
years?
A. Varus of 20°
B. Varus of 10°
C. Neutral alignment
D. Valgus of 10°
E. Valgus of > 20°
4 Chapter 1  SBA Questions

15. Which of the following constituent accounts for 65–80% of the dry mass of flexor
tendons?
A. Collagen type I
B. Collagen type II
C. Collagen type III
D. Collagen type IV
E. Elastin

16. Which of the following statements is correct with regard to a therapeutic intra-
articular facet joint injection in the lumbar region for low back pain?
A. Pain relief on two occasions after a facet joint injection is an indication for
facet joint ablation
B. It aims to relieve back pain during flexion of the lumbar spine
C. Steroid injection in the facet joint gives good pain relief, beyond six months
D. The best visualisation of the facet is on the lateral image intensifier view
E. It has a high risk of haematoma and infection

17. What percentage of success would you quote to patients being offered a
coccygectomy for coccygodynia that has been refractory to conservative
management?
A. < 20%
B. 20% to 30%
C. 40% to 50%
D. 60% to 70%
E. > 80%

18. Which one of the following statements is false with regard to the clinical
presentation of patients with tarsal tunnel syndrome?
A. Symptoms are variable
B. There is sensory disturbance along the big toe
C. There is atrophy of the intrinsic muscles of the foot
D. There is hind-foot varus deformity
E. Symptoms accentuate on eversion and dorsiflexion of the foot
SBA Questions 5

19. Meta-analysis comparing the intermediate and long-term outcome after total
ankle replacement and ankle arthrodesis has shown all of the following except:
A. Mean AOFAS (American Orthopaedic Foot and Ankle Society) Ankle–Hindfoot
Scale score is higher for patients with ankle arthrodesis
B. A greater number of patients have poor results after ankle replacement
compared with ankle arthrodesis
C. The revision rates for both procedures are similar
D. Five and ten year survival after ankle replacement are more than 75%
E. Below knee amputation rate is higher for patients with ankle arthrodesis

20. With regard to bearing surfaces in artificial joints:


A. Equatorial bearing is ideal
B. Polar bearing is more conducive to fluid film lubrication
C. Subhemispherical sockets are always advantageous
D. Sacrificial bearings will always fail before hard-on-hard bearings
E. The synovial fluid in prosthetic joints is indistinguishable from normal synovial
fluid

21. Which of the following statements regarding spinal tumours is false?


A. About 15% of all bone tumours are primary spine tumours
B. Combined surgical decompression and radiotherapy is generally superior to
radiotherapy alone in the treatment of metastatic spinal cord compression
C. Spinal metastases from renal cell carcinoma tend to be hypervascular
D. Spinal tumours often present with pain and weakness
E. Ependymoma is the most common type of intramedullary tumour in adults

22. Which of the following is NOT a cause of leg length shortening?


A. Epiphysiodesis
B. Coxa vara
C. Fracture
D. Klippel–Trénaunay–Weber syndrome
E. Poliomyelitis

23. In monostotic fibrous dysplasia, the prevalence of malignant transformation


(chondrosarcoma or osteosarcoma) is about:
A. 0.4%
B. 0.8%
C. 1%
D. 2%
E. 4%
6 Chapter 1  SBA Questions

24. Which of the following statements is true regarding an open repair of the
ruptured ulnar collateral ligament of the thumb?
A. It is unusual to see the superficial radial nerve
B. One should aim to identify the extensor pollicis brevis tendon
C. A Stener lesion will be found, if present, proximal to the adductor
aponeurosis
D. The ligament will have ruptured from its insertion into the metacarpal in most
cases
E. A Stener lesion will be found in 50% of the cases

25. Which one of the following is necessary for a good key grip?
A. Extension of the thumb metacarpophalangeal joint
B. Extension of the thumb interphalangeal joint
C. Function of the first interosseous muscles
D. Function of the radial nerve
E. Function of the median nerve

26. All of these are examples of enchondral bone formation except:


A. Embryonic long-bone formation
B. Fracture callus
C. Bone formation after use of demineralised bone matrix
D. During distraction osteogenesis
E. Longitudinal growth

27. All of these are diseases that involve the proliferative zone of the growth plate
except:
A. Kniest syndrome
B. Achondroplasia
C. Gigantism
D. Malnutrition
E. Irradiation injury

28. Which one of these regulates cartilage and bone formation in the fracture callus?
A. Bone morphogenic protein
B. Transforming growth factor-β
C. Insulin-like growth factor II
D. Platelet derived growth factor
E. Endothelial derived growth factor
SBA Questions 7

29. What is the diagnosis in a patient who presents with reduced serum calcium,
raised serum phosphate, normal alkaline phosphatase and parathormone level
and a reduced urinary calcium excretion?
A. Nutritional rickets
B. Hypoparathyroidism
C. Pseudohypoparathyroidism
D. Secondary hyperparathyroidism
E. Nutritional calcium deficiency

30. During revision knee arthroplasty with the trial components in place, the knee
is tight in extension and loose in flexion. Correction involves which of the
following changes?
A. Use of a thinner tibial insert
B. Use of a thinner distal femoral augmentation wedge
C. Use of a smaller femoral component
D. Resection of more proximal tibia
E. Shifting the femoral component anteriorly with an offset stem

31. A 26-year-old basketball player was noted to have an absent anterosuperior


labrum during a shoulder MRI arthrogram. Which of the following would be
true for a Buford complex?
A. The patient needs further assessment with shoulder arthroscopy
B. Biceps insertion site will usually be involved
C. Presence of a cord-like middle glenohumeral ligament actually represents the
torn labrum
D. Sites of insertion of anterior and middle glenohumeral ligament are often
abnormal
E. Reattaching the complex will lead to painful restriction of rotation

32. Which one of the following exercise regime would you recommend to someone
who wishes to improve his/her muscle bulk?
A. Isometric
B. Isotonic
C. Isokinetic
D. Plyometric
E. Aerobic
8 Chapter 1  SBA Questions

33. With regards to the Bunnell-Littler test, which of the following statements is
false:
A. An increase in proximal interphalangeal (PIP) joint flexion with flexion of
metacarpophalangeal (MCP) joint indicates intrinsic tightness
B. Reduced flexion of PIP joint in extended and flexed attitude of the MCP joint
would indicate a capsular contracture of the joint
C. Extension of PIP joint with attempted flexion of MCP joint may indicate
lumbrical tightness
D. Reduced PIP joint flexion with flexion of MCP joint may indicate contracture of
the extensor tendons
E. Flexor digitorum profundus laceration distal to lumbrical origin may give a
false positive test

34. A 13-year-old girl presents with a flexion deformity of the little finger PIP joint
(camptodactyly). Which one of the following statements is true?
A. The deformity is usually due to an abnormality in the lumbrical or flexor
digitorum superficialis insertion
B. The deformity is commonly associated with Down’s syndrome
C. The deformity will usually respond to splinting and stretching
D. Capsular release and tendon transfer is indicated if full PIP extension cannot
be achieved with MCP held in flexion
E. Corrective osteotomy is indicated

35. Which one of the following is true about arthrogryposis?


A. It results from defect in the motor unit
B. It is commonly myopathic in origin
C. It leads to a contracture of the joint
D. Joint contractures are progressive
E. The involvement of other organ systems is unusual

36. Which one of the following statements is false regarding septic arthritis in
children?
A. Males are affected twice as often as females
B. A lower extremity (hip) is affected in 80% of patients
C. Polyarticular involvement is in fewer than 10% of patients
D. Loss of proteoglycan starts at five days from the bacteria entering the joint
E. An aspirate white blood cell count of > 50 000/mm3 with 75%
polymorphonuclear leucocytes is diagnostic of sepsis in 60% to 70% of patients
SBA Questions 9

37. Which one of the following statements is true regarding the use of forest plots in
systematic reviews? They:
A. Summarise treatment efficacies (risk ratio) across trials
B. Detect publication bias in literature
C. Are a quantifiable way to test prospective studies for homogeneity
D. Are a measure of odds of failure against the size of the study
E. Detect outliers in the literature in terms of study inclusion criteria

38. Which of the following statements is false for the AAOS grades of
recommendation for summaries or reviews of orthopaedic studies?
A. A high-quality prospective prognostic study investigating the effect of a patient
characteristic on the outcome of disease would constitute a good evidence
study (Grade A)
B. A systematic review of Level I studies investigating the results of treatment with
inconsistent results would constitute a fair evidence study (Grade B)
C. A prospective comparative study investigating the results of a surgery with
consistent findings would constitute a fair evidence study (Grade B)
D. A retrospective comparative study investigating the results of an intervention
would constitute a fair evidence study (Grade B)
E. An expert opinion from someone who has performed more than 1000 complex
surgical intervention would constitute a poor evidence study (Grade C)

39. All of the following have been shown to slow disease progress in osteoarthritis
except:
A. Glucosamine sulphate
B. Chondroitin sulphate
C. Diacetylrhein
D. Hyaluronic acid
E. Doxycycline

40. Which of the following statements is true regarding the use of tranexamic acid in
orthopaedic surgery?
A. It increases the prothrombin time
B. Meta-analysis has shown that it increases the risk of thromboembolic
events
C. It reduces the need for transfusion after joint replacement
D. It is only effective when given intravenously
E. It should only be used in high risk cases
10 Chapter 1  SBA Questions

41. Which of the following statements is true when performing a unilateral wrist
arthrodesis?
A. 15° ulnar deviation is preferred
B. 10° to 20° of dorsiflexion is ideal to preserve grip strength
C. Ulna–triquetral abutment cannot be avoided
D. An open epiphyseal plate in the distal radius is not a contraindication
E. The most common surgical approach is volar

42. In the surgical treatment of de Quervain’s stenosing tenosynovitis, which one of


the following tendons should be decompressed?
A. Abductor pollicis longus
B. Adductor pollicis
C. Extensor pollicis longus
D. Flexor pollicis longus
E. Opponens pollicis

43. Which one of the following is true when comparing non-anatomical (reattachment
to the brachialis muscle) with anatomical reinsertion of the distal biceps brachii
tendon following rupture?
A. No difference in strength of flexion or supination
B. Improved strength of flexion and supination
C. Improved strength of supination
D. Decreased strength of flexion and supination
E. Decreased strength of supination

44. Which of the following is not a recognised technique for delayed posterolateral
corner reconstruction?
A. Popliteal bypass (Muller’s procedure)
B. Figure of eight reconstruction (Larsen’s procedure)
C. Two-tailed (Warren’s procedure)
D. Three-tailed (Warren/Miller procedure)
E. Mumford procedure

45. Which of the following is not a cause for failed anterior cruciate ligament
reconstruction?
A. Associated posterolateral corner injury
B. Cyclops lesion impingement
C. Tibial tunnel placement 10 mm to 11 mm anterior to posterior cruciate
ligament insertion
D. Arthrofibrosis of graft
E. Returning to full sporting activities at three months
SBA Questions 11

46. Which one of the following statements is false with regards to hyperbaric oxygen
therapy for the treatment of chronic osteomyelitis?
A. It promotes collagen formation
B. It improves healing of ischaemic wounds
C. It promotes angiogenesis
D. An 85% remission rate has been reported
E. It has no direct bactericidal effect

47. With regards to metacarpophalangeal joint arthritis of the fingers, which of the
following statements is true?
A. It is more common in osteoarthritis than rheumatoid arthritis
B. Silicone prostheses have a high rate of fracture requiring revision surgery
C. Silicone prostheses have been shown to improve range of movement in the
longer term
D. Overall long-term patient satisfaction after implantation of silicone prostheses
is below 50%
E. Unconstrained metal–polyethylene prostheses have good predictable results in
all patients

48. Which of the following statements is false with regards to tibial malunion?
A. Defined as an angulation of more than 10° in the coronal or the sagittal plane
B. Coronal plane malalignment is more symptomatic
C. Up to 20° of malalignment can be tolerated without significantly increasing the
pressure on the cartilage
D. Tibial lengthening should be considered for leg-length discrepancy of more
than one inch
E. Rotational malunion of more than 10° has been shown in up to 22% after tibial
nailing

49. Which one of the following is true with regards to the stages of posterior tibial
tendon insufficiency?
A. Forefoot abduction is noted in Stage I
B. Dynamic hind foot deformity is noted in Stage II
C. Hindfoot valgus is correctable in Stage III
D. Correctable ankle valgus is noted in Stage III
E. Forefoot pronation is present in Stage IV
12 Chapter 1  SBA Questions

50. Which one of the following is the strongest factor leading to symptomatic
degenerative disc disease?
A. Heavy lifting
B. Vibration forces
C. Torsional loads
D. Tobacco exposure
E. Genetic predisposition

51. In relation to orthopaedic screws:


A. Pitch is directly proportional to thread angle
B. Locking screws achieve stability by cold welding to the plate
C. The correct drill bit size for the threaded hole for a 4.5 mm cortical screw is
3.5 mm
D. Reducing shank diameter increasing the risk of fatigue failure
E. A larger ratio of core diameter to outer diameter leads to higher pull out
strength

52. Which one of the following statements is false with regards to the use of
autotransfusion in orthopaedic surgery?
A. Should be used only if the transfusion risk is more than 10%
B. Should not be used if bone cement is being used during joint replacement
surgery
C. Contraindicated in infected cases
D. Contraindicated in the presence of malignancy
E. Is a good source of clotting factors

53. What is the probable mechanism of failure of a cemented total hip replacement
with radiolucent lines on the anteroposterior radiograph in Gruen zones two
and six?
A. Medial stem pivot
B. Calcar pivot
C. Cantilever failure
D. Pistoning between cement and bone
E. Pistoning between cement and implant

54. Regarding genetic transmission, which one of the following inheritance patterns
is seen in patients with familial hypophosphataemic rickets?
A. Autosomal recessive
B. Autosomal dominant
C. X-linked recessive
D. X-linked dominant
E. Mixed pattern
SBA Questions 13

55. Delayed gadolinium-enhanced MR imaging to detect articular cartilage


degeneration relies on the content and depletion of which one of the following
in the hyaline cartilage?
A. Proteoglycan
B. Chondroitin sulphate
C. Keratin sulphate
D. Collagen
E. Water

56. During the process of nerve regeneration, which one of the following modalities
is the first to return?
A. Fine touch
B. Deep touch
C. Vibration
D. Pain
E. Motor

57. A 25-year-old weight-lifter had an MRI examination of his shoulder, which


shows a SLAP lesion with a cyst in the spinoglenoid notch. Which one of the
following clinical signs/tests would be expected to be positive in this scenario?
A. Jobe
B. Horn blower
C. Lift-off
D. Belly push
E. Speed

58. All of the following are possible sites for compression of the radial nerve except:
A. Fascial band at radial head
B. Edge of extensor carpi radialis brevis
C. Recurrent leash of Henry
D. Arcade of Frohse
E. Ligament of Struthers

59. Which one of the following statements is true about Wartenberg’s syndrome?
A. Pain along the ulnar side of forearm
B. Paraesthesiae along the dorso-radial side of the hand
C. Aggravated by forearm supination
D. Surgery is usually required
E. Typically associated with weakness of wrist dorsiflexion
14 Chapter 1  SBA Questions

60. Which one of the following would be the most suitable test to detect a difference
between the grade of Perthes’ disease (Herring classification) in two different
population groups belonging to the rural or urban areas?
A. Paired t-test
B. ANOVA
C. Mann–Whitney U test
D. Wilcoxon signed rank test
E. Kruskal–Wallis test

61. Which one of the following would be the most suitable approach to stabilise a
T-type fracture of the acetabulum?
A. Kocher–Langenbeck
B. Ilioinguinal
C. Modified Smith–Petersen
D. Ilio-femoral approach
E. Combined anterior and posterior approach

62. What is the mechanism of injury for a typical Weber C fracture as per the Lauge–
Hansen classification?
A. Supination–abduction
B. Supination–external rotation
C. Pronation–abduction
D. Pronation–external rotation
E. Pronation–dorsiflexion

63. All of these are associated with a pes cavus deformity except?
A. Forefoot adduction
B. Forefoot supination
C. Hindfoot varus
D. Plantar flexion of first metatarsal
E. Clawing of the great toe
Chapter 2
SBA Answers

1. B  A combination of white blood cell scan and a


technetium bone scan
There is no single pre-operative investigation that can reliably diagnose a
prosthetic joint infection. Diagnosis is reliant on a thorough history and clinical
examination. This can be followed by plain radiographs, blood tests, aspiration of
the joint and the use of specialised imaging. In 2010, a working group from the
American Academy of Orthopaedic Surgeons released a guideline and evidence
report,1 which gave recommendations for the management of prosthetic joint
infections. One of the recommendations was that “nuclear imaging is an option
in patients in whom diagnosis of peri-prosthetic joint infection has not been
established and are not scheduled for re-operation.” A combination of white blood
cell scan and a technetium bone scan is the most specific scanning method.

2. D 60%
Callaghan et al2 performed a retrospective study into the long-term outcomes of
twenty-eight patients following hip arthrodesis. About 60% of the patients had pain
in the ipsilateral knee, with an average time to onset of 23 years. A similar percentage
had back pain. Pain in the contralateral hip occurred in approximately 25%.

3. C  Recurrence is seen in 5% of cases


Subluxation of the head of radius from the annular ligament is a common elbow
injury. It is most common amongst young children, prior to the age of six. It results
from traction on the hand with the elbow extended and the forearm pronated. The
annular ligament either tears or slips over the radial head.
After reduction, the elbow joint capsule remains stretched for a few months.
Studies have reported recurrence rates up to 5%.

4. B  Low strain and high oxygen tension


The manner in which mechanical factors influence fracture healing is explained
by Perrin’s strain theory. Strain is the deformation of a material when a given
force is applied. It is the change in length in comparison to original length after
a given load has been applied. It is expressed as a percentage. Granulation tissue
16 Chapter 2  SBA Answers

has a strain tolerance of 100%, whereas bone has a normal strain tolerance of 2%.
Bony bridging between the distal and proximal callus can only occur when local
strain is less than the forming woven bone can tolerate. A low amount of strain is
therefore ideal. Hard callus will not form if movement is too great. Experimental
studies have elucidated that an environment with a high oxygen tension is
beneficial for osteogenic progenitor cell differentiation.3

5. A  Supraphysiologic glucocorticoid levels do not cause


secondary hyperparathyroidism
Glucocorticoid excess can lead to osteoporosis. Glucocorticoids decrease
intestinal calcium absorption and decrease renal tubular calcium reabsorption.
This in turn raises urinary calcium levels. Diminished calcium absorption and
increase of its excretion can lead to secondary hyperparathyroidism.4

6. D  Water is imbibed by cartilage and has a passive role in


resisting compression
Water shifts in and out of cartilage to allow deformation of cartilage in response
to stress. Water makes up 65–80% of the wet weight of hyaline cartilage. In
osteoarthritis this can rise to 90% but in very severe osteoarthritis the percentage
of water can be reduced.5

7. B  From the first web space to the hook of the hamate


Kaplan’s cardinal line is defined as the transverse line drawn from the apex of
the thumb-index web space towards the hook of the hamate, parallel with the
proximal palmar crease. It is just proximal to the superficial palmar arterial arch and
is an important consideration during hand surgery.6

8. B  Aβ (A beta)7

Table 2.1

Fibre Function
Aα Alpha motor neurones, muscle spindle primary endings,
Golgi tendon organs, touch
Aβ Touch, vibration sense, muscle spindle secondary endings

Aγ Touch, pressure, gamma-motor neurones

Aδ Pain, crude touch, pressure, temperature


SBA Answers 17

9 E  Tetracycline-labelled bone biopsy


Osteoporosis is a state of decreased bone mass with normal bone mineralisation.
In osteomalacia, bone mass may be variable but mineralisation is decreased. When
undertaking blood tests, serum calcium and phosphate are likely to be normal in
osteoporosis and may be low or normal in osteomalacia. However, osteomalacia
as a result of hypophosphatasia may cause elevated calcium and phosphate.
Urinary calcium may be high or normal in osteoporosis and low or normal in
osteomalacia (high in hypophoshatasia). For these reasons the blood and urine
tests will not differentiate well. In terms of bone biopsy, tetracycline labelling
allows the rate of calcium uptake and turnover to be estimated (mineralisation)
and therefore in osteomalacia tetracycline labels are abnormal and they are normal
in osteoporosis.5

10. D  Axillary nerve (C5, C6)


• Suprascapular nerve (C5, C6) supplies supraspinatus and infraspinatus.
• Lower subscapular nerve (C5, C6) supplies subscapularis and teres major
• Upper subscapular nerve (C5, C6) supplies subscapularis
• Axillary nerve (C5, C6) supplies deltoid and teres minor
• Medial supraclavicular nerve (C3, C4) is one of several nerves arising from the
cervical plexus that supply the skin over the upper medial part of the chest.8

11. A  Bruising in the anterior triangle of the neck


A pre-ganglionic plexus injury may be associated with bruising in the posterior
triangle of the neck rather than the anterior, due to the anatomical location of the
pre-ganglionic portion of the brachial plexus.

12. A  Male gender


Ianotti et al9 found that female gender was more likely to be associated with a poor
outcome following a latissimus dorsi transfer. It is postulated that men have an
increased muscle mass and therefore improved pre-operative shoulder function
and strength.

13. B  The acute angle between the lateral wall of the ilium
and the bony acetabular roof
The alpha angle is used most commonly as a measurement of acetabular concavity,
and it is calculated as the angle between the lateral wall of the ilium and the roofline.
A normal alpha angle is 60º or greater. This measurement is vital to understanding
the morphology of the immature acetabulum in developmental dysplasia.
18 Chapter 2  SBA Answers

14. D  Valgus of 10°


During development, the tibio-femoral alignment in children changes during
early years. At birth it is 10° to 15° of varus, which remodels to neutral at about 24
months of age and then becomes 10° of valgus by the age of three years. Over the
next four years it gradually goes to normal alignment of 7°.10

15. A  Collagen type I


Tendons are composed of groups of collagen bundles (fascicles) separated by
endotenon and surrounded by epitenon. Tendons are primarily composed of
water, but of the dry mass type I collagen makes up 65–80% of the tendon. Type III
collagen makes up less than 5%. Proteoglycans also make up less than 5% of the
tendon’s dry mass.
• Type I collagen: skin, tendon, vascular, ligature, organs, bone (main component
of bone)
• Type II collagen: cartilage (main component of cartilage)
• Type III collagen: reticulate (main component of reticular fibres)
• Type IV collagen: forms bases of cell basement membrane

16. A  Pain relief on two occasions after a facet joint injection


is an indication for facet joint ablation
If facet joint injection has provided successful pain relief the diagnosis is verified
and therefore facet joint ablation is indicated. However, the exact number of
lumbar facet joint injections required prior to facet joint ablation is disputed.11

17. E  > 80%


Kerr et al12 reported a favourable outcome in 84% of patients after coccygectomy
for refractory coccygodynia.

18. D  There is hind-foot varus deformity


Tarsal tunnel syndrome is a compressive neuropathy caused by compression
of the tibial nerve as it traverses the tarsal tunnel. This is the space between the
calcaneus and talus medially, the abductor halluces inferiorly and the flexor
retinaculum. Causes of compression may be intrinsic, such as a ganglion cyst; or
extrinsic, such as valgus deformity of the hindfoot. Symptoms are variable but the
main symptom is of vague medial foot pain with occasional burning. There is often
an association with intermittent parasthesiae related to prolonged standing or
SBA Answers 19

walking. Symptoms may be made worse by eversion and dorsiflexion of the foot.
On examination a Tinel’s sign may be evident and wasting of the intrinsic muscles
of the foot may be seen. However, the compression test is the most sensitive and
specific clinical sign. To elicit this sign, digital pressure is applied over the tarsal
tunnel whilst the foot is held in plantar flexion and inversion. Reproduction of the
symptoms is a positive response.

19. A  Mean AOFAS (American Orthopaedic Foot and Ankle


Society) Ankle–Hindfoot Scale score is higher for patients
with ankle arthrodesis
A 2007 meta-analysis of 49 primary studies confirmed that the intermediate
outcome of total ankle arthroplasty appears to be similar to that of ankle
arthrodesis. The mean AOFAS Ankle-Hindfoot Scale score was higher after
arthroplasty than after arthrodesis. For arthroplasty, the implant survival rate was
77% at 10 years. Revision rates were 7% after arthroplasty and 9% after arthrodesis.
The main reasons for revision were subsidence and/or loosening after arthroplasty
and nonunion after arthrodesis. Below-the-knee amputation rates were 1% and
5%, respectively.

20. B  Polar bearing is more conducive to fluid film lubrication


With equatorial bearing fluid lubrication is prevented. Although polar bearing is
more conducive to fluid film lubrication, mid-polar bearing is ideal as it allows fluid
in and out to lubricate the joint.13,14

21. A  About 15% of all bone tumours are primary spine


tumours
Bone tumours can be benign or malignant. They can also be classified as primary
(arising from the bone) or secondary (metastases from different sites). Secondary
bone tumours make up the vast majority of all bone tumours. Therefore 15% is
a significantly greater percentage than the expected figure for primary spinal
tumours.15

22. D  Klippel–Trénaunay–Weber syndrome


Klippel–Trénaunay–Weber syndrome is characterised by a triad of port-wine stain,
varicose veins, and bony and soft-tissue hypertrophy involving an extremity. It
therefore would not lead to leg length shortening.
20 Chapter 2  SBA Answers

23. A  0.4%
There is a reported rate of 0.4% risk of malignant transformation. Malignant
degeneration of fibrous dysplasia complicates less than 1% of all cases, presenting
clinically as pain and swelling. Radiographic findings include cortical destruction
and associated soft-tissue masses. The most common malignancies include
osteosarcoma, fibrosarcoma, and malignant fibrous histiocytoma. Transformation
to chondrosarcoma has been reported, sometimes erroneously on the basis of the
incidental finding of cartilaginous nodules in a specimen. The true number of cases
of malignant degeneration is likely to be overestimated given previous irradiation
of involved bone in many cases.16

24. C  A Stener lesion will be found, if present, proximal to


the adductor aponeurosis
During surgical repair it is important to identify branches of the superficial radial
nerve in the proximal aspect of the wound. One should aim to identify the extensor
pollicis longus. A Stener lesion,17 which has been reported in up to 80% of cases is
found proximal to the adductor aponeurosis. The ulnar collateral ligament nearly
always separates from the base of first phalanx of the thumb rather than the
metacarpal.

25. C  Function of the first interosseus muscles


A key pinch grip involves opposing the pulp of the distal phalanx of the thumb to
the radial border of the proximal phalanx of the index finger.

26. D  During distraction osteogenesis


Distraction osteogenesis mimics intramembranous ossification where recruitment
and differentiation of primitive cells create a new bony framework.

27. A  Kniest syndrome


Kniest syndrome (spondyloepiphyseal dysplasia) is condition that involves a
disproportionate dwarf. Patients have a short trunk and limbs with large joints. It
is inherited in an autosomal dominant pattern and is caused by mutations in the
COL2A1 gene. Kniest syndrome involves all zones of the growth plate as there is a
defective collage II molecule assembly.18
SBA Answers 21

28. B  Transforming growth factor-β19


Transforming growth factor-β (TGF-β) has a proven role in regulating cartilage
and bone formation in fracture callus. In contrast, only some of the more than 20
known bone morphogenic proteins (which all belong to the TGF-β superfamily)
induce bone formation; they do this by promoting differentiation of mesenchymal
cells into chondrocytes and osteoblasts. Insulin-like growth factor promotes
proliferation and differentiation of osteoprogenitor cells. Platelet derived growth
factor is a mitogen for both mesenchymal cells and osteoblasts. Endothelial
derived growth factor has a role in angiogenesis in fracture healing.

29. C  Pseudohypoparathyroidism
In this case there is a PTH receptor abnormality and PTH is not able to exert
its effect on target cells. This leads to a reduction in the active form of vitamin
D. The low PTH and low active vitamin D levels lead to the low calcium. In
hypoparathyroidism the PTH level would be low as well.18

30. B  Use of a thinner distal femoral augmentation wedge


Using a thinner distal femoral augmentation wedge will increase the extension gap
without affecting the flexion gap.20

31. E  Reattaching the complex will lead to painful


restriction of rotation
The Buford complex is an anatomical variant of the anterosuperior labrum.
Williams et al21 noted that it was present in 1.5% of shoulders arthroscoped
and consisted of a “cord-like” middle glenohumeral ligament that originated
directly from the superior labrum at the base of the biceps tendon and
crossed the subscapularis tendon to insert on to the humerus. There was
absent anterosuperior labral tissue present between this attachment and the
midglenoid notch. Anatomical variant could be confused with labral detachment.
However, if reattached to the glenoid, severe painful restriction of rotation and
elevation would occur.

32. A  Isometric
The following are different types of muscle contraction:
• isometric–muscle contraction with constant length (e.g. pushing a fixed object)
• isokinetic–muscle contraction with constant speed
22 Chapter 2  SBA Answers

• plyometric–initial rapid lengthening followed by contraction of muscle (e.g.


jumping up and down)
• isotonic–muscle contraction with constant tension
• concentric–muscle shortens during contraction (e.g. biceps curl)
• eccentric–muscle lengthens during contraction

33. E  Flexor digitorum profundus laceration distal to


lumbrical origin may give a false positive test
An FDP laceration distal to lumbrical origin would lead to a lumbrical plus finger, in
which flexion at the MCPJ would lead to PIPJ extension. Therefore a lumbrical plus
finger would not give a false positive Bunnell–Littler test.

34. A  The deformity is usually due to an abnormality in the


lumbrical or flexor digitorum superficialis insertion
There is very rarely any compromise in function; therefore operative intervention
is usually not indicated. Infantile type camptodactyly can respond to stretching. If
multiple digits are involved they can be associated with other syndromes, although
this is rare.

35. A  It results from a defect in the motor unit


Arthrogryposis is a rare congenital disorder that is characterised by non-
progressive joint contractures. Causes can be extrinsic (severe oligohydramnios
or other feto/maternal reasons for reduced fetal movement) or intrinsic (CNS,
PNS, connective tissue disorders). Other organ abnormalities can be noted
as well.

  Table 2.2

Grade Description

A Good evidence (Level I studies with consistent finding) for or against recommending
intervention.

B Fair evidence (Level II or III studies with consistent findings) for or against recommending
intervention.

C Poor-quality evidence (Level IV or V) for or against recommending intervention.


D There is insufficient or conflicting evidence not allowing a recommendation for or against
intervention.
See also levels of evidence http://www.aaos.org/research/evidence/levelstables.pdf
SBA Answers 23

36. E  An aspirate white blood cell count of > 50 000/mm3


with 75% polymorphonuclear leucocytes is diagnostic of
sepsis in 60% to 70% of patients
Male gender is a risk factor for septic arthritis. The knee and the hip are affected
in around one third of cases each, with the lower extremity accounting for
the majority of cases.22–24 An aspirate, WBC count > 50 000/mm3 with 75%
polymorphonuclear leucocytes is diagnostic of sepsis in 30–50%.

37. A  Summarise treatment efficacies (risk ratio) across trials


Forest plots are graphical representations of meta-analysis. They show the effect
estimate and weight of each individual study as well as the overall effect estimate
of combined studies.
The effect measures that are commonly used include odds ratio and risk ratio
amongst others.
A box and a horizontal line represent each study. The mid-point of the box
represents the point effect estimate of each study. The area of the box represents
the weight given to each study. The width/length of the horizontal line represents
the 95% confidence interval for the effect estimate of each study.
A diamond represents the overall effect estimate. The width of the diamond
represents the confidence interval for the overall effect estimate.
There is also a vertical line, known as the line of no effect.
The Forest plot also provides the summary data entered for each study. In addition,
it provides the weight for each study, the effect measure, method and the model
used to perform the meta-analysis, the confidence intervals used, the effect
estimate from each study, the overall effect estimate and the statistical significance
of the analysis. Information is also provided about the heterogeneity of the analysis.

38. B  A systematic review of Level I studies investigating


the results of treatment with inconsistent results would
constitute a fair evidence study (Grade B)
AAOS Grades of Recommendation for Summaries or Reviews of Orthopaedic
Surgical Studies: http://www.aaos.org/research/evidence/gradesofrec.asp

39. D  Hyaluronic acid


There is evidence that all of these agents have some effect on disease progression:
• Glucosamine (Cochraine review),
24 Chapter 2  SBA Answers

• Chondroitin (various studies, Cochrane review in progress),


• Diacetylrhein (Cochrane review),
• Doxycycline (Cochrane review “The symptomatic benefit of doxycycline is
minimal to non-existent. The small benefit in terms of joint space narrowing
is of questionable clinical relevance and outweighed by safety problems.
Doxycycline should not be recommended for the treatment of osteoarthritis of
the knee or hip”),
• Hyaluronic acid (Cochrane review on viscosupplementation does not comment
on disease progression, but concludes that HA is beneficial for pain and
function. Some papers quote a positive effect on disease progression).

40. C  It reduces the need for transfusion after joint


replacement
Tranexamic acid significantly reduces the need for allogeneic blood transfusion
after joint replacement surgery.25,26 Meta-analysis of existing RCTs does not
support an increased risk of thromboembolic events. However, the overall effect on
mortality, DVT and PE remains uncertain.

41. B  10° to 20° of dorsiflexion is ideal to preserve grip


strength
Maximum power grip is achieved at around 35° of extension, but this position
would interfere with activities of daily living.
In patients with rheumatoid arthritis, 5° to 10° ulnar deviation may be desirable
to counteract the “Z” deformity. The rheumatoid wrist should be fused in
slight flexionbecause the rheumatoid patient will often have difficulty with
MCPJ extension, and therefore, the wrist should be fused in slight flexion to
accommodate the extensor tenodesis effect at the MCPJ.

42. A  Abductor pollicis longus


In the surgical treatment of de Quervain’s stenosing tenosynovitis the first extensor
compartment is decompressed. Specifically the extensor pollicis brevis and abductor
pollicis longus tendons, either of which may be lying in a separate sub-sheath.

43. E  Decreased strength of supination


Klonz et al27 showed that following brachialis tenodesis supination strength was
only 42% to 56% of the uninjured arm, compared with 91% supination strength
following anatomic repair. Flexion strength was 97% following anatomic repair and
96% following brachialis tenodesis.
SBA Answers 25

44. E  Mumford procedure


The Mumford procedure is done to treat acromioclavicular joint disorders. The
distal end of the clavicle bone is resected to relieve the symptoms. This procedure
may be done arthroscopically or open.

45. C  Tibial tunnel placement 10 mm to 11 mm anterior to


posterior cruciate ligament insertion
Hutchinson and Bae28 showed that the anterior border of PCL was consistently 10.9
mm posterior to central sagittal insertion point of the ACL in a cadaveric study.

46. E  It has no direct bactericidal effect


An 85% remission rate has been reported with surgery, antibiotics and HBOT
together by Morrey et al.29 Hyperbaric oxygen therapy is bacteriostatic and
bactericidal as it increases oxygen free radicals.

47. D  Overall long-term patient satisfaction after


implantation of silicone prostheses is below 50%
Möller et al30 showed that there was a silicone spacer fracture in 20 out of 120
implants in a prospective RCT.
Silicone prosthesis have a higher rate of fracture but not all require revision
surgery. Unconstrained prosthesis don’t have predictable results in rheumatoid
patients with loss of soft tissues and collateral ligaments.31

48. A  Defined as an angulation of more than 10° in the


coronal or the sagittal plane
Tibial malunion is defined as an angulation more than 5° in the coronal or the
sagittal plane.32

49. B  Dynamic hind foot deformity is noted in Stage II


Johnson and Strom et al33 described tibialis posterior dysfunction in 4 stages.
A dynamic hindfoot valgus deformity, attenuation of the spring ligament and
progressive flattening of the longitudinal arch characterise stage II.
26 Chapter 2  SBA Answers

50. E  Genetic predisposition34


The aetiology of disc degeneration is complex and multifactorial, involving a
degenerative cascade of events in which mechanical and genetic factors combine
in an age-related process. Evidence from a Japanese case–control study suggests
that genetic susceptibility is the major factor.

51. D  Reducing shank diameter increases the risk of


fatigue failure
• Pitch is the distance between threads
• Lead is the distance advanced by a single revolution
• Screw working length is the length of bone traversed by the screw
• Inner diameter (core/root diameter) is the diameter of the metal core of the
screw discounting the thread depth; it affects the strength of the screw (i.e. its
risk of fatigue failure)
• Outer diameter is the total diameter including thread depth
• Pull-out strength is maximised by having a large difference between core and
outer diameters (i.e. deeper threads) and having a fine pitch (i.e. more threads)

52. E  Is a good source of clotting factors


Autotransfusion is not a good source of clotting factors (as they become depleted)
and it is commonly employed when cement is used.

53. A  Medial stem pivot


With a lack of superomedial and inferolateral cement support the prosthesis pivots
around the medial stem eventually causing cement fracture in Gruen zones 2 and 6.

54. D  X-linked dominant


The inheritance patterns of common orthopaedic conditions are commonly tested.
Generally structural conditions (e.g. achondroplasia) are inherited in an autosomal
dominant manner and enzyme deficiencies (e.g. Gauchers disease) are inherited
in an autosomal recessive manner. The common X-linked recessive conditions are
muscular dystrophy (both Becker’s and Duchenne’s) and haemophilia. X-linked
dominant conditions are rare; however, both Hypophosphataemic rickets and Leri-
Weill dyschondrosteosis are inherited in this manner.
SBA Answers 27

55. A  Proteoglycan
Gadolinium allows accurate assessment of the amount of proteoglycan in the
articular cartilage.35

56. D  Pain
Recovery of deep cutaneous sensibility (pain caused by deep pressure) is the first
sign of nerve recovery.

57. E  Speed
Speeds test is undertaken with the elbow extended, forearm supinated and
humerus elevated to 60° – positive for long head of biceps pathology and SLAP
tears.

58. E  Ligament of Struthers


The ligament of Struthers runs between the humerus and the medial
epicondyle. Its proximal attachment to the humerus may be from a
supracondylar process or from the humerus itself. It is a possible site of
compression of the median nerve.

59. B  Paraesthesia along the dorso-radial side of the hand


Wartenberg’s syndrome is caused by compression of the superficial sensory
branch of the radial nerve between the tendons of brachioradialis and extensor
carpi radialis longus with forearm pronation. Symptoms include pain, numbness
and paraesthesia over the dorsoradial aspect of the hand. Provocative tests
include forceful forearm pronation for 60 seconds and a Tinel sign over the
nerve.

60. E  Kruskal–Wallis test


Here we are comparing two different groups that are not paired or matched. The
Herring classification system is an ordinal measurement (i.e. Grade A, B, or C, with
increasing severity).36
28 Chapter 2  SBA Answers

61. A  Kocher–Langenbeck
This pattern combines a transverse component with a stem which exits either through
the obturator ring or – in unusual cases – at various levels through the ischium.
Factors which must be considered when making the approach choice are:
• Level of anterior and posterior column fractures, i.e. transtectal, juxtatectal, or
infratectal
• Relative column displacement
• Presence and configuration of posterior wall involvement
• Associated marginal impaction
The majority of fracture patterns have predominant posterior column displacement
and therefore can be operated through a Kocher-Langenbeck approach. In cases
where there is significant displacement of both columns the decision must be
made whether to use a sequential ilioinguinal/Kocher-Langenbeck, or an extended
iliofemoral approach.
An alternative to the extended iliofemoral approach for complex patterns in older
patients would be the Kocher-Langenbeck with digastric trochanteric osteotomy
(trochanteric flip extension). This approach can be used with or without true
surgical dislocation. However, this alternative should not be considered in any way
less demanding than the extended iliofemoral.37

62. D  Pronation–external rotation


Weber C fractures occur with the foot in pronation when an external rotation
force is applied. The first injury is a failure of the medial side, either with a deltoid
ligament rupture or a medial malleolar fracture. The talus moves anteriorly as
it rotates externally. The talus causes the fibula to rotate externally around its
longitudinal axis, sequentially tearing the anterior and interosseous ligaments.
Finally the fibula fractures proximal to the syndesmosis.

63. B  Forefoot supination


A pes cavus deformity is associated with hindfoot varus, forefoot adduction,
plantar flexion of the first metatarsal, forefoot pronation and clawing of the toes.
References 29

References
1. Della Valle C, Parvizi J, Bauer TW, et al. American Academy of Orthopaedic Surgeons clinical practice
guideline on: the diagnosis of periprosthetic joint infections of the hip and knee. J Bone Joint Surg
[Am] 2011;93-A:1355-7.
2. Callaghan JJ, Brand RA, Pedersen DR. Hip arthrodesis: a long-term follow-up.J Bone Joint Surg [Am]
1985;67-A:1328-35.
3. D’Ippolito G, Diabira S, Howard GA, Roos BA, Schiller PC. Low oxygen tension inhibits osteogenic
differentiation and enhances stemness of human MIAMI cells. Bone 2006;39:513-22.
4. Sobhani A, Moradi F, Pasbakhsh P, et al. Effects of glucocorticoid on bone metabolism markers and
bone mineral density in rats. J Dent 2005;2:64-9.
5. Miller MD. Review of Orthopaedics. Fifth ed. Philadelphia: Saunders Elsevier, 2008.
6. Green DP, Hotchkiss RN, Pederson WC. Green’s operative hand surgery. Sixth ed. New York: Churchill
Livingstone, 2005.
7. FitzGerald MJT, Gruener G, Mtui E. Clinical neuroanatomy and neuroscience. Philadelphia: Saunders.
8. Snell RS. Clinical anatomy. Seventh ed. Philadeplphia: Lippincott Williams & Wilkins, 2003.
9. Iannotti JP, Hennigan S, Herzog R, et al. Latissimus dorsi tendon transfer for irreparable
posterosuperior rotator cuff tears: factors affecting outcome. J Bone Joint Surg [Am] 2006;88-A:342-8.
10. Salenius P, Vankka E. The development of the tibiofemoral angle in children. J Bone Joint Surg [Am]
1975;57-A:259-61.
11. Cohen SP, Williams KA, Kurihara C, et al. Multicenter, randomized, comparative cost-effectiveness
study comparing 0, 1, and 2 diagnostic medial branch (facet joint nerve) block treatment paradigms
before lumbar facet radiofrequency denervation. Anesthesiology 2010;113:395-405.
12. Kerr EE, Benson D, Schrot RJ. Coccygectomy for chronic refractory coccygodynia: clinical case series
and literature review. J Neurosurg Spine 2011;14:654-63.
12a. Haddad SL, Coetzee JC, Estok R, Fahrbach K, Banel D, Nalysnyk L. Intermediate and long-term
outcomes of total ankle arthroplasty and ankle arthrodesis. A systematic review of the literature. J
Bone Joint Surg Am 2007;89:1899 -905.
13. Miller MD. Review of orthopaedics. Fifth ed. Philadelphia: Saunders Elsevier, 2008.
14. Canale ST, Beaty JH, eds. Campbell’s Operative Orthopaedics. 11th ed. Philadelphia: Mosby Elsevier;
2007.
15. Patchell RA, Tibbs PA, Regine WF, et al. Direct decompressive surgical resection in the treatment of
spinal cord compression caused by metastatic cancer: a randomised trial. Lancet 2005;366:643-8.
16. Schwartz DT, alpert M. The malignant transformation of fibrous dysplasia. Am J Med Sci
1964;247:1-20.
17. Stener B. Displacement of the ruptured ulnar collateral ligament of the metacarpo-phalangeal joint of
the thumb. J Bone Joint Surg [Br] 1962;44-B:869-79.
18. Miller MD. Review of orthopaedics. Fifth ed. Philadelphia: Saunders Elsevier, 2008.
19. Joyce ME, Jingushi S, Bolander ME. Transforming growth factor-beta in the regulation of fracture
repair. Orthop Clin North Am 1990;21:199-209.
20. Mihalko WM, Krackow KA. Flexion and extension gap balancing in revision total knee arthroplasty.
Clin Orthop Relat Res 2006;446:121-6
21. Williams MM, Snyder SJ, Buford D Jr. The Buford complex: the “cord-like” middle glenohumeral
ligament and absent anterosuperior labrum complex: a normal anatomic capsulolabral variant.
Arthroscopy 1994;10:241-7.
22. Goergens ED, McEvoy A, Watson M, Barrett IR. Acute osteomyelitis and septic arthritis in children. J
Paediatr Child Health 2005;41:59-62.
23. McCarthy JJ, Dormans JP, Kozin SH, Pizzutillo PD. Musculoskeletal infections in children: basic
treatment principles and recent advancements. Instr Course Lect 2005;54:515-28.
24. Kang SN, Sanghera T, Mangwani J, Paterson JM, Ramachandran M. The management of septic arthritis in
children: systematic review of the English language literature. J Bone Joint Surg [Br] 2009;91-B:1127-33.
25. Sukeik M, Alshryda S, Haddad FS, Mason JM. Systematic review and meta-analysis of the use of
tranexamic acid in total hip replacement. J Bone Joint Surg [Br] 2011;93-B:39-46.
26. Alshryda S, Sarda P, Sukeik M, et al. Tranexamic acid in total knee replacement: a systematic review
and meta-analysis. J Bone Joint Surg [Br] 2011;93-B:1577-85.
30 Chapter 2  SBA Answers

27. Klonz A, Loitz D, Wöhler P, Reilmann H. Rupture of the distal biceps brachii tendon: isokinetic power
analysis and complications after anatomic reinsertion compared with fixation to the brachialis
muscle. J Shoulder Elbow Surg 2003;12:607-11.
28. Hutchinson MR, Bae TS. Reproducibility of anatomic tibial landmarks for anterior cruciate ligament
reconstructions. Am J Sports Med 2001;29:777-80.
29. Morrey BF, Dunn JM, Heimbach RD, Davis J. Hyperbaric oxygen and chronic osteomyelitis. Clin Orthop
Relat Res 1979;144:121-7.
30. Möller K, Sollerman C, Geijer M, Kopylov P, Tägil M. Avanta versus Swanson silicone implants in the
MCP joint: a prospective, randomized comparison of 30 patients followed for 2 years. J Hand Surg Br
2005;30:8-13.
31. Fischgrund JS. Orthopaedic Knowledge Update: No. 9. Rosemont: American Academy of Orthopaedic
Surgeons, 2008: 366.
32. Fischgrund JS. Orthopaedic Knowledge Update: No. 9. Rosemont: American Academy of Orthopaedic
Surgeons, 2008: 479.
33. Johnson KA, Strom DE. Tibialis posterior tendon dysfunction. Clin Orthop Relat Res 1989;23:196-206.
34. Matsui H, Kanamori M, Ishihara H, et al. Familial predisposition for lumbar degenerative disc disease: a
case-controlled study. Spine 1998;23:1029-34.
35. Gray ML, Burstein D, Kim YJ, Maroudas A. 2007 Elizabeth Winston Lanier Award Winner. Magnetic
resonance imaging of cartilage glycosaminoglycan: basic principles, imaging technique, and clinical
applications. J Orthop Res 2008;26:281-91.
36. No authors listed. BMJ. Study design and choosing a statistical test. http://www.bmj.com/about-bmj/
resources-readers/publications/statistics-square-one/13-study-design-and-choosing-statisti (date last
accessed 7 March 2013).
37. Mayo K, Oransky M, Rommens P, Sancineto C. AO Foundation. Acetabulum: T-type fractures with
Kocher-Langenbeck. http://tinyurl.com/bvkgvm4 (date last accessed 7 March 2013).
Chapter 3
Viva Questions:
Adult pathology
64. A 32-year-old woman presents with a history of pain and occasional clicking in
her right hip. She gives a history of a skiing injury to her right groin 7 months
earlier. This is her radiograph (Fig. 3.1).
A. Describe the abnormality on the radiograph in this patient with hip pain.
B. Describe the various angles that can be measured to document hip
morphology.
C. What further investigations should be performed to investigate the hip pain?
D. What are the surgical options for management?
E. What are the classification systems for more severe cases?

Figure 3.1
32 Chapter 3  Viva Questions

65. A 58-year-old keen runner presents with a history of hip pain especially after
running. This is the radiograph obtained in the clinic (Fig. 3.2).
A. Describe the abnormality on the radiograph.
B. Describe the likely presentation and mechanism of injury of this patient.
C. What further investigations should be performed?
D. What are the options for management?
E. Describe the blood supply to the femoral head.

Figure 3.2

66. A 28-year-old man presents with pain in his chest and lower back. This is his
radiograph (Fig. 3.3).
A. Describe the abnormality on the radiograph. What is your diagnosis?
B. How can we assess the severity of this condition?
C. What determines the prognosis?
D. What other investigation/s would you like to request and why?
E. Describe a classification system for this pathology.
F. What are the options for treatment?
Adult pathology 33

Figure 3.3

67. A. Describe the abnormalities on this radiograph (Fig. 3.4) taken one year after
revision hip arthroplasty.
B. Describe the stages of the pathological process seen around the right hip.
C. What is the common classification used for this disease process?
D. What can be done to reduce the incidence of this process?
E. What are the consequences of the other abnormality?

Figure 3.4
34 Chapter 3  Viva Questions

68. A 68-year-old woman presents with a history of pain in the region of her thumb.
This is her radiograph (Fig. 3.5).
A. Describe the radiograph. What is the diagnosis?
B. What are the causes of this condition?
C. What is the treatment for this condition?
D. What are the possible complications of surgical intervention in patients with
this condition?

Figure 3.5

69. A 42-year-old female presents with a history of lower back and right leg pain.
This is her radiograph (Fig. 3.6).
A. Describe the abnormalities seen in the radiograph.
B. What are the associations of this condition?
C. What is the approximate incidence of this abnormality in the general population?
D. What are the main surgical implications of this condition?
E. Describe the embryology of the spine.

Figure 3.6
Adult pathology 35

70. A 44-year-old microlight pilot sustained this injury (Fig. 3.7) in a crash landing.
A. Describe the radiograph.
B. There is an 8 cm clean laceration overlying the injury with a pale foot and no
pulses. Describe your treatment strategy for this patient.
C. What is the common classification of talar fractures and what is the likely risk
of nonunion in this case?
D. What is Hawkin’s sign and what is the pathogenesis of this phenomenon?
E. Discuss the blood supply to the talus.

Figure 3.7
36 Chapter 3  Viva Questions

71. A 72-year-old man presents with a long-standing history of knee pain, which
is affecting his daily life and recreational activities. This is his radiograph
(Fig. 3.8).
A. Describe the radiograph.
B. What other investigations are appropriate?
C. The patient has a significant fixed flexion deformity and fixed valgus. Assuming
surgical intervention is appropriate discuss what form of knee replacement
would be appropriate?
D. Describe a method of lateral ligament complex release and balancing the knee
for valgus knees?
E. What is the importance of the possible medial joint line opening seen in the
radiograph?

Figure 3.8
Adult pathology 37

72. A fit and well 65-year-old fell down stairs five years after a successful total hip
replacement. He presents with pain in his right hip and an inability to bear
weight. This is his radiograph (Fig. 3.9).
A. Describe the radiograph.
B. Describe a classification system for this injury that helps to guide
management.
C. Where does this injury fit with the classification system you have described?
D. How would you treat this injury?
E. What is the expected outcome following treatment?

Figure 3.9
38 Chapter 3  Viva Questions

73. A 75-year-old woman presents with acute pain in the right hip after attempting
to tie her shoelaces. She had a total hip replacement 12 months previously. This
is her radiograph (Fig. 3.10).
A. Describe the radiograph.
B. What factors influence the risk of dislocation following a total hip
replacement?
C. What are the options for management of this patient?
D. What is the prognosis if the patient underwent a revision for this problem?
E. What are the options for treatment for recurrent dislocation following abductor
insufficiency?

Figure 3.10
Adult pathology 39

74. A 54-year-old woman who underwent a mastectomy for breast cancer 12 months
ago followed by adjuvant chemotherapy presents with a two week history of
increasing mid-thoracic back pain. Over the last two days her legs have felt
weak and she has struggled to walk. There is no history of urinary or faecal
incontinence. She is otherwise fit and well.
This is the MRI of her spine (Fig. 3.11).
A. Describe the image.
B. What is the likely diagnosis and how would you manage this patient in the
A&E department?
C. What are the options for definitive management?
D. What factors govern the decision making process?

Figure 3.11
40 Chapter 3  Viva Questions

75. A 42-year-old woman presents with a history of anterior knee pain for the past
three years. She has difficulty climbing up and down the stairs and also finds
it fairly difficult to drive for long distances because of the pain. These are her
radiographs (Figs 3.12a to c).
A. Describe the radiographs.
B. What is the diagnosis and what is the aetiology?
C. What are the options for management in this situation?
D. What factors govern the decision making process?
E. If you decided to go down the arthroplasty route, what prosthesis would you
use and why?
F. How are patellofemoral joint replacements performing on the National Joint
Registry in the UK?

a b

c
Figure 3.12a to c
Chapter 4
Viva Answers:
Adult pathology
64A. Describe the abnormality on the radiograph in this patient with
hip pain.
The radiograph shows evidence of mild dysplasia with a reduced lateral
centre-edge angle of Wiberg and an abnormal femoral head extrusion index.
Asphericity of the right femoral head/neck junction consistent with cam type
femoroacetabular impingement is also noted.

64B. Describe the various angles that can be measured to document


hip morphology.
A number of angles can be measured around the hip to help establish pathology.
1. Acetabular inclination (Tönnis angle) – This is the angle between two lines:
a. a line from the most inferior point of the acetabular sourcil to the lateral
margin of the acetabular sourcil
b. a horizontal line running through the most inferior part of the sourcil. This
latter line is parallel to the transverse pelvic axes that can be determined
by a line connecting the base of both acetabular teardrops. A normal
Tönnis angle is between 0 and 10°. A decreased Tönnis angle can lead to
pincer form of FAI and an increased Tönnis angle can indicate structural
instability.
2. Sharp’s angle – This acetabular angle is creating by measuring the intersection
between the following two lines:
a. a horizontal line from the inferior aspect of one teardrop to the other and
b. a line from the inferior aspect of the teardrop to the superolateral margin
of the acetabulum A normal angle is between 33° and 38°. Higher angles,
especially those above 47°, suggest dysplasia.
3. Lateral centre-edge angle of Wiberg – This assess the superolateral coverage
of the femoral head. It is obtained by calculating the angle between two
lines:
a. a vertical line through the centre of the femoral head (perpendicular to the
transverse pelvic axis), and
b. a line from the centre of the femoral head to the superolateral margin of
the acetabular sourcil. Values of less than 20° indicate dysplasia and values
above 40° may notify pincer FAI.
42 Chapter 4  Viva Answers

4. Anterior centre edge angle of Lequesne – This is measured on the false profile
view. It is the angle between two lines:
a. a vertical line through the centre of the femoral head and
b. a line from the centre of the femoral head to the most anterior point of the
acetabulum. It is a measure of anterior coverage of the femoral head. An
angle less than 20 is suggestive of instability.
64C. What further investigations should be performed to investigate
the hip pain?
A cross-table lateral and an MRI of the right hip should be performed to
investigate the cause of the hip pain. The MRI scan can detect chondral/labral
pathology and extracapsular abnormalities that may be causative.

64D. What are the surgical options for management?


Surgical options include a diagnostic hip injection with EUA to confirm the intra-
articular origin of pain if in doubt, arthroscopic hip surgery (manage chondro-
labral pathology and excision of a cam impingement lesion). Another option is
open hip surgery +/- dislocation of the head/periarticular osteotomy.

64E. What are the classification systems for more severe cases?
The Crowe classification of hip dysplasia is based on the extent of proximal
migration of the femoral headi
1. less than 50% subluxation
2. 50 to 75%
3. 75 to 100%
4. greater than 100% subluxation
65A. Describe the abnormality on the radiograph.
There is a transcervical fracture of the right neck of femur. This is minimally
displaced on the AP view and is on the tension side of the neck.

65B. Describe the likely presentation and mechanism of injury of this


patient.
This is likely to be a stress fracture. The radiograph represents a female pelvis and at
the age of 58 the patient is very likely to post-menopausal. As the protective effects
of oestrogen on bone (promotes bone formation) are diminished post-menopause,
patients may develop osteoporosis and are more susceptible to fractures. In this
case, the stress fracture has likely resulted from repetitive microtrauma from
running. The presentation of stress fracture is normally insidious and slow due to
the repetitive microtrauma that occurs. These patients are normally very active and
will only seek medical attention when the pain becomes severe.

65C. What further investigations should be performed?


Blood tests should be performed including calcium, phosphate and alkaline
phosphatase to rule out causes other than osteoporosis. Urinary calcium should
Adult pathology 43

also be measured. A DEXA (dual energy X-ray absorptiometry) scan should


also be undertaken – however this would likely follow the initial management.
Additionally pre-emptive pre-operative bloods tests should be undertaken
including FBC, U&E and Group and Save.

65D. What are the options for management?


Options for management would depend on the full history and patient co-
morbidities and beliefs. As the patient is a keen runner it is likely that the patient
would like to return to that level of function, this would be very unlikely without
operative intervention.
Operative options include closed reduction and internal fixation, open
reduction and internal fixation and arthroplasty options. As the patient is 58
years old, bone preservation is preferred to arthroplasty and thus fixation
would be the operation of choice. Additionally, as the fracture appears
minimally displaced on the AP view, fixation is likely to be possible with a
closed reduction, leading to the least disruption of blood supply to the femoral
head. Fixation options would include cannulated screws or 2-hole dynamic hip
screw with a derotation screw.
The major concern with closed reduction and internal fixation is avascular
necrosis to the femoral head

65E. Describe the blood supply to the femoral head.


The blood supply to the femoral head is as follows:
Extracapsular arterial ring at the base of the femoral neck,
• formed posteriorly by large branch of medial femoral circumflex artery
• formed anteriorly by smaller branches of lateral femoral circumflex artery
• superior and inferior gluteal arteries have minor contributions
Ascending cervical branches:
• these give rise to retinacular arteries
gives rise to subsynovial intra articular ring
Artery of ligamentum teres:
• derived from obturator or medial circumflex femoral artery
• forms the medial epiphyseal vessels
• only small amount of the femoral head is nourished this artery
Epiphyseal blood supply:
• arises primarily from lateral epiphyseal vessels that enter head
posterosuperiorly
• vessels from medial epiphyseal artery entering through ligamentum teres
Metaphyseal blood supply:
• arises from extracapsular arterial ring
• arise from branches of ascending cervical arteries and subsynovial intra-articular
ring
44 Chapter 4  Viva Answers

66A. Describe the abnormality on the radiograph. What is your diagnosis?


The radiograph shows an abnormal right-sided lateral curvature of the thoracic
spine. The diagnosis in this 28-year-old is of adult scoliosis. This is most likely
to be due to pre-existent idiopathic adolescent scoliosis, but this also could be
secondary to a neuromuscular disease, a tumour or degenerative change.

66B. How can we assess the severity of this condition?


The severity can be assessed by performing a full history and examination to
elucidate the clinical effect of the scoliosis. The severity of the curve can be
radiologically assessed by measuring the Cobb angle. This is derived by drawing
intersecting perpendicular lines from the superior surface of the superior end
vertebra of the curve and from the inferior surface of the inferior end vertebra
of the curve. Vertebral rotation can also be measured to give an indication of
severity.

66C. What determines the prognosis?


The severity of the curve would determine prognosis. Curve progression is
unlikely if the Cobb angle is less than 30°. Progression is highest for right sided
thoracic curves > 50° (1 mm/year). The underlying causative reason behind
the scoliosis would naturally affect prognosis. A simple idiopathic adolescent
scoliosis at this stage at the age of 28 would have a much better prognosis than a
neuromuscular or a tumour-related curve.

66D. What other investigation/s would you like to request and why?
Other investigations that we be appropriate would include posteroanterior and
lateral radiographs of the whole spine with the patient standing. Lateral bending
films are also of benefit if planning further treatment. An MRI would be essential
to rule out a syrinx and also if there were any features to suggest a malignancy or
any neurological findings on examination.

66E. Describe a classification system for this pathology.


The classification of King et al1 is used to describe thoracic curves as follows;
• King I – lumbar curve larger than thoracic curve, which is more flexible on
bending films.
• King II – thoracic curve is larger than lumbar curve, less flexible and the
thoracic rib hump is larger than the lumbar rotational prominence.
• King III – a thoracic scoliosis where the lumbar curve does not cross the
midline.
• King IV – single long thoracic curve
• King V – double structural thoracic curve.
66F. What are the options for treatment?
The options for treatment are non-operative and operative. In this case, if
no suspicious features were discovered on history and examination, simple
observation and repeat interval radiography of this < 30° curve would be
Adult pathology 45

indicated. Analgesia and physiotherapy would also be of benefit. Other options


for younger patients with more severe curves would include orthotic bracing
or casting and possibly operative intervention. This may include an anterior/
posterior approach with instrumented (Harrington, CD and Luque) fusion.

67A. Describe the abnormalities on the radiograph, taken one year


after revision hip arthroplasty.
There is heterotopic ossification (HO) around the right hip joint. I would classify
this as Brooker Grade III.2 There is also significant osteoarthritis of the left hip and
there is also an element of leg length discrepancy.

67B. Describe the stages of the pathological process seen around the
right hip.
HO is, by definition, the formation of bone within soft tissue. The transformation
of primitive cells of mesenchymal origin, present in the connective tissue septa
within muscle, into osteogenic cells is thought to be the pathogenesis. Chalmers
et al3 proposed three conditions needed for HO: osteogenic precursor cells,
inducing agents, and a permissive environment.
The heterotopic bone may begin some distance from normal bone, later moving
toward it. Studies have also shown that muscle injury alone will not cause the
ectopic ossification, concomitant bone damage also being required.4 Other
contributing factors include hypercalcemia, tissue hypoxia, changes in sympathetic
nerve activity, prolonged immobilisation and imbalance of PTH and calcitonin.
Early in the course of HO, oedema with exudative cellular infiltrate is present,
followed by fibroblastic proliferation and osteoid formation. The development of
HO is extra-articular and bone forms in the connective tissue between the muscle
planes and not within the muscle itself. The new bone can be continuous with
the skeleton but generally does not involve the periosteum. Mature HO shows
cancellous bone and mature lamellar bone, vessels, and bone marrow.

67C. What is the common classification used for this disease process?
The Brooker classification2 is used and it is based on an anteroposterior
radiograph.
• Class I: represents islands of bone in soft tissues around the hip.
• Class II: includes bone spurs in pelvis or proximal end of femur leaving at least
1 cm between the opposing bone surfaces.
• Class III: represents bone spurs that extend from pelvis or the proximal end of
femur, which reduce the space between the opposing bone surfaces to less
than 1 cm.
• Class IV: indicates radiologic ankylosis of the hip.
67D. What can be done to reduce the incidence of this process?
Non-steroidal anti-inflammatory drugs (NSAIDs) have been shown to reduce
the incidence of HO.5 Pre-operative radiation has also been shown to prevent
HO. Pakos et al6 demonstrated the efficacy of combined radiotherapy
46 Chapter 4  Viva Answers

and indomethacin in preventing heterotopic ossification after total hip


arthroplasty.
Prophylactic measures against HO after hip and knee replacement should be
administered before the fifth postoperative day, optimally within 24 to 48 hours.7
Meticulous clearance of bone debris and avoidance of muscle damage have also
been shown to prevent HO.8

67E. What are the consequences of the other abnormality?


The other abnormality on the radiograph is leg length discrepancy. This can lead
to a limp, pain in the other joints, potentially sciatic nerve injury and more likely
legal consequences.

68A. Describe the radiograph. What is the diagnosis?


The radiograph of the wrist shows evidence of decreased joint space and
subchondral sclerosis of the joints between the scaphoid, the trapezium and the
trapezoid. The diagnosis is arthritis of the scapho-trapezio-trapezoid (STT) joint.

68B. What are the causes of this condition?


The causes of STT arthritis are age-related osteoarthritis, post-traumatic arthritis
(including STT arthritis post-scaphoid ORIF), abnormal trapezio-trapezoidal
inclination, capito-trapezial ligament laxity and rotatory subluxation.

68C. What is the treatment for this condition?


Non-operative treatment options include rest, splinting, hand therapy, oral anti-
inflammatory medication, and intra-articular steroid injection. Surgical options
for ongoing symptoms will depend on the extent of symptoms, arthritis and
the age and functional demands of the patient. The options include excision
arthroplasty, interposition arthroplasty (using tendon or pyrocarbon implant) and
arthrodesis. The latter is preferred in the young active patient as it is more likely
to result in a stable thumb for high power manual work.
Arthrodesis provides a stable column across the carpus and is a definitive
operation to deal withpain at the level of the STT joint. It is contra-indicated with
radio-scaphoid OA and has a variously quoted non-union rate of 4% to 29%.9
Furthermore, there is the risk of altering wrist kinematics permanently resulting
in secondary arthrosis of other joints such as the radiocarpal joint as well as 1st
carpometacarpal joint osteoarthritis both of which have been reported in the
literature. Kirk-Watson however found no further OA at five years. A key to a
successful fusion is maintaining the relationship between individual carpal bones
and correcting any secondary deformities. Some of the tips to a successful fusion
include:10
• Articular surface must be thoroughly denuded of cartilage
• Original external dimensions of the STT joint must be maintained
• Radial styloidectomy must be carried out to prevent impingement
Adult pathology 47

Excision of the distal end of the scaphoid is also a useful procedure in the
management of symptomatic STT OA for the following reasons:11
• Relatively simple
• Allows flexion and extension of the scaphoid in radial/ulnar deviation therefore
does not lead to impingement
• Can be combined with soft-tissue interposition
• Can lead to early mobilisation
However, excision of the distal pole of the scaphoid can increase carpal instability
and any pre-existing DISI (dorsal intercalated ligament instability) may be
exaggerated.

68D. What are the possible complications of surgical intervention in


patients with this condition?
Surgical intervention may lead to a number of complications including
ongoing pain, neurovascular injury, infection, thumb instability, deformity,
pseudoarthrosis, carpal translation and progressive adjacent arthritis.

69A. Describe the abnormalities seen in the radiograph.


This is an example of lumbosacral transitional vertebra (LSTV); there is an
abnormal joint between right lateral process and ileum/sacrum with evidence of
sclerosis of this joint.

69B. What are the associations of this condition?


Lumbosacral transitional vertebrae are associated with cervical ribs, altered nerve
root functioning and facet joint arthrosis. These patients are at higher risk of disc
degeneration and back pain.12

69C. What is the approximate incidence of this abnormality in the


general population?
A systematic review of studies from 200712 has shown a total prevalence of
7.5% sacralisation in the general population. There is a prevalence of 5.5% for
lumbarisation. The total for both is under 13%.

69D. What are the main surgical implications of this condition?


Treatment of this condition is largely conservative. The majority of patients are
asymptomatic. If symptoms occur, a steroid and local anaesthetic injection is the
first line treatment and can provide pain relief.
There is little data regarding surgical interventions and surgery may be indicated
in radiculopathy, degenerative disc disease higher up the spine and in certain
chronic pain cases.
Finally, failure to recognise this can lead to errors in lumbar numbering in surgery
for other lumbar spinal problems.
48 Chapter 4  Viva Answers

69E. Describe the embryology of the spine.


During the pre-cartilaginous stage sclerotomes appear as paired condensations
around the notochord and neural tube in a four-week-old embryo. These sclerotomes
consist of cells, which move cranially to form the intervertebral discs, and cells, which
move caudally to form the primordial of the body of the vertebra. Where vertebral
bodies surround the notochord it degenerates. Between the vertebrae the notochord
expands to form the nucleus pulposus. Mesenchymal cells that surround the neural
tube become the neural arch (primordium of the vertebral arch).
There are two primary ossification centres in the vertebrae – ventral and dorsal.
Ossification becomes evident in the neural arches at eight weeks.13

70A. Describe the radiograph.


This is a lateral radiograph of the distal tibia/fibula and hindfoot/midfoot. It
shows a talar fracture with extrusion of the talus anteriorly. The tibiotalar, subtalar
and talo-navicular joint are dislocated.

70B. There is an 8 cm clean laceration overlying the injury with a pale


foot and no pulses. Describe your treatment strategy for this
patient.
The mechanism of injury and the radiograph are consistent with an extremely
high energy injury and therefore I would manage this patient according to
ATLS protocol, identifying life and then other limb-threatening injuries. If this is
deemed to be an isolated injury, the extruded talus and vascular compromise is
a surgical emergency and would need emergency treatment in theatre. I would
also assess for signs of compartment syndrome and contact plastic surgical and
vascular surgical teams to assess the patient and to be present in theatre. The
fracture would be treated as any open fracture in the emergency department
with intravenous antibiotics, tetanus cover, irrigation if any gross contamination,
pictorial documentation and temporary splintage.
In theatre, a thorough debridement and washout of the laceration would be
performed by joint orthopaedic and plastic surgical teams and an open reduction
of the extruded fragment performed. A vascular surgeon would be present to
assess restoration of pulses post reduction and consideration of an on-table
angiogram would be made. A negative pressure vacuum dressing would be
applied to the open wound. Consideration could be made at the time for
definitive fixation and soft-tissue coverage, but given the high energy injury and
associated swelling, I feel a spanning external fixator would be the treatment
of choice. This would be combined with foot fasciotomies if indicated. At a later
stage, further CT imaging of the injury would be performed to plan definitive
fixation. Of note, if the extruded fragment was significantly contaminated and
had no soft-tissue attachments, then consideration could be made to removal
of the fragment and the use of a cement spacer, with a view to a later pan-
talar fusion. This is because of the high infection rates associated with retained
contaminated, denuded extruded fragments.
Adult pathology 49

70C. What is the common classification of talar fractures and what is


the likely risk of nonunion in this case?
Talar fractures can be classified anatomically into head, neck, body, lateral and
posterior process fractures. The most common fracture is one of the talar neck, for
which the Hawkins classification is prognostic. Delayed union rates of up to 15% have
been noted but non-union is rare. The more common problem is that of avascular
necrosis (AVN) and this varies according to the Hawkins classification as follows.
• Hawkins I – undisplaced neck fracture – AVN risk 0 to 13%
• Hawkins II – displaced with subtalar subluxation/dislocation AVN 20 to 50%
• Hawkins III – displaced with subtalar and tibiotalar dislocation AVN 50 to 100%
• Hawkins IV – displaced with subtalar/tibiotalar/talonavicular dislocation 80 to
100%
70D. What is Hawkin’s sign and what is the pathogenesis of this
phenomenon?
Hawkin’s sign is the appearance of reduced subchondral bone density in the talar
dome six to eight weeks following injury. This subchondral resorption indicates
that vascularity to the talus is sufficient to allow normal resorption associated
with disuse osteopaenia.

70E. Discuss the blood supply to the talus.


Talar blood supply is from various sources:
1. Artery of the tarsal canal (dominant supply to body) – branch of posterior
tibial artery
2. Deltoid branch of posterior tibial artery (may be only remaining supply and
must be preserved during fixation)
3. Anterior tibial artery – supplies head and neck
4. Perforating peroneal arteries via the artery of the tarsal sinus (head and neck)
71A. Describe the radiograph
This is an AP radiograph of the right knee. It shows a valgus deformity with loss
of the lateral compartment joint space and lateral compartment osteophytes,
consistent with lateral compartment osteoarthritis of the knee.

71B. What other investigations are appropriate?


Other appropriate investigations would include a lateral radiograph of the knee
(AP and extension lateral weight-bearing radiographs), flexion lateral (90° to
100°), Merchant’s view, and an AP radiograph of the pelvis if there were any
concerns regarding hip osteoarthritis. Full length standing radiographs of the
lower limbs including the hip and ankle joints may be appropriate if deformity is
present, in order to determine the mechanical axis of the lower limbs.
If considering an operation then routine blood group and save and an ECG would
be required as a minimum.
50 Chapter 4  Viva Answers

71C. The patient has a significant fixed flexion deformity and fixed
valgus. Assuming surgical intervention is appropriate discuss
what form of knee replacement would be appropriate?
If the patient has a significant fixed flexion deformity and fixed valgus this
will require soft-tissue releases in order to correct the deformity. The type of
knee implant required depends on the extent of the soft-tissue release and
the competence of the ligaments following the release. In the valgus knee the
competence of the medial collateral ligament (MCL) is a factor that should
be assessed pre-operatively, as well as whether or not the FFD and valgus are
correctable.
If the MCL and the lateral collateral ligament (LCL) are competent after the
soft-tissue releases and bone cuts have been made, an unconstrained TKR
prosthesis may be used. This may be PCL retaining or posterior stabilised (PCL
substituting). If the PCL is attenuated and/or if the joint line is significantly
altered, a posterior stabilised design is recommended. If a PCL retaining implant
is planned, then a PCL substituting option should always be available at the
time of surgery.
If the MCL is attenuated then a more constrained prosthesis will be required.
If the LCL has to be released then, again, a more constrained prosthesis will be
required. The next level of constraint is a constrained non-hinged prosthesis. This
implant has a large central post that substitutes for the MCL or LCL function.
The use of a constrained non-hinged prosthesis for complete MCL deficiency is
controversial. If the MCL is completely deficient, then all of the valgus forces are
placed upon the polyethylene post leading to post breakage.
Any form of constrained knee prosthesis should use a long medullary stem for
both the femoral and tibial components in order to reduce the stress experienced
at the bone-cement-prosthesis interface.

71D. Describe a method of lateral ligament complex release and


balancing the knee for valgus knees.
This may or may not be approached via the lateral parapatellar approach. Lateral
release should begin with removal of the lateral osteophytes off the femur and
tibia and release of the lateral capsule off the tibia. If the knee is tight laterally in
extension the iliotibial band can be released either sub-periosteally off Gerdy’s
tubercle, or by Z-lengthening or “pie-crusting”. The next structure to be released
is depopliteus tendon, which is released sub-periosteally off the lateral condyle
of the femur. In a large > 15° valgus deformity these releases will usually be
necessary. If the knee still remains tight laterally the next step would be to
release the LCL sub-periosteally off the lateral condyle of the femur. At this stage
a constrained knee prosthesis should be considered and one should be aware
of the possibility of stretching the peroneal nerve when correcting large valgus
deformities.
Adult pathology 51

71E. What is the importance of the possible medial joint line opening
seen in the radiograph?
The significance of the possible medial joint line opening is that this could
represent deficiency of the MCL. In this situation one should have the option of a
constrained prosthesis available when it comes to surgery.

72A. Describe the radiograph.


This is an anteroposterior radiograph of the right hip. It shows a peri-prosthetic
femoral fracture around a cemented total hip replacement (tapered stem). The
acetabular component is cemented and appears to be well fixed with acceptable
inclination. There is a degree of heterotopic ossification. The femoral fracture
involves the cement mantle and there is subsidence of the stem within the
cement mantle, indicating that the stem is loose.

72B. Describe a classification system for this injury that helps to guide
management.
Vancouver classification of peri-prosthetic femur fractures.14,15 The Vancouver
classification assimilates three key factors: fracture location, stability of the
implant, and the surrounding bone stock (Table 4.1). Anatomical fracture location
groups divides these fractures into three categories: Type A occurring around the
trochanteric region, Type B near or just distal to the femoral stem, and Type C well
below the femoral stem. Studies have found that the Vancouver classification is
reliable, reproducible, and valid.
One limitation of the system is that plain radiographs may not always provide
enough information to distinguish between type B1, type B2, and type B3
fractures. If there is any question pertaining to implant stability, it should
be assessed intra-operatively. Therefore pre-operative planning is essential.
Appropriate positioning (supine versus lateral), availability of equipment for

  Table 4.1

Type Subtype Description Treatment


A AL Lesser trochanter Conservative (consider ORIF if large segment of
medial cortex involved)
AG Greater trochanter Conservative with abduction precautions
(consider ORIF if displaced > 2.5 cm)

B B1 Well-fixed prosthesis ORIF with or without cortical strut allograft

B2 Prosthesis loose Revision THR with long-stem prosthesis

B3 Prosthesis loose with Revision THR and augmentation of bone stock


poor bone stock with allograft or oncologic prothesis

C Fracture well below tip ORIF


of the prosthesis
52 Chapter 4  Viva Answers

fracture fixation, and revision femoral components is critical in allowing the


surgeon options if the implant is found to be more or less stable than was
interpreted pre-operatively.

72C. Where does this injury fit with the classification system you have
described?
This is a Vancouver type B2. The fracture involves the femoral stem and the
cement mantle is fractured with resulting subsidence of the prosthesis. The bone
stock appears good, with no obvious deficiency/lysis.

72D. How would you treat this injury?


According to the Vancouver classification, I would perform a revision total
hip replacement using a long-stem prosthesis. Prior to the operation I would
ascertain the implant type and sizes of all of the components. I would have all the
equipment and prostheses available to enable me to deal with any intra-operative
eventuality. I would approach the hip through the old incision, curving this
posteriorly at its proximal end to perform a posterior approach.
I would open the fracture site and expose the cement mantle and stem using
an extended trochanteric osteotomy if necessary. I would remove as much
cement as possible with an osteotome. Distally this could be done with a drill and
sequential reaming or by using an OSCAR cement removal system or osteotomes.
I would reconstitute the femoral canal with cerclage cables. I would then plan to
use a long stem modular, uncemented, tapered, fluted revision prosthesis, aiming
for a good distal press-fit after reaming the femoral canal.
If this cannot be achieved, further options would be either a long cemented
femoral stem or a distally locking stem.
I would assess the acetabular component for wear and stability after removal
of the femoral stem. If loose, or if there is evidence of polyethylene wear I
would remove the acetabular component and cement. I would then assess the
remaining bone stock and decide upon either a further cemented or uncemented
acetabular component.

72E. What is the expected outcome following treatment?


Springer et al16 reported a survival of 90% at five years and 79.2% at ten years for
Vancouver type B peri-prosthetic femoral fractures with revision or removal of the
femoral implant for any reason as the end point. Those treated with uncemented,
extensively porous-coated implants having the best results. Corten et al17
reported no revisions at 46 months follow-up after treatment of 31 B2 fractures
with a cemented long-stem. Similarly, Fink et al18 reported all excellent results
in 22 B2 and B3 fractures treated with a modular, uncemented, tapered, fluted
revision prosthesis. A review of 1049 peri-prosthetic fractures of the femur from
the Swedish National Hip Arthroplasty Register found that the risk of failure of
treatment was reduced for Vancouver type B2 injuries if revision of the implant
Adult pathology 53

was undertaken or revision and open reduction and internal fixation were
performed.19

73A. Describe the radiograph.


There is a dislocation of the hip in a cemented total hip replacement. There is also
an avulsion of the greater trochanter possibly due to non-union and broken and
loose cerclage wires.

73B. What factors influence the risk of dislocation following a total hip
replacement?
Dislocation occurs in around 3.1% of primary total hip replacements and in 10%
to 15% of revision total hip replacements. If occurring in the first year it is likely to
be due to the surgical technique. Factors associated with an increased risk can be
divided into implant factors, patient factors and surgical factors.
• Patient factors include patients with neuromuscular problems, females,
alcoholism, THR for trauma, soft-tissue laxity, DDH, osteonecrosis, takedown of
arthrodesis, age > 70 years, and trochanteric migrations.
• Implant factors include a reduced offset, smaller head size, wear of the
polyethylene, an inappropriate head neck ratio and loosening.
• Surgical factors include malpositioning of the implant, soft-tissue, bony or
cement impingement, a less experienced surgeon and following a revision
THR.
Dislocations occurring after five years are classified as late dislocations. Von
Knoch et al20 found that risk factors for this included component malposition, a
younger patient age at index arthroplasty, substantial trauma, wear of more than
2 mm and onset of cognitive or neurological decline. The late dislocation rate
from the Mayo clinic was 0.8%.

73C. What are the options for management of this patient?


The patient would require a reduction in the first instance. Following reduction
definitive management could be conservative or surgical depending on
the patient. Surgical management would include a revision of the total hip
arthroplasty and I would consider using a larger head size and also a constraint
if necessary to prevent further dislocations. The trochanter would need to be
reattached with a trochanteric claw or a cobra plate.

73D. What is the prognosis if the patient underwent a revision for this
problem?
There is a high rate of greater trochanter non-union in this case. Hamadouche et
al21 in a study of 72 greater trochanter avulsions were able to achieve 51 osseous
unions following claw plate fixation (70%).
54 Chapter 4  Viva Answers

73E. What are the options for treatment for recurrent dislocation
following abductor insufficiency?
Conversion to hemiarthroplasty with a larger femoral head which is indicated for
soft-tissue deficiency or dysfunction. This is contraindicated if acetabular bone
stock is compromised.
Conversion to a constrained acetabular component. Indicated for recurrent
instability with a well-positioned acetabular component due to soft-tissue
deficiency or dysfunction.
Resection arthroplasty (Girdlestone procedure) may be undertaken when all
options have been exhausted and there is a significant amount of bone loss and
soft-tissue deficiency including for psychiatric patients who are dislocating for
secondary gain.

74A. Describe the image.


This is a T2-weighted sagittal MRI of the thoracic spine. There are multiple lesions
in the vertebral bodies with vertebral collapse of T4 and encroachment of the
cord at T6 representing spinal cord compression. There is also an increase in the
thoracic kyphosis.

74B. What is the likely diagnosis and how would you manage this
patient in the A&E department?
The likely diagnosis is metastatic vertebral lesions from breast cancer, now
causing spinal cord compression. I would take a full history and examination,
especially neurological examination. Analgesia would be offered and unless
contraindicated a loading dose of at least 16 mg of dexamethasone would be
given as soon as possible after assessment. This would be followed by a short
course of 16 mg dexamethasone daily while treatment is being planned. I would
also consider immobilisation of the spine if unstable and an urgent referral to the
oncologists and the spinal surgeons.

74C. What are the options for definitive management?


Management is guided by NICE guidelines. Conservative management will
include radiotherapy and this may be suitable for those requiring definitive
treatment or who are unsuitable for surgery. If surgery is appropriate in
patients with (metastatic spinal cord compression) MSCC, attempt to achieve
both spinal cord decompression and durable spinal column stability. Consider
vertebral body reinforcement with cement for patients with MSCC and
vertebral body involvement who are suitable for instrumented decompression
but are expected to survive for less than one year. Consider vertebral body
reconstruction with anterior bone graft for patients with MSCC and vertebral
body involvement who are suitable for instrumented decompression, are
expected to survive for one year or longer and who are fit to undergo a more
prolonged procedure.
Adult pathology 55

  Table 4.2

Surgery more likely Radiotherapy more likely


Prognosis > 3 months < 3 months
Tumour burden Low High
Spine stability Unstable spine Stable
Diagnosis New primary Established disease
Levels of compression One Several
Performance Status Good Poor
Time since onset of neurology < 48 hours > 48 hours

74D. What factors govern the decision making process?


As mentioned above a number of factors influence decision-making including
fitness for surgery and life expectancy. Surgical factors may include degree of
kyphosis, surrounding bone quality and associated neurology. Vertebroplasty
or kyphoplasty can be considered for vertebral metastases with no evidence of
MSCC or spinal instability.22

75A. Describe the radiographs.


These are anteroposterior (AP), lateral and skyline radiographs of both knees.
The AP views show an osteophyte of the left lateral femoral condyle, with
well preserved joint spaces in the medial and lateral compartments. There are
osteophytes of the tibial spines in both knees. In both knees the patella appears
to be sitting more laterally than normal.
On the lateral views there are signs consistent with osteoarthritis of the
patellofemoral joints (PFJ), with osteophytes of the patellae and femoral condyles
at the articular margins.
The skyline views show loss of the normal joint spaces, with lateral patellofemoral
joint space narrowing, osteophytes, cysts and sclerosis. The patellae are sitting
laterally in the trochlear grooves and there is concave erosion of the lateral facets.

75B. What is the diagnosis and what is the aetiology?


The diagnosis is patellofemoral osteoarthritis.
Aetiology
Factors influencing the magnitude and distribution of PFJ pressure have
important implications. The PFJ reaction force can reach more than three times
body weight during stair ascent and descent and seven to eight times body
weight during squatting activities.
PFJ malalignment leads to increased contact pressure on an individual facet
(lateral tilt of the patella leads to increased contact pressure on the lateral facet).
56 Chapter 4  Viva Answers

Factors affecting alignment include:


• Passive structures - osseous anatomy and soft-tissue constraints.
• Active structures - medial (vastus medialis obliquus (VMO)) and lateral
quadriceps (vastus lateralis (VL)).
• Overall lower limb alignment.
Osseous abnormalities include a shallow or dysplastic femoral trochlea or patella
alta.
Soft-tissue abnormalities include an incompetent medial patello-femoral
ligament (MPFL) or a tight lateral retinaculum, particularly the two distal
expansions of the iliotibial band.
Younger patients with PFJ pain have demonstrated a delayed onset of VMO
activity relative to the VL during a monosynaptic reflex (patellar tap), during
isokinetic knee extension and during a stair-stepping task. People with
generalised knee OA do not exhibit a delayed onset of VMO relative to VL.
Lower limb alignment may affect patellar tracking by altering the relative position
of the femoral trochlea and changing the tension in soft-tissues. Experimentally
induced femoral internal rotation (anteversion) or tibial external rotation have
been associated with increased lateral patellar tilt and rotation and increased
lateral PFJ pressure.
Valgus malalignment is associated with isolated moderate to severe PFJ OA, and
is associated with an increased risk of disease progression.
Experimental increases in the Q-angle shifts the PFJ contact area laterally and
further increases pressures within the lateral facet. Increases in the lateral
compartment contact pressures correlate with the pattern of PFJ osteoarthritis
(OA), in which the lateral compartment is most commonly affected.
Knees with a more laterally positioned patella and greater patella tilt laterally
demonstrate a higher prevalence of PFJ OA.23 Work by the same authors in
Caucasian and African-American people shows that patella subluxation is
associated with knee pain severity and risk of disease progression.24
Significantly greater lateral tilt of the patella is seen in patients with isolated PFJ
OA compared with those with concurrent tibiofemoral joint (TFJ) OA.
Macro/Microtrauma
28% of patients with isolated PFJ OA reported a history of patellar dislocation/
subluxation, compared with none of the patients with combined PFJ/TFJ
OA,25 suggesting that a history of patella dislocation/subluxation may be a
predisposing factor for PFJ OA.
Obesity has been associated with PFJ OA.26
There is no definite evidence of gender preponderance.27,28
Adult pathology 57

75C. What are the options for management in this situation?


Management options are broadly non-operative or operative:
Non-operative
These should always be recommended as the first line of treatment:
• Activity modification
• Weight loss
• Physiotherapy; core stability, posture, VMO strengthening, taping
• Analgesia/anti-inflammatory medications
• Intra-articular injections – corticosteroid or hyaluronic acid
• Other preparations, including glucosamine, chondroitin and diacetylrhein
Operative
May consist of one or a combination of the following:
• Arthroscopic debridement/washout/chondroplasty/microfracture/
osteochondral allograft
• Partial lateral facetectomy
• Extensor mechanism realignment procedures (Fulkerson/Maquet procedures)/
lateral release
• Patellofemoral arthroplasty (PFA)
• Total knee arthroplasty (TKA)
75D. What factors govern the decision making process?
Factors governing the management include:
Patient factors – general:
• Age and activity levels
• Patient demands and expectations
• BMI
• Co-morbidities
• Occupation
Patient factors – disease-specific:
• Degree of cartilage loss/bone erosion
• Patellar alignment – tilt/height/soft-tissue stabilisers and quadriceps function/
tibial tubercle trochlea distance (TTTD)
• Limb alignment
• Presence of trochlear dysplasia
• Tibiofemoral joint disease
Surgeon factors:
• Experience with various patellofemoral procedures
75E. If you decided to go down the arthroplasty route, what prosthesis
would you use and why?
In this case the patient is relatively young and female. We have no information on
the general patient factors listed in the previous answer.
58 Chapter 4  Viva Answers

Arthroplasty options are TKA or PFA:


Total knee arthroplasty
Because of its relationship with patellofemoral instability, total knee arthroplasty
in patients with isolated patellofemoral osteoarthritis is a technically more
demanding procedure. A systematic review of treatments for isolated
patellofemoral osteoarthritis in 2010 found low quality, weak evidence for
the use of TKA.29 Furthermore, there is little information in the literature on
patellar resurfacing in TKA for this indication. TKA with patellar resurfacing
gives satisfactory five to seven year results. However, anterior knee pain can be
problematic after TKA.
Patellofemoral arthroplasty
The same systematic review reported that the clinical results are related to
prosthetic design, surgical technique, patient selection and indication, and
length of follow-up, and have shown good to excellent three- to 17-year results in
two-thirds to all patients.29
Progression of femorotibial osteoarthritis, malposition of the prosthesis, and
wear or loosening may result in failure of the patellofemoral arthroplasty. It
was also reported that patellofemoral arthroplasty does not have a negative
effect on the outcome of later total knee arthroplasty. Poor prognostic markers
include evidence of tibiofemoral osteoarthritis prior to surgery, BMI > 30, Prior
meniscectomy, patella alta or baja, ligamentous instability.30
I would perform a patellofemoral arthroplasty in this patient (Avon implant).
TKA with patellar resurfacing results in predictable and durable good results.
However, for osteoarthritis involving only one compartment, it may be too
aggressive, especially in this relatively young patient. The reported outcome
after patellofemoral arthroplasty in selected patients is good to excellent in the
literature. Total knee replacement can be performed later if painful femorotibial
osteoarthritis develops.

75F. How are patellofemoral joint replacements performing on the


National Joint Registry in the UK?
In the 9th annual report of the national joint registry (NJR), 201231 the median
age of patients undergoing patellofemoral arthroplasty is lower than for
other knee arthroplasty operations. The eight-year revision rate was 14.7% for
patellofemoral knee replacements, considerably higher than TKA. They had the
highest three-year, five-year and eight-year revision rates. In comparison, at eight
years the all-cemented TKA revision rate was 2.82%. Pain and asceptic loosening
were the most common reasons for revision. The Avon patellofemoral implant
has the best record – 10.4% revision rate at seven years.
References 59

References
1. King HA, Moe JH, Bradford DS,Winter RB. The selection of fusion levels in thoracic idiopathic
scoliosis. J Bone Joint Surg [Am] 1983;65:1302-13.
2. Brooker AF, Bowerman JW, Robinson RA, Riley LH Jr. Ectopic ossification following total hip
replacement: incidence and a method of classification. J Bone Joint Surg [Am] 1973;55-A:1629-32.
3. Chalmers J, Gray DH, Rush J. Observations on the induction of bone in soft tissues. J Bone Joint
Surg [Br] 1975;57:36-45.
4. Garland DE. A clinical perspective on common forms of acquired heterotopic ossification. Clin
Orthop Relat Res 1991;263:13-29.
5. Fransen M, Neal B, Cameron ID, et al. Determinants of heterotopic ossification after total hip
replacement surgery. Hip Int 2009;19:41-6.
6. Pakos EE, Tsekeris PG, Paschos NK, et al. The role of radiation dose in a combined therapeutic
protocol for the prevention of heterotopic ossification after total hip replacement. J BUON
2010;15:74-8.
7. Iorio R, Healy WL. Heterotopic ossification after hip and knee arthroplasty: risk factors,
prevention, and treatment. J Am Acad Orthop Surg 2002;10:409-16.
8. Pavlou G, Salhab M, Murugesan L, et al. Risk factors for heterotopic ossification in primary total
hip arthroplasty. Hip Int 2012;22:50-5.
9. Rogers WD, Watson HK. Degenerative arthritis at the triscaphe joint. J Hand Surg Am
1990;15:232-5.
10. Watson HK, Hempton RF. Limited wrist arthrodesis. I: the triscaphoid joint. J Hand Surg
1980;5:320-7.
11. Weiss KE, Rodner CM. Osteoarthritis of the wrist. J Hand Surg Am 2007;32:725-46.
12. Bron JL, van Royen BJ, Wuisman PI. The clinical significance of lumbosacral transitional anomalies.
Acta Orthop Belg 2007;73:687-95.
13. Moore KL, Persaud TVN, Torchia MG. Before we are born: essentials of embryology and birth
defects. 7th ed. Philadelphia: Elsevier Health Sciences, 2007.
14. Duncan CP, Masri BA. Fractures of the femur after hip replacement. Instr Course Lect 1995;44:
293-304.
15. Masri BA, Meek RM, Duncan CP. Periprosthetic fractures evaluation and treatment. Clin Orthop
Relat Res 2004;420:80-95.
16. Springer BD, Berry DJ, Lewallen DG. Treatment of periprosthetic femoral fractures following total
hip arthroplasty with femoral component revision. J Bone Joint Surg [Am] 2003;85-A:2156-62.
17. Corten K, Macdonald SJ, McCalden RW, Bourne RB, Naudie DD. Results of cemented femoral
revisions for periprosthetic femoral fractures in the elderly. J Arthroplasty 2012;27:220-5.
18. Fink B, Grossmann A, Singer J. Hip revision arthroplasty in periprosthetic fractures of vancouver
type B2 and B3. J Orthop Trauma 2012;26:206-11.
19. Lindahl H, Malchau H, Odén A, Garellick G. Risk factors for failure after treatment of a
periprosthetic fracture of the femur. J Bone Joint Surg [Br] 2006;88-B:26-30.
20. von Knoch M, Berry DJ, Harmsen WS, Morrey BF. Late dislocation after total hip arthroplasty. J
Bone Joint Surg [Am] 2002;84-A:1949-53.
21. Hamadouche M, Zniber B, Dumaine V, Kerboull M, Courpied JP. Reattachment of the ununited
greater trochanter following total hip arthroplasty: the use of a trochanteric claw plate. J Bone
Joint Surg [Am] 2003;85-A:1330-7.
22. No authors listed. NHS National Institution for Health and Clinical Excellence. Metastatic spinal
cord compression: diagnosis and management of adults at risk of and with metastatic spinal cord
compression, CG75, 2008. http://www.nice.org.uk/CG75 (date last accessed 5 March 2012).
23. Niu J, Zhang Y, Nevitt M, et al. Patella malalignment is associated with prevalent patellofemoral
osteoarthritis: the Beijing Osteoarthritis Study. Arthritis Rheum 2005;52:S456-7.
24. Hunter D, Zhang Y, Niu J, et al. Patella malalignment and its consequences: the Health ABC Study.
Arthritis Rheum 2005;52:S686.
25. Iwano T, Kurosawa H, Tokuyama H, Hoshikawa Y. Roentographic and clinical findings of
patellofemoral arthritis. Clin Orthop Relat Res 1990;252:190-7.
26. Cicuttini FM, Baker JR, Spector TD. The association of obesity with osteoarthritis of the hand and
knee in women: a twin study. J Rheumatol 1996;23:1221-6.
60 Chapter 4  Viva Answers

27. McAlindon TE, Snow S, Cooper C, Dieppe PA. Radiographic patterns of osteoarthritis of
the knee in the community: the importance of the patellofemoral joint. Ann Rheum Dis
1992;51:844-9.
28. Davies AP, Vince AS, Shepstone L, Donell ST, Glasgow MM. The radiological prevalence of
patellofemoral osteoarthritis. Clin Orthop Relat Res 2002;402:206-12.
29. van Jonbergen HP, Poolman RW, van Kampen A. Isolated patellofemoral osteoarthritis. Acta
Orthop 2010;81:199-205.
30. Tarassoli P, Punwar S, Khan W, Johnstone D. Patellofemoral arthroplasty: a systematic review of
the literature. Open Orthop J 2012;6:340-7.
31. No authors listed. National Joint Registry for England and Wales. http://www.njrcentre.org.uk/
(date last accessed 7 March 2013).
Chapter 5
Viva Questions:
Trauma

76. A 70-year-old man fell on his arm while working in his garden and presented to
A&E with the following injury (Figs 5.1a and b).
A. Describe the findings on the radiographs.
B. What else would you like to know about the pre-injury status of the patient?
C. What are the principles of management of such an injury?
D. What are the pitfalls of the proposed modalities of treatment?
E. What are the absolute indications for operative treatment in such an injury?

a b
Figure 5.1a and b
62 Chapter 5  Viva Questions

77. A 74-year-old female living alone in a warden-controlled home, tripped and fell
at home injuring her right leg. She was unable to weight-bear and was brought
into A&E. This is a radiograph of her pelvis (Fig. 5.2).
A. Describe the radiograph.
B. What are the classification systems that are commonly used to describe this
fracture and which system allows for recommendation of treatment based on
the type of fracture?
C. What are the principles of management of such an injury?
D. If you decided to proceed with operative intervention, what kind of implant
would you use and why?
E. What drug has been recently implicated in the treatment of these fractures and
what are the classical radiological features to suggest its usage?

Figure 5.2

78. A 48-year-old right hand dominant civil servant presented to the fracture clinic
following a fall from a horse on his right arm. This is the radiograph obtained in
A&E (Fig. 5.3) .
A. Describe the abnormality on the radiograph.
B. What is the classification system commonly used for this injury and how would
you classify this fracture?
C. What are the ligaments attached to the distal end of clavicle and how do they
influence displacement at the fracture site?
D. Which type of fracture has the highest rate of nonunion?
E. How would you treat this injury?
Trauma 63

Figure 5.3

79. A 25-year-old man sustained this injury after a falling from his motorbike
(Figs 5.4a and b).
A. Describe the abnormalities in the radiographs.
B. How would you initially manage this patient?
C. What classification system would you use to describe this injury? Please explain.
D. How will you manage this injury definitively?
E. What are the Ruedi–Allgower principles of operative fixation for the
management of these fractures?

a
Figure 5.4a and b
64 Chapter 5  Viva Questions

80. An 18-year-old male presents with an acutely swollen knee after an incident
whilst football training during which he was tackled from the side. These are the
radiographs obtained in A&E (Figs 5.5a and b).
A. Describe the radiographs. What is your diagnosis?
B. How would you investigate this patient further?
C. What is the classification of this injury?
D. What would your management be? What are the impediments to successful
closed reduction?
E. If you had to treat this patient operatively, what would be the post-operative
management and what would you specifically warn the patient about prior to
surgery?

a b
Figure 5.5a and b

81. A 46-year-old patient was brought into A&E following a fall from a horse. The
patient is haemodynamically stable and this is the radiograph of the pelvis
(Fig. 5.6).
A. Describe the abnormalities on the radiograph.
B. What is the possible mechanism of this injury and how would you classify
pelvic fractures in general?
C. Where would this injury fit in your classification?
D. What is the risk of urological injury or impotence in patients with such
injuries?
E. What are the radiological signs of rotational and vertical instability in pelvic
fractures?
F. How would you manage this patient?
Trauma 65

Figure 5.6

82. A 26-year-old motorcyclist was involved in a road traffic accident at a speed of


50 mph and sustained this injury (Figs 5.7a and b).
A. How would you classify this injury?
B. How do you classify soft-tissue injuries with closed fractures?
C. How would you manage this injury, assuming that the initial ATLS guidelines
have been followed and the patient is stable?
D. What decision making tools are available for predicting limb salvage after
high-energy trauma to the lower extremity? Describe them in detail.
E. What is the evidence to support the use of these tools?

a b Figure 5.7a and b


66 Chapter 5  Viva Questions

83. A 43-year-old bricklayer presents to A&E with the following injury to his left
shoulder (Fig. 5.8).
A. Describe the radiograph.
B. What are the deforming forces involved with a fracture of this type?
C. How would you define nonunion of the clavicle and what is the rate of
nonunion with diaphyseal fractures of the clavicle?
D. What are the risk factors for nonunion and which is the most predictive?
E. What is the rate of dissatisfaction following these fractures and what are the
possible problems with malunion?

Figure 5.8

84. A 27-year-old man fell from a bike onto his right shoulder and presented
with the following isolated injury to A&E. This is the radiograph obtained on
admission (Fig. 5.9).
A. Describe the radiograph.
B. What other investigation/s would you request?
C. What are the other injuries are associated with this injury?
D. What is the classification system used for describing these fractures?
E. What are the indications for surgery for these fractures and what approach
would you use if you decided to proceed with operative intervention?

Figure 5.9
Trauma 67

85. A six-year-old boy presents after a fall on his right arm with the following
radiographs (Figs 5.10a to c).
A. Describe the radiographs. What are the critical lines used to assess in these
radiographs?
B. What is the commonly used classification of these injuries?
C. What are Monteggia equivalent fractures?
D. What would your management of this injury be and what is the most common
complication?
E. What are the reasons for inability to achieve anatomical reduction?
F. How would you treat a chronic post-traumatic dislocation of the radial head in
a child?

a b

c
Figure 5.10a to c
68 Chapter 5  Viva Questions

86. A 42-year-old woman presented with a fall onto her dominant right hand with a
painful and stiff elbow. These are her radiographs (Figs 5.11a and b).
A. Describe the radiographs.
B. What is the pattern of injury in these fractures?
C. Which gender is this injury most commonly seen in and why?
D. How would you classify this injury?
E. How would you treat it and what approach will you use?
F. What are the complications?

a b

Figure 5.11a and b


Trauma 69

87. A 17-year-old footballer presented with a sudden onset of pain in his right groin
while trying to kick the ball. These are the radiographs obtained in A&E
(Figs 5.12a and b).
A. Describe the radiographs.
B. What is the diagnosis?
C. What is the mechanism of injury and how does it reflect upon the type of fracture?
D. What is the common age group of presentation for these injuries?
E. What is the prognosis and how soon can the patient expect to return to sport?

a b

Figure 5.12a and b


Chapter 6
Viva Answers:
Trauma

76A. Describe the findings on the radiographs.


There is a comminuted fracture of the midshaft of the right humerus. There
is medial translation of the distal fragment and the bone appears osteolytic,
particularly at the fracture site. There are also significant osteoarthritic changes at
the glenohumeral joint.

76B. What else would you like to know about the pre-injury status of
the patient?
I would be concerned about a pathological fracture, possibly due to secondary
metastases from a tumour, and thus would like to know about any constitutional
symptoms (weight loss, fever etc) as well as any specific symptoms relating to any
possible malignancy if such a diagnosis had not been made previously. I would
also like to know about shoulder pain and function prior to the injury in view of
significant glenohumeral OA.

76C. What are the principles of management of such an injury?


Obtaining length, alignment and rotational alignment, and relative stability of the
fracture with concomitant treatment of the shoulder pathology would be the main
principles behind the management of this injury. A long stem hemi-arthroplasty or
a shoulder replacement should be considered.

76D. What are the pitfalls of the proposed modalities of treatment?


Pitfalls of
1. Non-operative treatment – non-union
2. Intramedullary nailing – obtaining access to perform the nailing with high risk
of iatrogenic causing a proximal humeral fracture
3. Plate fixation – shoulder pathology will be left untreated
76E. What are the absolute indications for operative treatment in such
an injury?
Open fracture and vascular injury are absolute indications. Nerve injury may be a
relative indication for surgical management.1
72 Chapter 6  Viva Answers

77A. Describe the radiograph.


There is a 4-part subtrochanteric fracture of the right femur. There is medial
displacement of the shaft and lesser trochanter and the split through the greater
trochanter and involvement of the piriformis fossa extending into the lateral wall
down to the fracture site. This is a Seinsheimer type V fracture.

77B. What are the classification systems that are commonly


used to describe this fracture and which system allows for
recommendation of treatment based on the type of fracture?
Seinsheimer, Fielding’s, AO and Russell-Taylor are classification systems.
Seinsheimer2
1. undisplaced fracture with less than 2 cm displacement
2. 2 part fracture
2a. Transverse fracture
2b. Spiral configuration with the lesser trochanter attached to proximal fragment
2c. Spiral configuration with the lesser trochanter attached to distal fragment
3. 3 part fracture
3a. Three-part spiral configuration with the lesser trochanter a part of the third
fragment
3b. Three-part spiral configuration with the third part a butterfly fragment
4. Comminuted fracture with four or more fragments
5. Subtrochanteric-intertrochanteric configuration
Fielding’s3
1. fracture is at the level of the lesser trochanter
2. fracture is 2.5 to 5 cm below the lesser trochanter
3. fracture is 5 to 7.5 cm below the lesser trochanter
Russell-Taylor4 (based on piriformis fossa involvement)
1. fracture does not extend into piriformis fossa
1a. comminution and fracture lines extend from below lesser trochanter to
femoral isthmus
1b. fracture lines and comminution involve area of lesser trochanter to isthmus
2. fracture extends proximally into greater trochanter and involves piriformis
fossa
2a. no significant comminution or fracture of lesser trochanter
2b. significant comminution of medial femoral cortex and loss of continuity of
lesser trochanter
Of these classifications the Russell-Taylor system is most useful as treatment can
be tailored specifically for different fracture configurations. If the piriformis fossa
is intact then an intramedullary nail can normally be utilised and if the piriformis
fossa is fractured then a fixed angle hip screw device may be preferred.

77C. What are the principles of management of such an injury?


Initial management of such an injury in the emergency department would
include a full ATLS work up including resuscitation and analgesia. The fracture
Trauma 73

is best treated with operative intervention, given anaesthetic suitability, to


ease nursing care, enable a return to mobility and to reduce future medical
complications. Informed consent would then be obtained. Pre-operative
optimisation of the patient status would include orthogeriatric and anaesthetic
input. Regional anaesthesia (spinal) could be used in the event of significant
cardio-respiratory co-morbidity. Post-operative management would centre on
early mobilisation, nutrition, fluid balance and DVT prophlyaxis.
Surgical principles would be to obtain indirect reduction and restoration of
length, alignment and rotation and stabilisation using principles of relative
stability.

77D. If you decided to proceed with operative intervention, what kind


of implant would you use and why?
The Russell-Taylor classification would recommend a fixed angle device such as
a DCS given that the piriformis fossa is involved and the entry point for a nail is
comminuted. The key issue for this fracture is to attempt to restore the anatomy
prior to fixation and therefore one should have a low threshold to perform an
open reduction with cabling/wiring of the subtrochanteric element. Modern
cepahlomedullary long intramedullary nails with distal locking can also be used
and would be my implant of choice.

77E. What drug has been recently implicated in the treatment of


these fractures and what are the classical radiological features to
suggest its usage?
Bisphosphonates have been recently associated with subtrochanteric fractures.
This is typically a transverse fracture in an area of thickened cortices, following
low energy trauma. Before the fracture a conical ridge at the lateral aspect of the
femur in the subtrochanteric area can be seen.5

78A. Describe the abnormality on the radiograph.


The radiograph shows a vertically displaced, comminuted fracture of the lateral
third of the clavicle.

78B. What is the classification system commonly used for this injury
and how would you classify this fracture?
The Rockwood and Green classification6 of lateral clavicle fractures:
• Type I – fracture lateral to the coracoclavicluar ligaments
• Type II – fracture medial to the coracoclavicluar ligaments
• Type III – injury extends into the ACJ
• Type IV – Paediatric periosteal sleeve injury
• Type V – comminuted fracture
The radiograph shows a Type V fracture.
74 Chapter 6  Viva Answers

78C. What are the ligaments attached to the distal end of clavicle and
how do they influence displacement at the fracture site?
The ligaments attached to the distal end of the clavicle are the acromioclavicular
and coracoclavicular ligaments. The acromioclavicular ligament provides
anterior/posterior stability. It has superior, inferior, anterior, and posterior
components, with the superior being the strongest. This is why the distal
fragment in a lateral clavicle fracture is held in place adjacent to the acromion.
The coracoclavicular ligaments (trapezoid and conoid) provide vertical stability.
The trapezoid ligament inserts 3 cm from end of clavicle and the conoid ligament
inserts 4.5 cm from end of clavicle in the posterior border. The conoid ligament is
strongest. Rupture of these ligaments leads to significant vertical displacement of
the clavicle fracture as seen in the radiograph.

78D. Which type of fracture has the highest rate of nonunion?


Robinson et al7 found that displacement of the fracture and increased age
of patient were associated with a higher risk of non-union in lateral clavicle
fractures. These fractures in themselves have an increased rate of non-union,
when compared to middle or medial third fractures.

78E. How would you treat this injury?


A fall from a horse should be considered a high-energy injury and I would
therefore manage the patient according to ATLS protocol. Assuming this is an
isolated injury, I would perform a history and a thorough examination with
special reference to neurovascular status also ensuring that this was a closed
injury. Using Robinson’s prognostic index I would explain to the patient that
there was a 30% chance of nonunion at six months and that treatment could be
operative or non-operative. Non-operative treatment would eliminate surgical
risk. Indeed Robinson8 advocated non-operative management in middle-aged
individuals with displaced lateral clavicle fractures:
Nonoperative treatment of most displaced lateral fractures of the
clavicle in middle-aged and elderly patients achieved a good medium-
term functional result. Symptoms that were severe enough to warrant
a delayed reconstructive procedure developed in only 14% of the
patients. Asymptomatic nonunion does not appear to adversely affect the
functional outcome in the medium term.8
I would therefore advocate non-operative management. However, operative
intervention may lead to a quicker improvement in function and I would
discuss this with the patient. Surgical options could include a hook plate or
reconstruction of the coracoclavicular ligaments.

79A. Describe the abnormalities in the radiographs.


There is a comminuted distal tibial fracture with intra-articular extension (pilon
fracture). Moderately displaced transverse distal diaphyseal fibular fracture. Both
fractures show varus angulation.
Trauma 75

79B. How would you initially manage this patient?


Initial management would involve an advanced trauma life support (ATLS)
approach to rule out any life threatening injuries. Following this, the limb would
be assessed and neurovascular status documented.
If this was an open injury, intravenous antibiotics would be administered, tetanus
prophylaxis given, a photograph would be taken and a preliminary lavage and
betadine dressing would be applied.
This injury represents a significant soft-tissue insult and this must be considered
prior to any definitive fixation. The injury would be splinted temporarily in
and above knee back slab and temporary external fixation (+/- debridement
+/- plastic surgical consultation) would then be undertaken in the operating
theatre.
A CT scan would be ideal pre-operatively to plan definitive fixation.

79C. What classification system would you use to describe this injury?
Please explain.
Ruedi–Allgower classification of pilon fractures:
• Type 1 - Mild to moderate displacement and no comminution without major
disruption of ankle joint
• Type 2 - Moderate displacement and no comminution without significant
dislocation of ankle joint
• Type 3 - Explosion fracture with severe comminution and displacement
79D. How will you manage this injury definitively?
Definitive management would involve a CT scan to accurately assess the degree
of intra-articular involvement and plan the surgical approach to the distal tibia.
The preferred option to restore articular congruity would be open reduction
and internal fixation of the distal tibia once the soft tissue insult has been
minimised.
An anterolateral or anteromedial approach could be utilised depending on exact
fracture pattern on the CT scan.
Options for fixation would be ORIF, External fixation (bridging or non-bridging),
and combination of internal and external fixation or percutaneous plating.

79E. What are the Ruedi–Allgower principles of operative fixation for


the management of these fractures?
The best functional results in the past series were observed in patients treated
according to the following four sequential principles:9
1. Reconstruction of the correct length of the fibula
2. Anatomical reconstruction of the articular surface of the tibia
3. Insertion of a cancellous autograft to fill gaps left by impaction and
comminution
76 Chapter 6  Viva Answers

4. Stable internal fixation of the fragments by a plate placed on the medial


aspect of the tibia.
80A. Describe the radiographs. What is your diagnosis?
The radiographs demonstrate a lipohaemarthrosis, a Segond fracture and an
avulsion fracture of the tibial spine. Originally described by the French surgeon
Paul Segond in 1879 after a series of cadaveric experiments, the Segond fracture
describes a cortical avulsion of the tibial insertion of the middle third of the
lateral capsular ligament. It is commonly associated with injury to the anterior
cruciate ligament.

80B. How would you investigate this patient further?


I would perform a full history and examination and request an MRI scan of the
knee, to confirm an ACL injury and to identify any other injuries within the knee
to the menisci, collateral ligaments and to the PCL.

80C. What is the classification of this injury?


The Myers and McKeever10 classification for fractures of the intercondylar
eminence is as follows:
• Type I: Undisplaced fracture
• Type II: Partially displaced fracture with anterior elevation of the eminence;
• Type III A: Completely displaced fracture with no contact between the
eminence and proximal end of tibia
• Type III B: The eminence is rotated as well as displaced
80D. What would your management be? What are the impediments to
a successful closed reduction?
The radiographs show a Type II injury. This could be managed with a trial of
closed reduction under anaesthetic.
A block to full extension may be caused by interposition of the anterior horns
of either the medial or the lateral meniscus. If full extension is not possible,
reduction is not complete. This would necessitate an arthroscopic assisted/open
reduction and internal fixation of the tibial spine.

80E. If you had to treat this patient operatively, what would be the
post-operative management and what would you specifically
warn the patient about prior to surgery?
Post-operatively, I would immobilise the knee in extension for six to eight weeks.
At that stage, a knee brace would be employed, gradually increasing flexion
by 30° every two weeks. Closed chain exercises would be advocated as per a
standard ACL rehabilitation protocol.
I would warn the patient about the usual risks of surgery, but more specifically,
the risk of ACL laxity upon fracture healing and also arthofibrosis with resultant
stiffness of the knee.
Trauma 77

81A. Describe the abnormalities on the radiograph.


There is a significant diastasis of the symphysis pubis and significant widening of
the right sacro-iliac (SI) joint.

81B. What is the possible mechanism of this injury and how would you
classify pelvic fractures in general?
This is likely to be an antero-posterior compression injury. This injury can be
classified by the Tile11 or Young and Burgess12 classification systems.
The Tile classification includes Type A (stable), Type B (rotationally unstable but
vertically stable) and Type C (rotationally and vertically unstable).
The Young and Burgess system is based on mechanism of injury, Type A (lateral
compression), Type B (AP compression) and Type C (vertical shear).
Within the AP compression (APC) type there are three further subdivisions:
APC-I
• Slight widening of pubic symphysis/anterior SI joint
• Intact anterior SI, sacrotuberous and sacrospinous ligaments
• Intact posterior SI ligaments

APC II
• Widened anterior SI joint; disrupted anterior SI, sacrotuberous and
sacrospinous ligaments
• Intact posterior SI ligaments

APC III
• Complete SI joint disruption with lateral displacement
• Disrupted anterior SI, sacrotuberous and sacrospinous ligament
• Disrupted posterior SI ligaments
81C. Where would this injury fit in your classification?
This is therefore a Type B, APC II injury.
More specifically this is an open book pelvis injury. As the posterior SI joint is
intact it acts as a hinge from which the displaced anterior SI joint and the right
hemipelvis can externally rotate.

81D. What is the risk of urological injury or impotence in patients with


such injuries?
A recent study (Pavelka et al) showed that primary injury to the urological tract
was recorded in 13.5% of patients with pelvic fractures. Injury to the urethra was
found in 7.5% and urinary bladder trauma in 6%.
Injury to the urological tract was associated with a Type A pelvic ring fracture in
5%, type B in 34% and type C in 61% of the patients.
Out of the patients with urethral trauma, 35 % receive therapy for urethral stenosis
and 30% reported urinary incontinence. 30% have problems related to impotence.13
78 Chapter 6  Viva Answers

81E. What are the radiological signs of rotational and vertical


instability in pelvic fractures?
Radiological signs of rotational instability include pubic symphyseal diastasis of
more than 2.5 cm; fracture of the ischial spine and avulsion fracture of lateral sacrum.
Radiological signs of vertical instability include a sacral fracture with a gap;
avulsion fracture of the tip of the L5 transverse process; vertical displacement of
the sacroiliac joint of more than one cm.

81F. How would you manage this patient?


I would adopt the ATLS approach, initially assessing airway with cervical spine
control, breathing and then circulation. There is likely to be massive blood loss
from this injury so rapid fluid replacement including type O negative blood will
be key. A full primary survey would be undertaken with a chest radiograph,
lateral cervical spine and AP radiograph of the pelvis. A pelvic binder would be
applied on suspicion of a pelvic injury. A detailed trauma CT would ideally be
obtained, which may obviate the need for a chest and lateral C-spine radiograph.
The genitourinary examination would be undertaken and retrograde
gastrograffin cystogram would be undertaken to assess urethral integrity prior to
catheter placement.
A pelvic external fixator could be considered in this case if the appropriate
expertise was available. Also, transfer to a Level 1 trauma centre should be
considered for angiographic embolisation if necessary.
After initial resuscitation and temporary stabilisation (either pelvic binder or
external-fixator) definitive management would involve stabilisation of anterior
symphyseal (open reduction and internal fixation with a symphyseal plate) and
posterior sacroiliac diastases (sacroiliac screws).

82A. How would you classify this injury?


The radiographs shows a displaced, comminuted, segmental and intra-articular
fracture of the tibia with an associated fibula fracture. Using the Schatzker
classification this is a type VI injury and according to AO this would be a 41-C3
multifragmentary complete articular fracture.
Trauma 79

82B. How do you classify soft-tissue injuries with closed fractures?

  Table 6.1

Grade 0 Minimal soft-tissue damage


Indirect injury to limb (torsion)
Simple fracture pattern
Grade 1 Superficial abrasion or contusion
Mild fracture pattern

Grade 2 Deep abrasion


Skin or muscle contusion
Severe fracture pattern
Direct trauma to limb
Grade 3 Extensive skin contusion or crush injury
Severe damage to underlying muscle
Compartment syndrome
Subcutaneous avulsion

82C. How would you manage this injury, assuming that the initial ATLS
guidelines have been followed and the patient is stable?
Assuming that this is an isolated injury in a haemodynamically stable patient and
is closed, initial management would also involve assessment of compartment
syndrome with compartment pressure monitoring. I would inform the patient
that this is a limb-threatening injury and limb salvage may not eventually be
possible. Consultation with consultant vascular/plastic surgeons would aid in this
decision making process. However, assuming that the injury was closed, isolated
and vascularity was intact to a sensate foot in a physiologically stable patient, I
would discuss the options with the patient. One option is to immediately have
an above knee amputation, which would lead to a quicker return to function
but with the inherent problems of an above- knee amputation. The other
option would involve temporary spanning external fixator +/- fasciotomies and
then definitive treatment with a fine wire external fixator aiming to achieve a
mechanically aligned, united and non-infected tibia. This would be a long process
potentially taking months/years of surgical intervention and compliance by the
patient.

82D. What decision making tools are available for predicting limb
salvage after high-energy trauma to the lower extremity?
Describe them in detail.
The mangled extremity severity score (Johansen et al14)
The mangled extremity severity score is probably the most well known limb
salvage scoring system and is as follows:
80 Chapter 6  Viva Answers

Skeletal/soft-tissue injury
Low energy (stab; simple fracture; pistol gunshot wound): 1
Medium energy (open or multiple fractures, dislocation): 2
High energy (high speed MVA or rifle GSW): 3
Very high energy (high speed trauma + gross contamination): 4
Limb ischaemia
Pulse reduced or absent but perfusion normal: 1*
Pulseless; paresthesias, diminished capillary refill: 2
Cool, paralysed, insensate, numb: 3*
Shock
Systolic BP always > 90 mm Hg: 0
Hypotensive transiently: 1
Persistent hypotension: 2
Age (years)
< 30: 0
30 to 50: 1
> 50: 2
It has been stated that a MESS score of greater than 7 equates to a poor limb
viability prognosis.
Limb Salvage Index (LSI) (Russell et al15)
Limb trauma associated with vascular injury. Absolute indications for amputation
included a score of 6 or more. Retrospective analysis of 70 limbs. 26 had pulse
deficits requiring revascularisation.
Seven components related to injury:
• Arterial
• Nerve
• Bone
• Skin
• Muscle
• Deep venous injury
• Warm ischaemia time
Predictive Salvage Index (PSI) (Howe et al16)
Combined orthopaedic and vascular injuries. Intent to help prevent the
attempted salvage of a doomed or useless limb. Retrospective, 21 limbs, studied
to determine which variables influenced salvage or loss.
A limb-salvage score was developed that weighted:
• Level of the vascular injury
• Degree of osseous injury
• Degree of muscle injury
• Warm ischemia time
Trauma 81

82E. What is the evidence to support the use of these tools?

Table 6.2

MESS LSI PSI


Bosse (n = 556) 0.46 (sensitivity) 0.46 (sensitivity) 0.46 (sensitivity)
(all limbs, subgroups
analysed in original 0.91 (specificity) 0.97 (specificity) 0.87 (specificity)
paper)

Bosse et al17 in a prospective study of 556 limbs found that all the above lower-
extremity injury-severity scoring systems have limited usefulness and cannot
be used as the sole criterion by which amputation decisions are made. Overall,
the lower-extremity injury-severity scores lack sensitivity, although in some
cases they were very specific. The high specificity of the scores confirms that low
scores could be used to predict limb-salvage potential. The converse, however,
was not true. The low sensitivity of the indices failed to support the validity of
the scores as predictors of amputation. The LSI perormed better than the MESS
or PSI scores overall, especially when only Grade IIIb open tibial fractures were
considered.

83A. Describe the radiograph.


The radiograph shows a comminuted fracture of the mid-shaft of the clavicle with
complete displacement and shortening.

83B. What are the deforming forces involved with a fracture of


this type?
For a midshaft fracture:
The medial segment is stabilised by the sternoclavicular ligaments. Sternohyoid
and subclavius muscles also insert medially.
The clavicular head of the sternocleidomastoid muscle pulls the medial segment
superiorly.
The lateral segment is pulled inferiorly and medially by the clavicular head of
pectoralis major.
The shoulder girdle and the weight of the arm exert an inferiorly deforming
force on the lateral segment through the acromioclavicular joint and the
coracoclavicular ligaments.
Fibres of the trapezius also insert into the posterior border of the lateral third of
the clavicle, providing a stabilising force against inferior displacement.
82 Chapter 6  Viva Answers

83C. How would you define nonunion of the clavicle and what is the
rate of nonunion with diaphyseal fractures of the clavicle?
Non-union of the clavicle is defined as a lack of radiological union by four
months. Clinically, this can be assessed by the presence of pain and movement
at the fracture site upon stressing. Radiologically, it can be assessed on either
radiographs or a CT scan. Radiographs will show a lack of bridging callus, but
these may be difficult to interpret due to the difficulty in obtaining orthogonal
views. Usually the clavicle is assessed using an AP and 15° to 20° elevated AP
view. An apical oblique view may also be useful. Clavicle fractures can also be
assessed using an axillary view for the more lateral fractures. A CT scan will give a
more reliable diagnosis of non-union.
The incidence has been reported to be between 0.13% and 15% for midshaft
fractures.
In the paper by Robinson et al18 studying 868 clavicle fractures, the reported
incidence of non-union in diaphyseal clavicle fractures was 4.5% at 24 weeks
post-injury.

83D. What are the risk factors for nonunion and which is the most
predictive?
Robinson et al18 found that following a diaphyseal clavicle fracture, advancing age,
female gender, displacement of the fracture, and the presence of comminution
significantly increased the risk of non-union. On multivariate analysis, all of these
factors remained independently predictive of non-union, and, in their final model,
the risk of non-union was increased by lack of cortical apposition.
In Robinson’s study a complete fracture displacement was the most predictive of
non-union.
Other factors that have been found to relate to non-union include shortening
> 20 mm, displacement > 20 mm, increasing severity of trauma and re-fracture.
Other general risk factors for non-union such as co-morbidities, smoking and
medications should also be taken into account.19

83E. What is the rate of dissatisfaction following these fractures and


what are the possible problems with malunion?
Hill et al20 found that malunion with shortening of 2 cm or more in a middle
third fracture led to a poor symptomatic and functional result. 31% reported
unsatisfactory results, including pain and cosmesis.
There was also a recognised rate of thoracic outlet syndrome. In a RCT by the
Canadian Orthopaedic Trauma Society there was a significantly higher rate of
dissatisfaction with the appearance of the shoulder in the non-operative group
(47%). This included a “droopy” shoulder, a bump, pain and asymmetry.21
A recent meta-analysis reported symptomatic malunions in 17 of 200 (8.5%) non-
operatively treated patients after midshaft fractures of the clavicle.22
Trauma 83

In this patient, who works as a bricklayer, one must consider whether he carries
heavy loads on the injured shoulder, as a bump from a malunited clavicle fracture
may be problematic.

84A. Describe the radiograph.


There is a displaced right scapular neck fracture, the articular surface of the
glenoid does not appear to be involved in the fracture.

84B. What other investigation/s would you request?


I would request true anteroposterior radiograph of the shoulder and
axillary views. Additionally a CT scan to further evaluate the degree of
displacement, rotation and intra-articular extension (glenoid fracture) should
be undertaken.

84C. What are the other injuries are associated with this injury?
• Rib fractures
• Haemo-/pneumothorax
• Flail chest/tension pneumothorax
• Brachial plexus injuries
• Vascular injuries
84D. What is the classification system used for describing these
fractures?
The classification system used here is as follows – Ideberg:23
• Type I – non-angulated, non-displaced
• Type IIa – shortened/displaced > 1 cm
• Type IIb – Angulated > 40°
84E. What are the indications for surgery for these fractures and what
approach would you use if you decided to proceed with operative
intervention?
Indications for surgery include more than 1 cm displacement or 40° angulation
(Type IIa and IIb fractures). I would use a posterior (Judet) approach. A curved
skin incision starts at the lateral prominence of acromion, courses medially along
scapular spine and caudally to inferior angle of the scapula. Deltoid is detached
from the scapular spine and retracted laterally. The approach is deepened
through the interval between infraspinatus and teres minor.

85A. Describe the radiographs. What are the critical lines that are used
to assess these radiographs?
There is a lateral dislocation of the radial head and a fracture of the proximal
ulna. The radiocapitellar line is used to assess radial head dislocation. When a
line is drawn through the proximal radial shaft and neck, and extended through
the joint, it should pass through to the articulating capitellum. An abnormal
84 Chapter 6  Viva Answers

radiocapitellar line will demonstrate dislocation of the radial head. Additionally


the anterior humeral line should also be assessed. This is a line drawn on a lateral
view along the anterior surface of the humerus and it should pass through the
middle third of the capitellum. This is altered in displaced supracondylar humeral
fractures.

85B. What is the commonly used classification of these injuries?


Monteggia fracture classification:
• Bado I – anterior dislocation of radial head with proximal ulna fracture
• Bado II – posterior dislocation of radial head with proximal ulna fracture
• Bado III – lateral dislocation of radial head with proximal ulna fracture
• Bado IV – anterior radial head dislocation and radial neck and proximal ulna
fracture
85C. What are Monteggia equivalent fractures?
1. Isolated radial head dislocation
2. Fracture of proximal ulna and radial neck
3. Both bones proximal third fractures with the radial fracture more proximal
than the ulnar fracture
85D. What would your management of this injury be and what is the
most common complication?
Initial management would involve accurate history taking and assessment of
neurovascular status. This injury would have an initial closed reduction attempted
but may require open reduction. Internal fixation of the proximal ulna would be
carried out in an older child.

85E. What are the reasons for inability to achieve anatomical


reduction?
Interposition of the annular ligament, inadequate flexion of the elbow
and buttonholing of the radial head through the capsule. If there is plastic
deformation of the ulna (without fracture) this may also impede reduction.

85F. How would you treat a chronic post-traumatic dislocation of the


radial head in a child?
If the dislocation is symptomatic the radial head can be resected at skeletal
maturity, prior to this may lead to angular deformity at the elbow and wrist.
Within six months an open reduction and reconstruction of the annular ligament
can sometimes be undertaken and also a transcapitellar pin insertion for
unstable dislocations. For reconstruction many procedures have been described.
The Bell-Tawse procedure uses a central slip of triceps fascia whilst the Lloyd-
Roberts procedure uses the lateral slip of triceps fascia attached distally.24
Trauma 85

86A. Describe the radiographs.


Anteroposterior and lateral radiographs of the elbow showing a fracture of the
capitellum.

86B. What is the pattern of injury in these fractures?


This is normally a coronal shear injury that occurs following a fall onto
outstretched hand or elbow.

86C. Which gender is this injury most commonly seen in and why?
This injury is more common in females due to the greater carrying angle of the
elbow, and also the ability to hyperextend the elbow allowing the radial head to
forcefully shear off the capitellum.

86D. How would you classify this injury?


Type I
• Hahn-Steinthal fracture
• Fracture of capitellum in coronal plane
• Large osseous fragment
The other main type is Kocher-Lorenz (Type II) which is a sleeve fracture of the
articular surface with little osseous bone.
Broberg and Morrey (Type III) – comminuted fracture
McKee modification also mentions Type IV – coronal shear including capitellum
and trochlea.

86E. How would you treat it and what approach will you use?
This is a displaced fracture and would cause a block to flexion, I would treat this
injury operatively. I would employ a Kocher’s lateral approach to the elbow,
starting over the lateral supracondylar ridge, 5 cm proximal to the elbow joint
and then continuing distally to the lateral surface of the proximal forearm
posterior to the radial head. I would keep the forearm in pronation to avoid injury
to the posterior interosseous nerve during the approach. The interval between
ECU and anconeus is dissected and a capsulotomy is made. After reduction of the
fracture I would use headless screws to fix the fracture and prevent impingement
at the radiocapitellar joint.

86F. What are the complications?


Nonunion – 1% to 11% with open reduction and internal fixation, heterotrophic
ossification (4%) and avascular necrosis.
86 Chapter 6  Viva Answers

87A. Describe the radiographs.


The radiographs are an AP and frog leg lateral view of the right hip in a skeletally
immature patient. They show an avulsion of the anterior inferior iliac spine
apophysis.

87B. What is the diagnosis?


Traction injury resulting in the straight head of the rectus femoris muscle avulsing
the anterior inferior iliac spine apophysis.

87C. What is the mechanism of injury and how does it reflect upon the
type of fracture?
Forceful concentric or eccentric contraction of the straight head of the rectus
femoris muscle avulses the anterior inferior iliac spine apophysis. This can typically
occur when kicking whilst playing football. A sudden, large force may be more
likely to produce a complete avulsion, whereas chronic repetitive traction forces
may produce an incomplete avulsion fracture or apophysitis without a history of
acute trauma. A hyperflexion injury may lead to an avulsion of the reflected head of
rectus femoris and a hyperextension injury leads to this type of a fracture.

87D. What is the common age group of presentation for these injuries?
These injuries occur in children aged 11 to 17 years. They occur through secondary
centres of ossification, before the centre fuses with the pelvis. The secondary
ossification centre at the anterior inferior iliac spine may be present at approximately
14 years of age, fusing around 16 years of age.

87E. What is the prognosis and how soon can the patient expect to
return to sport?
Further fracture displacement may be prevented because the rectus femoris
reflected head will still be attached in its normal location.
Most paediatric pelvic avulsion fractures will do well with non-operative
treatment. Two case series reported that all patients returned to pre-injury
activity levels.25,26 However, some authors recommend open reduction and
internal fixation in acute fractures displaced > 1 cm to 2 cm.27
I would advise this patient to rest and partially weight bear for at least two weeks,
avoiding stretching of the rectus femoris muscle (avoid hip extension postures).
I would advise that they should return to football in around three months,
however, this would ultimately be determined with serial clinical and radiological
follow-up. I would advise them to miss the rest of the football season.
References 87

References
1. Bishop J, Ring D. Management of radial nerve palsy associated with humeral shaft fracture: a
decision analysis model. J Hand Surg Am 2009;34:991-6.
2. Seinsheimer F. Subtrochanteric fractures of the femur. J Bone Joint Surg [Am] 1978;60-A:300-6.
3. Fielding JW, Magliato HJ. Subtrochanteric fractures. Surg Gynecol Obstet 1966;122:555-69.
4. Nork S, Reilly M, Russell TA. Subtrochanteric fractures. In: Browner BD, Levine AM, Jupiter JB,
Trafton PG, eds. Skeletal Trauma. 1998;2:1891-7.
5. Lenart BA, Neviaser AS, Lyman S, et al. Association of low-energy femoral fractures with
prolonged bisphosphonate use: a case control study. Osteoporos Int 2009;20:1353-62.
6. Craig EV. Fractures of the clavicle. In: Rockwood CA Jr, Green DP, Bucholz RW, eds. Fractures in
adults. Third ed. Philadelphia, etc: JB Lippincott Company, 1991:928-90.
7. Robinson CM, Court-Brown CM, McQueen MM, Wakefield AE. Estimating the risk of nonunion
following nonoperative treatment of a clavicular fracture. J Bone Joint Surg [Am] 2004;86-A:
1359-65.
8. Robinson CM, Cairns DA. Primary nonoperative treatment of displaced lateral fractures of the
clavicle. J Bone Joint Surg [Am] 2004;86-A:778-82.
9. Rüedi TP, Allgöwer M. The operative treatment of intra-articular fractures of the lower end of the
tibia. Clin Orthop Relat Res 1979;138:105-10.
10. Meyers MH, McKeever FM. Fracture of the Intercondylar Eminence of the Tibia. J Bone Joint Surg
[Am] 1970;52-A:1677-84.
11. Tile M. Pelvic ring fractures: should they be fixed? J Bone Joint Surg [Br] 1988;70-B:1-12.
12. Burgess AR, Eastridge BJ, Young JW, et al. Pelvic ring disruptions: effective classification system
and treatment protocols. J Trauma 1990;30:848-56.
13. Pavelka T, Houcek P, Hora M, Hlavácová J, Linhart M. Urogenital trauma associated with pelvic
ring fractures. Acta Chir Orthop Traumatol Cech 2010;77:18-23 (in Czech).
14. Johansen K, Daines M, Howey T, Helfet D, Hansen ST Jr. Objective criteria accurately predict
amputation following lower extremity trauma. J Trauma 1990;30:568-73.
15. Russell WL, Sailors DM, Whittle TB, Fisher DF Jr, Burns RP. Limb salvage versus traumatic
amputation: a decision based on a seven-part predictive index. Ann Surg 1991;213:473-81.
16. Howe HR Jr, Poole GV, Hansen KJ, et al. Salvage of lower extremities following combined
orthopedic and vascular trauma: a predictive salvage index. Am Surg 1987;53:205-8.
17. Bosse MJ, MacKenzie EJ, Kellam JF, et al. A prospective evaluation of the clinical utility of lower
extremity injury severity scores. J Bone Joint Surg [Am] 2001;83-A:3-14.
18. Robinson CM, Court-Brown CM, McQueen MM, Wakefield AE. Estimating the risk of nonunion
following nonoperative treatment of a clavicular fracture. J Bone Joint Surg [Am] 2004;86-
A:1359-65.
19. Gaston MS, Simpson AH. Inhibition of fracture healing. J Bone Joint Surg [Br] 2007;89-B:1553-60.
20. Hill JM, McGuire MH, Crosby LA. Closed treatment of displaced middle-third fractures of the
clavicle gives poor results. J Bone Joint Surg [Br] 1997;79-B:537-9.
21. No authors listed. Nonoperative treatment compared with plate fixation of displaced midshaft
clavicular fractures: a multicenter, randomized clinical trial. J Bone Joint Surg [Am] 2007;89-A:1-10.
22. McKee RC, Whelan DB, Schemitsch EH, McKee MD. Operative versus nonoperative care of
displaced midshaft clavicular fractures: a meta-analysis of randomized clinical trials. J Bone Joint
Surg [Am] 2012;94-A:675-84.
23. Ideberg R. Fractures of the scapula involving the glenoid fossa. In: Bateman JE, Welsh RP, eds.
Surgery of the shoulder. Philadelphia: Decker, 1984:63-6.
24. Canale ST, Beaty JH. Campbell’s operative orthopaedics. 11th ed. St. Louis: Mosby, 2008.
25. Fernbach SK, Wilkinson RH. Avulsion injuries of the pelvis and proximal femur. AJR Am J
Roentgenol 1981;137:581-4.
26. Metzmaker JN, Pappas AM. Avulsion fractures of the pelvis. Am J Sports Med 1985;13:349-58.
27. Lynch SA, Renström PA. Groin injuries in sport: treatment strategies. Sports Med 1999;28:137-44.
Chapter 7
Viva Questions:
Hands
88. A 32-year-old electrician presents to the clinic with a history of pain in his
elbow, which is radiating down his forearm. He does not give any history of
trauma but complains of occasional numbness and tingling in his forearm.
A. What is your differential diagnosis?
B. How will you confirm your diagnosis clinically?
C. What are the common sites of entrapment of the ulnar nerve along its course
in the upper arm and forearm?
D. What investigation would you request if any and why?
E. How would you treat him at this stage?
F. If conservative management fails, what kind of surgical intervention would you
perform for this condition?
G. What is the expected prognosis following surgical intervention?

89. A 34-year-old man sustained a hyperextension injury to his little finger leading
to this injury (Fig. 7.1).
A. What is your differential diagnosis?
B. How would you confirm your diagnosis?

Figure 7.1
90 Chapter 7  Viva Questions

C. Which finger is commonly involved?


D. What is the classification system associated with this problem?
E. What are the treatment options?
F. How would you like to treat this patient?

90. A 40-year-old scaffolder presents with persistent pain in his wrist following a
fall on his outstretched hand about six months ago. These are the radiographs
(Figs 7.2a and b).
A. What is your diagnosis?
B. What is the classification system associated with this problem?
C. What stage/grade would you assign to this radiograph?
D. What is the natural history of this condition?
E. What are the options for treatment?
F. How would you treat this patient?

a b
Figure 7.2a and b
Hands 91

91. A 42-year-old woman presents to you with a pain and swelling in the small joints
of her hand. These are her radiographs (Figs 7.3a and b).
A. What is the diagnosis?
B. Which skin condition is associated with this problem?
C. What are the characteristic radiological features?
D. What is the natural history of this condition?

a b
Figure 7.3a and b

92. A 14-year-old left-hand-dominant boy presents with a painless deformity of his


hand. These are the clinical photograph and radiograph (Figs 7.4a and b).
A. Describe the abnormalities seen in the photograph and the radiograph.
B. Classify this abnormality.
C. Discuss common inheritance patterns of this condition.
D. What are the main indications for surgical treatment in this condition?
E. What are the principles when considering surgical treatment of this particular
case?
92 Chapter 7  Viva Questions

a b

Figure 7.4a and b

93. A 50-year-old female presents with a history of a painful swelling in her finger.
It appears to burst on occasion. This is the clinical photograph and radiograph
(Figs 7.5a and b).
A. What are the differential diagnoses?
B. What is the diagnosis?
C. What are the causes of this condition?
D. How would you treat this patient?
E. What are the possible complications?

a b

Figure 7.5a and b


Hands 93

94. A 32-year-old weightlifter presents with this injury (Figs 7.6a and b).
A. What is your diagnosis?
B. How does this injury usually occur?
C. What are the treatment options?
D. What is the prognosis if the patient opted for surgical intervention?

b
Figure 7.6a and b

95. A 32-year-old pianist who is left-hand dominant presents after a fall from stairs
and injuring his left hand. This is his radiograph (Fig 7.7).
A. Describe the radiograph.
B. What is your diagnosis?
C. How is this injury assessed?
D. What are the options for treatment?
E. How will you manage this patient?
94 Chapter 7  Viva Questions

Figure 7.7

96. A 55-year-old IT professional presents with painful superficial nodules over his
palm over a period of a year (Fig. 7.8).
A. What is your diagnosis?
B. What is the cause of pain in this condition?
C. How would you stage this condition?
D. What is the treatment for this condition?
E. What are the possible complications of surgical intervention?

Figure 7.8
Hands 95

97. This is the clinical photograph (Fig. 7.9) of a man who has had a previous
procedure for an injury to one of the tendons in the hand.
A. What procedure has this patient had?
B. What deficit would the patient have prior to surgery?
C. What are the causes of this pathology?
D. Which other tendon transfers have been described?
E. What are the principles of a tendon transfer?

Figure 7.9
96 Chapter 7  Viva Questions

98. This is the clinical photograph (Fig. 7.10) of a 28-year-old who presented to the
hand clinic with a long-standing history of a problem with his thumb.
A. Describe the clinical photograph.
B. What is the differential diagnosis?
C. What is the most likely diagnosis?
D. What is the pathophysiology of this condition?
E. What is the treatment of this condition?

Figure 7.10
Hands 97

99. A 14-year-old boy presents with a hyperextension injury to his index finger
while playing football. Clinically the finger appears slightly swollen and
tender with severely reduced range of movements particularly flexion. Several
attempts at reduction under a ring block in A&E were unsuccessful. Here are the
radiographs (Figs 7.11a to c).
A. Describe the radiographs.
B. What is the diagnosis?
C. What are the causes of irreducibility?
D. What is the treatment?
E. What is the prognosis?

a b c

Figure 7.11a to c
Chapter 8
Viva Answers:
Hands
88A. What is your differential diagnosis?
• Local
–– Ulnar nerve entrapment in the cubital tunnel
–– Lateral/medial epicondylitis
–– Radial/median nerve entrapment at the elbow
–– Elbow arthropathy
–– Brachial plexus lesion (Pancoast lung tumour)
–– Cervical radiculopathy
• Systemic
–– Peripheral neuropathy
–– Mononeuritis multiplex
–– Multiple sclerosis
–– Leprosy
–– Lyme disease
88B. How will you confirm your diagnosis clinically?
History – do symptoms come on at night, does patient sleep with elbows bent
(ulnar nerve compression). Is the pain affected by movement?
Examination – evidence of claw hand, looking for evidence of ulnar nerve
dysfunction (weakness of small muscles of hand, Froment’s sign). Sensory deficit
in ulnar 1.5 fingers. Tapping over nerve in cubital tunnel may re-create symptoms.
Also assess elbow range of movement and look for tenderness over epicondyles
to rule out other pathology.

88C. What are the common sites of entrapment of the ulnar nerve
along its course in the upper arm and forearm?
Cubital tunnel – the ulnar nerve becomes commonly compressed in the cubital
tunnel which has the aponeurotic attachment of the two heads of flexor carpi
ulnaris (FCU) as a roof with the arcuate (Osborne’s) ligament and the floor of the
tunnel is formed by the medial collateral ligament.
Guyon’s canal – pisiform and hamate and roof is the volar carpal and pisihamate
ligament - both ulnar nerve and artery travel through here.
100 Chapter 8  Viva Answers

88D. What investigation would you request if any and why?


Investigation would depend on clinical history and examination. If the patient
was experiencing mild discomfort and there were few clinical signs, simple
measures such as avoiding aggravating positions of the elbow may help.
Nonetheless it would be useful to establish a clear diagnosis and therefore I
would obtain nerve conduction studies.

88E. How would you treat him at this stage?


Avoid excessive elbow flexion and any aggravating manoeuvres. Consider
elbow splintage, particularly at night. Non-steroidal anti-inflammatories unless
otherwise contra-indicated. Further follow up in 3 months.

88F. If conservative management fails, what kind of surgical


intervention would you perform for this condition?
Simple decompression of the ulnar nerve (cubital tunnel decompression):
1. The incision is 8 cm in length and is made over the medial aspect of the
elbow. An attempt to identify the medial antebrachial cutaneous nerve is
made (often crossing FCU).
2. The ulnar nerve is identified and tagged proximally.
3. The nerve is followed upward and released at the intermuscular septum
4. The aponeurosis of the cubital tunnel and FCU are then incised.
5. Submuscular or subcutaneous transposition of the ulnar nerve maybe
considered prior to closure
88G. What is the expected prognosis following surgical intervention?
Keith & Wollstein1 showed that overall 90% of the patients reported
improvement in function following cubital tunnel decompression. Complete
recovery is less frequent than in carpal tunnel syndrome. Poor prognostic factors
include:
a. Symptoms existing for over one year before surgery
b. Severe existing atrophy
c. Alcoholism
d. Absence of response to electrical stimulation of the nerve
89A. What is your differential diagnosis?
The clinical picture shows the loss of the normal cascade of the fingers when
the hand is in the resting position. Given the acute hyperextension injury, the
likely diagnosis is of flexor digitorum proundus (FDP) avulsion of the little finger.
Other differentials include a trigger finger. Rupture of the flexor tendons could
be secondary to rheumatoid arthritis, cystic degeneration, post-fracture, or
calcification of the triangular fibrocartilage complex (TFCC).

89B. How would you confirm your diagnosis?


The diagnosis would be confirmed with a full history to rule out antecedent
pathology and a thorough clinical examination. This would include separate
Hands 101

testing of the FDP and flexor digitorum superficialis (FDS) tendons. In FDP
avulsion the avulsed fragment can be felt as a mass over the proximal phalanx
or sometimes in the palm. Radiographs may help localise the position of the
retracted tendon. MRI allows for accurate pre-operative assessment of tendon
position and degree of retraction, thereby facilitating surgical planning and
approach. Some centres advocate the use of ultrasound rather than MRI.

89C. Which finger is commonly involved?


The ring finger is most commonly affected (in 75% of patients). The ring fingertip
is usually the most prominent or “longest” during grip in 90% of cases and that
it absorbs more force than any other finger during pull-away testing. Secondly
the flexor digitorum profundus tendon, as demonstrated in cadaver specimens
indicates a significantly weaker insertion of the ring finger compared with the
middle finger. These factors contribute to the susceptibility of the ring finger to
the profundus avulsion injury.2

89D. What is the classification system associated with this problem?


Leddy and Packer3 have classified profundus avulsions. In type I, the tendon
retracts to the palm, held up by the lumbrical origin with disruption of the entire
vincular system. Type II avulsion is characterized by retraction of the tendon to
the PIP level that spares the vinculum longum, presumably maintaining blood
supply. In type III, the profundus tendon avulsion occurs in addition to a fracture
of the distal phalanx base.

89E. What are the treatment options?


Treatment can either be operative or non-operative. Return to function is
significantly better with surgical intervention. Options include re-attachment to the
distal phalanx, tendon grafts, staged reconstruction, tenodesis and arthrodesis.

89F. How would you like to treat this patient?


The factors that influence treatment are:
1. the length of time between injury and treatment
2. tendon retraction
3. blood supply to the fragment
4. the presence of bony fragments. My preferred treatment would depend on all
of the above. An early diagnosed Type 1 injury would be amenable to delivery
of the tendon from the palm and re-attachment to the distal phalanx with a
pull-out wire. For three weeks the limb is immobilized in a dorsal splint with
the wrist in flexion, MCPJ in 70° of flexion and the IPJs in extension. The wire is
then removed and hand therapist completes the rehabilitation. Late injuries
can be treated by tendon grafting, tenodesis or arthrodesis. Type 2 and 3
injuries can be repaired at a later date because of preservation of circulation.
90A. What is your diagnosis?
This is a radiograph of a chronic scapholunate (SL) injury with a dorsal
intercalated segmental instability (DISI) deformity.
102 Chapter 8  Viva Answers

90B. What is the classification system associated with this problem?


These injuries are assessed and best treated based on which restraints are
damaged and the duration from the actual index event.
Garcia-Elias et al4 set out the following assessment criteria in the assessment of SL
injuries.
• Is the dorsal scapholunate ligament partially or completely torn?
• If complete, can the ligament be repaired and what is the healing potential?
• What is the status of the secondary scaphoid stabilizers (i.e. is the
radioscaphoidangle less than 45°)?
• Is the carpal malalignment reducible?
• Are the cartilaginous surfaces normal?
Based on these conditions SL injuries can be classified as per Table 8.1:

Table 8.1

Stages Pathology
S1: Pre-dynamic SLIL partial tear
S2: Dynamic Complete SLIL injury and repairable DSL

S3 : Static no DISI Complete nonrepairable D and VSL, no DISI STT


ligaments intact

S4: Static with reducible DISI Complete scapholunate ligament injury with a
non-repairable reducible rotary subluxation of
the scaphoid

S5: Static with irreducible DISI Complete scapholunate ligament injury with
irreducible malalignment but normal cartilage

S6: Static with irreducible DISI and degenera- Complete scapholunate ligament injury with
tive change irreducible malalignment and cartilage degen-
eration

90C. What stage/grade would you assign to this radiograph?


This appears to be a static deformity. The reducibility can be checked on table
during surgery. It is likely that this is stage 4 to 5 injury based on radiographs.

90D. What is the natural history of this condition?


If left untreated chronic scapho-lunate dissociation will lead to a scapholunate
advanced collapse (SLAC) wrist. This describes arthritic changes that result due
to abnormal loading. The first joint to develop degenerative changes is the radio-
scaphoid, then capitolunate and then the scapho-trapezoid/trapezium joints.

90E. What are the options for treatment?


See Table 8.2
Hands 103

 Table 8.2 Treatment

Stage Pathology Management


S4: Static with Complete scapholunate Ideal indication for the tendon weaves.
reducible DISI ligament injury with a 3 LT tenodesis (Figs 8.1a and b) of Brunelli
nonrepairable reducible rotary procedure63
subluxation of the scaphoid

S5: Static with Complete scapholunate Better to accept the morbidity caused by
irreducible DISI ligament injury with irreducible a partial fusion than to attempt a ligament
malalignment but normal reconstruction that most likely will fail.
cartilage Options: STT, scaphocapitate fusions.

a b
Figure 8.1a and b

90F. How would you treat this patient?


See Table 8.2

91A. What is the diagnosis?


The radiological abnormalities and clinical finding would be consistent with a
diagnosis of a seronegative spondyloarthopathy affecting the hand. In this case
radiographic features of psoriatic arthritis are present.

91B. Which skin condition is associated with this problem?


Psoriasis.

91C. What are the characteristic radiological features?


The classic radiographic features of psoriatic arthritis that can be seen in this
case include joint space narrowing, peri-articular joint erosions, osteolysis,
PIPJ/DIPJ ankylosis and the development of a “pencil-in-cup” deformity of
104 Chapter 8  Viva Answers

marked lysis of the distal end of a phalanx with bony remodelling of the
proximal end of the more distal phalanx. Other radiological signs include
resorption of the distal phalanges i.e. Morningstar appearance, bony
proliferation including shaft and periarticular peri-ostitis, spur formation and
spondylitis. Radiological changes in psoriatic arthritis are often asymmetric
and oligoarticular, most commonly involving the carpus, MCP, PIP, and DIP
joints.

91D. What is the natural history of this condition?


Psoriatic arthritis is a pleomorphic disease that can affect any joint and has
a variable course and prognosis. It was initially considered to be less severe
than rheumatoid arthritis. However, up to 20% of patients having a severe,
debilitating form of degenerative arthritis. Patients with psoriatic arthritis
experience intermittent symptomatic flares, with variable lengths of intervening
remission. The natural history of the disease varies by subtype of clinical and
radiologicalpresentation.
Risk factors for severe, progressive destructive disease include female gender,
polyarticular disease at presentation, younger age at symptom onset, and
acute onset of arthritis. The mortality rate of patients is higher than that of
the general population. The disease leads to significant functional disability
and a reduced quality of life. At time of diagnosis almost a third of patients
are bedridden or have limited their activities of daily living to self-care.
Progression of clinical damage is seen in the majority of patients, and only
small percentages achieve complete, prolonged remission without therapeutic
intervention.5

92A. Describe the abnormalities seen in the photograph and the


radiograph.
The photograph and the radiograph show polydactyly of the thumb, with
duplication of the proximal and distal phalanges and a broad based metacarpal
head.

92B. Classify this abnormality.


The Wassel6 classification is used to determine the different types of duplication;
• Type I: Bifid distal phalanx
• Type II: Duplicated distal phalanx
• Type III: Bifid proximal phalanx
• Type IV: Duplicated proximal phalanx (most common)
• Type V: Bifid metacarpal
• Type VI: Duplicated metacarpal
• Type VII: Triphalangia
This case is classified as a Type IV, which is the most common form (43%).7
Hands 105

92C. Discuss the common inheritance patterns of this condition.


Isolated duplication is normally unilateral and sporadic, whereas a triphalangeal
thumb usually arises form autosomal dominant inheritance. Type VII is also
associated with several syndromes such as Holt-Oram syndrome, Fanconi’s
anemia, Blackfan-Diamond anemia, imperforate anus and cleft palate.

92D. What are the main indications for surgical treatment in this
condition?
Surgical intervention is almost always indicated for better function and cosmetic
reasons. This is normally performed from the age of 18 months to 5 years. The
type IV polydactyly patient should be treated very early, because if one waits
too long, the supernumerary component displaces the normal component into
marked radial or ulnar deviation, and growth continues in this direction.

92E. What are the principles when considering surgical treatment of


this particular case?
Surgical intervention in this Type IV duplication involves excision of the most
hypoplastic (usually radial) thumb, narrowing of the MCPJ articular surface,
ligament (especially radial collateral) reconstruction, intrinsic transfer and
possibly centralisation of the extrinsic flexor and extensor tendons.8

93A. What are the differential diagnoses?


The differential diagnoses here would include the following:
• Mucous cyst
• Tumour (benign/malignant)
• Xanthoma
• Abscess
93B. What is the diagnosis?
The diagnosis is a mucous cyst with associated breakdown of the skin. There are
degenerative changes in the DIPJ (dorsal osteophytes).

93C. What are the causes of this condition?


Osteoarthritis of the DIPJ and trauma can cause this condition. There is mucoid
degenration of the connective tissue.

93D. How would you treat this patient?


I would take a detailed history and examination and ascertain the effect of
the cyst on function of the finger and the patient’s symptoms. Conservative
measures include massage and compression. Aspiration can be attempted but
re-accumulation is likely.
The definitive management is excision of the cyst, which can result in loss of skin
106 Chapter 8  Viva Answers

over the ulcerated area. In order to achieve skin coverage a flap can be rotated
using the excess skin over the PIP joint. There are several ways of doing this, one
of which is demonstrated below in Fig. 8.2a.
The procedure is performed using a digital block and a digital tourniquet for
haemostasis. The flap is marked and with the area of skin ulceration that needs to
be excised (Fig. 8.2b).
The flap is the raised and rotated to cover the defect (Fig. 8.2c). This is facilitated

a c

Figure 8.2a to c

by a proximal release (arrow) of the flap, which allows rotation and coverage. Any
osteophyte associated with the cyst must be removed to decrease chances of
recurrence.

93E. What are the possible complications?


Recurrence, nail deformities, risk of nerve/tendon/vessel damage (rare).9

94A. What is your diagnosis?


The diagnosis is a right-sided distal biceps tendon rupture. In addition due to
the obvious retraction of the biceps muscle belly it is likely that the bicipital
aponeurosis is also not intact.

94B. How does this injury usually occur?


This occurs due to eccentric loading of the flexed elbow during activity such as
weight lifting. Almost solely in middle-aged males, with steroid and tobacco use
being risk factors.

94C. What are the treatment options?


Treatment options are non-operative but this leads to achievement of 50%
normal supination strength and 70% normal flexion strength at one year.
Surgical intervention is recommended for active individuals and can be
Hands 107

performed through a single/dual incision technique. Fixation techniques using


suture anchors/interference screws and an endobutton have all been described.

94D. What is the prognosis if the patient opted for surgical


intervention?
Peeters et al10 found that at 16 months following fixation using an endobutton
showed excellent functional outcomes with an almost full range of movement.
80% of flexion strength and 91% of supination strength was regained. One out
of 26 patients required removal of the endobutton and there were no other
significant complications. Other reported complications in other series are
heterotopic ossification, radioulnar synostosis, infection and neurological injury.

95A. Describe the radiograph.


There is a comminuted intra-articular fracture at the base of the middle phalanx
affecting the PIP joint. The fracture affects > 50% of the articular surface and there
is subluxation of the joint.

95B. What is your diagnosis?


Hastings type III unstable dorsal fracture subluxation of the PIPJ. Dorsal dislocations
of the PIP joint are often associated with palmar lip fractures and can be classified
based on the degree of stability. When instability persists with greater than thirty
degrees of flexion at the PIP joint to maintain reduction, the cup shaped geometry
of the volar lip of the middle phalanx will need to be reconstructed.
The amount of articular surface involvement and the resultant clinically
demonstrated instability can be used to group palmar lip PIP fracture dislocations
into one of three categories: stable, tenuous, and unstable. Stable fractures involve
< 30% of the joint surface and maintain a congruent reduction, throughout the joint
excursion. Tenuous fractures will show an involvement of 30 to 50% and are unstable
injuries. Unstable injuries are those in which > 50% of the articular surface is involved.
Also included in this group are fractures that remain unstable at 30° of flexion.

95C. How is this injury assessed?


A detailed history and examination is necessary. The history should include hand
dominance, profession, hobbies/sports and to what level these are participated in
(professional/amateur). The examination should include the neurovascular status of
the digit and record any laceration, bruising and areas of tenderness. The alignment
(coronal, sagittal and rotational) and swelling of the digit should be noted. Under
a digital nerve block the range of movement and subluxation can be assessed.
One should also document the status of the tendons FDS, FDP and the central slip,
although the latter is more commonly injured in volar PIPJ dislocations.
Radiologically the injury is assessed with an AP and true lateral radiograph of
the digit. Look for the “V” sign dosally at the PIPJ on the lateral radiograph, which
represents subluxation and an incongruent joint.
108 Chapter 8  Viva Answers

A CT scan may give useful information about the fracture configuration if one is
considering an ORIF.

95D. What are the options for treatment?


The joint is subluxed and incongruent, therefore the aim of treatment is to reduce
the joint and hold the reduction to allow healing of the fracture in a favourable
position. Early movement is desirable in order to minimise stiffness. The options
of treatment are broadly operative or non-operative.
Table 8.3 lists the various options with pros and cons of each. This particular
injury is unstable and affects more than 50% of the joint surface; therefore,
extension block splinting is unlikely to be effective. The other treatment options
listed here are all possible in this patient.

Table 8.3

Treatment Pros Cons


Option
Extension block Treatment of choice for most Requires careful assessment of congruence
splinting < 40% articular surface volar and follow-up, not effective in most > 40%
base fractures. articular surface volar base fractures. With a
greater amount of joint surface involved the
blocking must begin at a higher angle. May
have a residual fixed flexion contracture.

Extension block Selected volar base fractures Pin tract infection.


pinning that reduce spontaneously with
PIP joint flexion.

ORIF Can restore articular congruity Stiffness, infection. May not be an option if
and may permit early range of fracture is comminuted.
movement.
Dynamic Allows some movement. Pin tract infection.
external fixator
Volar plate Specific strategy for subset of Numerous specific anatomic requirements
arthroplasty volar base fractures involving must be met for procedure to be effective;
between 30–60% of the articular surface is not hyaline cartilage.
surface.
Osteochondral Bone to bone healing, true hya- Technically complex; graft resorption and
reconstruction line cartilage restoration, recre- collapse are possible.
ates volar lip buttress effectively.
Hands 109

95E. How will you manage this patient?


It is important to inform the patient that this is a serious injury and no matter
how it is treated, the joint will have significant stiffness and swelling. I would
seek advice from a specialist hand surgeon, especially as the patient is a pianist.
I would inform the patient that for this injury there is no single treatment option
that is proven to give a superior outcome.
Stable palmar lip fractures can be treated by early range of motion exercises with
buddy taping and protected motion. PIP joint hyperextension should be avoided
with the help of extension block splints.
Tenuous PIP fracture dislocations can be treated with static splinting but
this does not lend itself to early motion. Dorsal extension block splinting
(McElfresh) maintains reduction by limiting extension at the PIP joint while
allowing for early active flexion. Extension block pinning can also be used to
limit extension.
In unstable fractures, treatment options include open reduction and internal
fixation, palmar plate arthroplasty, external fixation and hemi hamate autograft.
Open reduction is suitable for large minimally comminuted palmar lip fractures.
Depressed fragments can be elevated and the buttressing effect of the volar lip
can be restored.
In the presence of extensive comminution a number of traction devices can be
used to treat unstable fractures. However these systems in addition to being
bulky may fail to restore the volar lip, resulting in recurrent dislocation and post-
traumatic arthritis.
Eaton in 1980 advanced the palmar plate into the defect after excising the
comminuted fragment.
Ishida advocated the use of an osteochondral graft to resurface the damaged PIP
joint in 1994. Hill Hastings in 1999, noting the similarity in shape of the middle
phalanx base and the distal hamate, described the use of a hamate allograft in
the treatment of PIP fracture dislocations.
A volar approach is used and the joint is “shotgunned” (Fig. 8.3a). With the
fracture prepared (Fig. 8.3b) the defect is measured and a matching graft is
harvested from the distal hamate (Figs 8.3a to c). This is then fixed to the base of
the middle phalanx with screws of adequate length (Fig. 8.3e).

Further Reading
Williams RM, Hastings H 2nd, Kiefhaber TR. PIP fracture/dislocation treatment technique: use of a
hemi-hamate resurfacing arthroplasty. Tech Hand Up Extrem Surg 2002;6:185-92.
Williams RM, Kiefhaber TR, Sommerkamp TG, Stern PJ. Treatment of unstable dorsal proximal
interphalangeal fracture/dislocations using a hemi-hamate autograft. J Hand Surg Am
2003;28:856-65.
110 Chapter 8  Viva Answers

a b

c d

e
Figure 8.3a to e

96A. What is your diagnosis?


Dupuytren’s nodes.

96B. What is the cause of pain in this condition?


Pain is not always a feature of Dupuytren’s but may be present in the initial
presentation of nodules due to local pressure effects.
Work undertaken by von Campe et al11 showed that at dissection tiny nerve
fibres were found passing through excised nodules. Biopsies showed nerve fibres
embedded in the dense fibrous tissue in six of their cases.
The authors also found neuromas and the presence of intraneural or perineural
fibrosis. The authors concluded that the findings may be a consequence of local
nerve compression by the fibromatosis. Fibrous tissue tends to be inelastic
thus persistent compression of a nerve may be followed by decrease in axonal
Hands 111

transport and nerve perfusion resulting ultimately in a fibrosis of the nerve and
the development of a pseudoneuroma.
Enlarged Pacinian corpuscles are found in Dupytren’s disease. It was theorised
that the increase in density and size of the corpuscles may be related to the
release of local nerve growth factors. Such enlarged Pacinian corpuscles can be
present in patients with no pain but can also cause pain.12
In the paper by von Campe et al11 the authors discuss the association of
Dupuytrens disease with sprouting of substance P positive nerve fibres which have
been implicated in causing pain in Achilles tendinosis. This pathophysiological
mechanism, it was felt, may explain pain in the early stages of Dupuytrens disease.

96C. How would you stage this condition?


I would use the Woodruff classification system:13
• Stage 1 - early palmar disease with no contracture (as in this case)
• Stage 2 - one finger involved with MCPJ contracture
• Stage 3 - one finger MCPJ + PIPJ
• Stage 4 - stage 3 + more than one finger involved
• Stage 5 - finger in palm deformity
96D. What is the treatment for this condition?
Patients with persistently painful nodules for greater than a year can be offered
excision. However the evidence base for this is poor. The authors (von Campe
et al)11 acknowledge that their numbers were small and that their study did not
include controls.

96E. What are the possible complications of surgical intervention?


A 20-year review of surgical complications associated with fasciectomy for
Dupuytren’s contracture showed that major complications occurred in 15.7% of
cases, including digital nerve injury (3.4%), digital artery injury (2%), infection
(2.4%), hematoma (2.1%), and complex regional pain syndrome (5.5%), in
addition to minor complications including painful flare reactions in 9.9% of cases
and wound healing complications in 22.9% of cases. Amputation may be required
in cases of digital artery injury.14

97A. What procedure has this patient had?


EIP (extensor indicis proprius transfer) for rupture of the extensor pollicis longus.

97B. What deficit would the patient have prior to surgery?


The extensor pollicis longus originates from the middle third of posterior surface of
the ulna and the radioulnar interosseous membrane. It inserts into the dorsal base
of the thumb distal phalanx through the thumb extensor mechanism. Its function is
to retropulse the thumb at the interphalangeal joint. In simple terms retropulsion is
the term used for lifting the thumb off a table while keeping the hand flat.
112 Chapter 8  Viva Answers

97C. What are the causes of this pathology?


Most ruptures of the extensor pollicis longus (EPL) occur at the level of Lister’s
tubercle and are the consequence of mildly displaced distal radial fractures
that have healed, leaving a rough surface on the floor of the third extensor
compartment. A combination of tendon attrition and reduced blood supply to
such a tight compartment may explain the late appearance of the rupture.
Reports of EPL tendonitis secondary to an isolated fracture of Lister’s tubercle or
due to a direct injury to the tendon without a distal radial fracture is much less
common.
Ferreres et al15 also have demonstrated wrist hyperextension injury as a possible
cause of EPL rupture.
Spontaneous rupture of the EPL has also been described in a patient with
long-term systemic steroid usage.16

97D. Which other tendon transfers have been described?


Mensch in 1925 was the first to describe the use of extensor indicis for tendon
transfer and this is the most commonly used tendon. Extensor carpi radialis
longus (ECRL), extensor pollicis brevis (EPB) and an intercalated tendon graft
have also been used. APL is unsuitable for reconstruction of EPL due to its small
amplitude of movement, and it also has a different direction of pull. However,
Chitnis and Evans17 used APL in their study of 20 patients and described
satisfactory objective and subjective results.

97E. What are the principles of a tendon transfer?


Principles when deciding on tendon transfers are:
• Match muscle strength
• Force should be proportional to cross sectional area
• Amplitude should be proportional to length of the muscle
• Work capacity = force × amplitude
• Motor strength will decrease one grade after transfer
• Appropriate tensioning
• Appropriate excursion (can adjust with pulley or tenodesis effect)
Requirements also include patient compliance, no joint contractures, no active
infection and grade 5/5 power (this will drop one grade).

98A. Describe the clinical photograph.


Clinical photograph of macrodystrophia lipomatosa (ML) which is a rare cause
of congenital macrodactyly, associated with progressive proliferation of all
mesenchymal elements, but primarily the fibroadipose tissue.18
Unusually it may involve the whole upper limb and abdominal wall involvement
has been reported.19 Clinically the condition may present as early as the neonatal
Hands 113

period to late adulthood. There is a male preponderance and usually a single


sclerotome appears to be involved.
Commonly the lateral aspect of the upper limb along the median nerve
distribution appears to be involved while the medial aspect of the lower limb (i.e.
along the plantar nerve distribution) is can be affected.
The lower limb is more often involved than the upper limb and the 2nd and
3rd digits are more commonly affected. Growth usually stops with the onset of
puberty.

98B. What is the differential diagnosis?


There are certain conditions, which can present with localized limb
hypertrophy. The differential diagnosis of ML includes neurofibromatosis
type 1 (plexiform neurofibroma), fibrolipomatous hamartoma (FLH),
lymphangiomatosis, hemangiomatosis and Klippel–Trénaunay–Weber
syndrome, Mafucci syndrome, Ollier disease and Proteus syndrome. It is
important to note that in all these conditions there is a positive family
history and these conditions are characterized by cutaneous or systemic
manifestations.
Fibrolipomatous hamartoma (FLH) is a condition in which digital overgrowth
occurs like ML. It usually presents as an isolated nerve lesion and associated with
intramuscular fat deposition. But in ML, in addition to deposition of fat in nerve
sheaths, subcutaneous and muscle compartment, there is periosteal involvement
leading to the bony changes including exostosis, fatty invasion of the medullary
cavity, hypertrophy and ankylosis.20,21

98C. What is the most likely diagnosis?


Macrodactyly

98D. What is the pathophysiology of this condition?


Several theories exist as to the cause of macrodatyly including abnormal nerve and
vascular supply to the digits although none have been proven. Macrodactyly may
be associated with neurofibromatosis, tuberous sclerosis and Maffuci syndrome.

98E. What is the treatment of this condition?


The usual presentation is with problems associated with cosmesis. However
patients may present with secondary osteoarthritis or compression of
neurovascular structures resulting in entrapment syndromes.22–24
Surgical usually takes the form of a debulking procedure and is dependent on the
patient’s symptoms, age, and extent and severity of the disease. A conservative
approach is advocated as growth usually stops at puberty also the incidence of
nerve injury following extensive debulking or lesion removal is approximately
30% to 50%. A high recurrence rate of 33% to 60% has been reported. In a
localised forms of the disease ray amputations have been described.
114 Chapter 8  Viva Answers

99A. Describe the radiographs.


The radiographs are AP lateral and oblique views centred on the index finger
metacarpophalangeal joint (MCPJ). They show a complex dislocation of the MCPJ
of the index finger and is usually associated with significant trauma. These injuries
are uncommon and early recognition is crucial in the management of this injury.
The index finger is commonly involved and dorsal osteochondral fractures are
reported in 50% cases in some series. Radiological features may be subtle and
three radiographic views are mandatory. The AP view may show an ulnar drift
of the proximal phalanx. The true lateral is of limited value due to overlap of
the other fingers. The third oblique view is most useful and at times may show
interposition of a sesamoid, which is pathognomonic of a complex dislocation of
the MCPJ.

99B. What is the diagnosis?


Any dislocation associated with a fracture is termed complex. Thus this is a
complex dorsal dislocation of the metacarpophalangeal joint of the left index
finger.

99C. What are the causes of irreducibility?


Causes of irreducibility may be interposition of any one of the following
structures:
• Metacarpal head “button-holed” between the flexor tendon (ulnar side) and
the lumbrical (radial side); most common cause.
• Volar plate
• Sesamoid bones
• Superficial transverse metacarpal ligament
• Natatory ligaments
99D. What is the treatment?
Complex dorsal dislocations will almost always require open reduction. Attempts
at closed reduction and excessive traction make the “noose” tighter and result in
excessive trauma to the affected digit. Closed reductions may be attempted in the
thumb, open injuries or subluxed MCPJs which present with significant deformity.
Excessive traction runs the risk of converting a simple subluxation into a complex
dislocation. The preferred treatment for complex dislocations is open reduction.
The MCPJ may be approached through a standard extensile volar approach. In
the index finger the radial digital nerve lies in close proximity to the to the first
lumbrical and the dislocated metacarpal head and great care must be exercised
as injuries to this nerve are well documented. A key step in reducing the tension
around the metacarpal head is to release the A1 pulley thereby allowing a degree
of play of the flexor tendons. The use of a freer elevator placed under the 1st
Lumbrical can be used to replace the dorsally trapped volar plate. Simultaneously
the flexor tendons are retracted in a volar direction and an assistant applies distal
traction holding the index finger.
References 115

A dorsal approach may also be used. It has the advantage of being technically
simpler and does not involve encroach upon the digital nerve and allows for
treating associated dorsal osteochondral fractures. However it does entail
resection of the entrapped volar plate and can in theory cause MCP joint
instability. However there is no published work to support this view.

99E. What is the prognosis?


Previous reports are limited to small case series focusing on surgical approaches.
Barry et al25 presented four patients who regained nearly full function after open
reduction. With a concentric reduction, it seems likely that patients will regain the
majority of their pre-injury motion within four to six weeks. Similar good results
were reported in seven cases treated with a volar approach, all regaining normal
ROM, power and sensibility.26

References
1. Keith J, Wollstein R. A tailored approach to the surgical treatment of cubital tunnel syndrome.
Ann Plast Surg. 2011;66:637-9.
2. Bynum DK Jr, Gilbert JA. Avulsion of the flexor digitorum profundus: anatomic and biomechanical
considerations. J Hand Surg Am 1988;13:222-7.
3. Leddy JP, Packer JW. Avulsion of the profundus tendon insertion in athletes. J Hand Surg Am
1977;2:66-9.
4. Garcia-Elias M, Lluch AL, Stanley JK. Three-ligament tenodesis for the treatment of scapholunate
dissociation: indications and surgical technique. J Hand Surg Am 2006;31:125-34.
5. Manuel J, Moran SL. The diagnosis and treatment of scapholunate instability. Hand Clin
2010;26:129-44.
6. Strauss EJ, Alfonso D, Baidwan G, Di Cesare PE. Orthopedic manifestations and management of
psoriatic arthritis. Am J Orthop (Belle Mead NJ) 2008;37:138-47.
7. Wassel HD. The results of surgery for polydactyly of the thumb: a review. Clin Orthop Relat Res
1969;64:175-93.
8. Yen CH, Chan WL, Leung HB, Mak KH. Thumb polydactyly: clinical outcome after reconstruction. J
Orthop Surg (Hong Kong) 2006;14:295-302.
9. Miller MD. Review of orthopaedics. Fifth ed. Philadelphia: Saunders Elsevier, 2008.
10. Peeters T, Ching-Soon NG, Jansen N, et al. Functional outcome after repair of distal biceps tendon
ruptures using the endobutton technique. J Shoulder Elbow Surg. 2009;18:283-7
11. von Campe A, Mende K, Omaren H, Meuli-Simmen C. Painful nodules and cords in dupuytren
disease. J Hand Surg Am 2012;37:1313-18.
12. Fletcher CD, Theaker JM. Digital pacinian neuroma: a distinctive hyperplastic lesion.
Histopathology 1989;15:249-56.
13. Woodruff MJ, Waldram MA. A clinical grading system for Dupuytren’s contracture. J Hand Surg Br
1998;23:303-5.
14. Denkler K. Surgical complications associated with fasciectomy for Dupuytren’s disease: a 20-year
review of the English literature. Eplasty 2010;10:e15.
15. Ferreres A, Llusá M, García-Elías M, Lluch A. A possible mechanism of direct injury to the EPL
tendon at Lister’s tubercle during falls with the wrist fully extended. J Hand Surg Eur Vol
2008;33:149-51.
16. Anwar I, Owers KL, Eckersley R. Spontaneous rupture of the extensor pollicis longus tendon. J
Hand Surg Br 2006;31:457-8.
17. Chitnis SL, Evans DM. Tendon transfer to restore extension of the thumb using abductor pollicis
longus. J Hand Surg Br 1993;18:234-8.
18. Krengel S, Fustes-Morales A, Carrasco D, et al. Report of eight cases and review of literature.
Pediatr Dermat 2000;17:270–6.
116 Chapter 8  Viva Answers

19. Aydos SE, Fitoz S, Bökesoy I. Macrodystrophia lipomatosa of the feet and subcutaneous lipomas.
Am J Med Genet A 2003;119:63-5.
20. Jain R, Sawhney S, Berry M. CT diagnosis of macrodystrophia lipomatosa: a case report. Acta
Radiol 1992;33:554-5.
21. D’Costa H, Hunter JD, O’Sullivan G, et al. Magnetic resonance imaging in macromelia and
macrodactyly. Br J Radiol 1996;69:502-7.
22. Oztürk A, Baktiroğlu L, Oztürk E, Yazgan P. Macrodystrophia lipomatosa: a case report. Acta
Orthop Traumatol Turc 2004;38:220-3 (in Turkish).
23. Watt AJ, Chung KC. Macrodystrophia lipomatosa: a reconstructive approach to gigantism of the
foot. J Foot Ankle Surg 2004;43:51-5.
24. Brodwater BK, Major NM, Goldner RD, Layfield LJ. Macrodystrophia lipomatosa with associated
fibrolipomatous hamartoma of the median nerve. Pediatr Surg Int 2000;16:216-18.
25. Barry K, McGee H, Curtin J. Complex dislocation of the metacarpophalangeal joint of the index
finger: a comparison of the surgical approaches. J Hand Surg 1988;13:466-8.
26. Durakbasa O, Guneri B. The volar surgical approach in complex dorsal metacarpophalangeal
dislocations. Injury 2009;40:657-9.
Chapter 9
Viva Questions:
Children’s orthopaedics
100. Here is the radiograph of an infant who presented with a history of a fall from
a chair and inability to move her right arm (Fig. 9.1).
A. What is the diagnosis and how would you manage the case?

Figure 9.1
118 Chapter 9  Viva Questions

Here are a photograph and radiograph of a child who presented with an


unstable right knee and foot deformities (Figs 9.2a and b).
B. What is the diagnosis and how would you manage this child?

a b

Figure 9.2a and b


Children’s orthopaedics 119

101. This child has no complaints but her parents are disappointed about the shape
of her left upper limb one year after an elbow injury (Figs 9.3a and b). They
want an explanation and a remedy.
A. How would you manage the case?

b
Figure 9.3a and b
120 Chapter 9  Viva Questions

Here is a radiograph (Fig. 9.4) showing a recent fracture of the upper tibial
metaphysis in a three-year-old boy.
B. Whose name is associated with the injury, how might it cause a problem and
how should it be managed?

Figure 9.4

102. Here are clinical photographs and a radiograph of a 14-year-old girl who
complains of intermittent discomfort and loss of full extension and supination
of the right elbow and forearm (Fig. 9.5).
A. What is the diagnosis of the elbow disorder?
B. What is the underlying condition and how could you confirm this?
C. How would you treat the elbow?
Children’s orthopaedics 121

Figure 9.5

Clinical photograph and radiograph of a five-year-old boy who walks on his


left heel (Fig. 9.6). The foot is painless but the foot deformity is progressive.
D. What are the deformities?
E. What is the likely cause?
F. How would you manage the condition?

Figure 9.6
122 Chapter 9  Viva Questions

103. Here is a pelvic radiograph of a six-year-old child with a dislocated right hip
(Fig. 9.7).
A. How would you manage the condition?

Figure 9.7

Here is the chest radiograph of a nine-year-old child (Fig. 9.8).


B. What is the diagnosis and how would you address the problem?

Figure 9.8
Children’s orthopaedics 123

104. This baby is about to undergo an operation (Fig. 9.9).


A. What is the condition and the operation planned for it?

Figure 9.9

Clinical photograph (Fig. 9.10a) and radiograph (Fig. 9.10b) of an active,


symptom-free boy with severe genu varum.
B. How would you manage the case?

a b

Figure 9.10a and b


124 Chapter 9  Viva Questions

105. A. What is this condition (Fig. 9.11), how would you manage it and what is the
likely outcome?

Figure 9.11

B. Describe the foot deformities in the photographs (Figs 9.12a and b).
How should they be treated?

a b

Figure 9.12a and b


Children’s orthopaedics 125

106. This 11-year-old girl presented with a painless swelling of her right ring finger
(Figs 9.13a and b). The swelling was noted several months ago and it is slowly
increasing in size. She is concerned about the appearance.
A. What is the condition and how would you manage the case?

a b
Figure 9.13a and b

Here is the radiograph of a baby who has spontaneously developed an


increasingly swollen left forearm (Fig. 9.14). The infant is generally well but is
irritable when the arm is moved.
B. What is the diagnosis and how would you manage the condition?

Figure 9.14
126 Chapter 9  Viva Questions

107. Here are a clinical photograph and lateral radiograph of the spine of a six-
year-old girl who is of short stature (Figs. 9.15a and b)
A. What is the likely diagnosis and what are the manifestations and aetiology of
the condition?

a b Figure 9.15a and b

This 12-year-old boy is about to undergo an operation because of progressive


valgus of the left elbow and early symptoms of ulnar neuritis (Figs. 9.16a and b).
B. What is the condition and what operation would you undertake?

a b

Figure 9.16a and b


Children’s orthopaedics 127

108. A five-year-old who has never walked but wants to do so (Fig. 9.17). He has
fixed flexion deformities of his wrists and knees and severe, stiff talipes
equinovarus.
A. What is the diagnosis?
B. How would you manage the case?

Figure 9.17

The parents of this six-week-old baby have just noticed a swelling in the right
side of his neck (Fig. 9.18).
C. What is the likely diagnosis and how would you manage the case?

Figure 9.18
128 Chapter 9  Viva Questions

109. Clinical photograph and radiograph of a 12-year-old girl with a thoracic


kyphosis (Figs 9.19a and b).
A. Describe the radiological features.
B. What is the likely diagnosis?

a b
Figure 9.19a and b

Here are the clinical photographs of a four-year-old boy (Figs 9.20a and b). He
is of normal intelligence but can only walk with difficulty.
C. What is the condition in general terms?
D. What is the diagnosis?
E. How would you treat his feet?

a b

Figure 9.20a and b


Children’s orthopaedics 129

110. Figures 9.21a and b show the initial and post-operative radiographs of a ten-
year-old boy who injured his knee in a road traffic accident. Figure 9.21c is a
current radiograph, two years later.
A. Describe the original injury, the current radiological features and how you
would manage the case.

a b c

Figure 9.21a to c

This 14-year-old boy, previously normal, presented with a painless


prominence of his right scapula following a canoeing holiday (Fig. 9.22).
B. What is the condition, what are the differential diagnoses and, of these,
which is the most likely in this case?

Figure 9.22
130 Chapter 9  Viva Questions

111. Here is a clinical photograph and radiograph of a child who wants his feet
corrected (Figs 9.23a and b).
A. How would you manage the case?

b
Figure 9.23a and b

Here is the clinical photograph of an 11-year-old boy (Fig. 9.24).


B. What is the condition?
C. What are the clinical features?
D. What are the principles of management?
Children’s orthopaedics 131

Figure 9.24
Chapter 10
Viva Answers:
Children’s orthopaedics
100A. What is the diagnosis and how would you manage the case?
The radiograph shows an undisplaced spiral fracture of the humerus. This would
be unexpected from the history and raises the suspicion of non-accidental
injury.
The child should be referred to the paediatricians and, if further investigations
are indicated, these should include a skeletal survey and a bone scan to look for
injuries at different times elsewhere.
In this case the chest X-ray showed healing fractures of the ribs (Fig. 10.1a)
and the bone scan (Fig. 10.1b) showed evidence of activity in several areas,
compatible with earlier trauma.

a b
Figure 10.1a and b
134 Chapter 10  Viva Answers

100B. What is the diagnosis and how would you manage this child?
The diagnosis is Larsen’s syndrome, the orthopaedic features of which are
hypermobility with multiple joint dislocations, usually present at birth, and extra
bones in the wrist and tarsus. The diagnosis can be confirmed by the double
ossification centre of the os calcis, as seen in this case (Fig. 10.2).
Function is usually good and most surgeons would not try to reconstruct the
joints, as operative treatment is likely to fail.
The child may therefore be helped by knee and foot orthoses
He should also be monitored through growth for spinal deformity, which can be
severe with neurological problems.

Figure 10.2

101A. How would you manage the case?


The diagnosis is cubitus varus following a supracondylar fracture. A full
explanation is necessary. Regardless of the initial treatment, there is a significant
number of malunions after this injury. It is also important to explain that, although
the deformity would be permanent, there is usually no functional disability and
correction of the deformity by valgus supracondylar osteotomy is for appearance.
As with all cosmetic operations, the patient has to want the appearance
improved and their expectations may not be realised because it may not be
possible to correct the deformity completely and there will be a scar. Therefore,
do not offer supracondylar osteotomy unless all these points are understood
and the child herself finds the deformity unsightly.

101B. Whose name is associated with the injury, how might it cause a
problem and how should it be managed?
The radiograph shows a Cozen’s fracture of the proximal tibial metaphysis.
This fracture can lead to a valgus angular deformity, even if the fracture
Children's orthopaedics 135

appears to be undisplaced. I would manage this patient by taking a full history


and performing a thorough clinical examination. I would warn the parents
about the risk of later deformity. This may result from asymmetrical growth
stimulation of the proximal tibial physis following the injury. It would be best
managed in a long-leg straight cast with varus moulding. Regular radiographs
should be taken for the first three weeks and the cast wedged appropriately.
If a suitable reduction was not possible and the fracture became further
displaced, open reduction and removal of periosteum or even the pes
anserinus from the fracture site, followed by fixation may be necessary to
allow for suitable reduction and stabilisation.

102A. What is the diagnosis of the elbow disorder?


The radiograph of the elbow demonstrates a posterior dislocation of the radial
head.1 The radial head looks convex. This probably represents a congenital
dislocation of the radial head and is often associated with a hypoplastic
capitellum. Additionally there may be both shortening and increased bowing of
the abnormal radius.

102B. What is the underlying condition and how could you confirm
this?
Nail–patella syndrome.
Examination and radiographs of the knees are likely to show absence or
hypoplasia of the patellae and a pelvic radiograph may show iliac horns. The
inheritance is autosomal dominant so siblings and parents may be affected.

102C. How would you treat the elbow?


The symptoms are not bad enough for operative treatment but if they became
so the radial head could be excised.

102D. What are the deformities?


Calcaneo-cavus and tibial bowing.

102E. What is the likely cause?


Osteomyelitis with destruction of the lower tibial growth plate.

102F. How would you manage the condition?


The foot should be made plantigrade by correction through the ankle and lower
fibula. The tibial deformity may be left to mature and remodel but may need
correction depending on progress and resources.

103A. How would you manage the condition?


The radiograph shows a right-sided high hip dislocation. There is delayed
ossification of the femoral head and an increased acetabular index. These
136 Chapter 10  Viva Answers

findings are consistent with developmental dysplasia of the hip. I would take
a thorough history from the parents and the child and perform a full clinical
examination. I would enquire about treatment to date, current symptoms and
co-morbidities. I would fully explain the condition and the future prognosis.
The child has a myelomeningocele. The spinal abnormality, ventriculo-
peritoneal shunt and bowel stasis are obvious. There is little acetabular
dysplasia, indicating the dislocation is related to muscle weakness. The child
is a non-walker and the position of the hip with regard to sitting and propped
standing is good. The child should be managed non-operatively.

103B. What is the diagnosis and how would you address the problem?
The diagnosis is a Sprengel deformity on the right. Management depends on
cosmetic and functional disability. In this case the condition is not severe and
there is no associated Klippel–Feil anomaly. If the appearance is a problem, it
could be addressed by excision of the upper angle of the scapula with division
of any vertebral connection.
In more severe cases a vertical scapular osteotomy can be helpful. It is debatable
whether more extensive procedures are overall better as scarring can be a major
cosmetic disability and function may not be significantly improved.2,3

104A. What is the condition and the operation planned for it?
The condition is constriction band syndrome of the right lower leg. The distal
limb is significantly oedematous and a small toe is seen. It is possible that
there has been auto-amputation of the other toes. Surgical intervention
is based on limb salvage with release of the constriction bands and
Z-lengthening procedures or amputation of the limb, if it is felt that that
salvage is not feasible.

104B. How would you manage the case?


The photograph and the radiograph show bilateral genu varum. It is important
to know the age of the child, as varus is physiological before the age of two and
is definitely pathological after three. It is likely to be physiological as there is
symmetric flaring of the femora and tibiae. I would document height, weight
and percentiles for age. Short stature may point to pathological causes of genu
varum, including rickets and skeletal dysplasias. Other pathological causes
include Blount’s disease, trauma, infection, polio and spina bifida.
Examination includes full assessment of the back, pelvis, hips, knees and feet. In
addition rotational profile of the lower limb would be measured to document
any internal tibial torsion. The intercondylar distance is also important to
measure to determine progression.
Management of this case would again depend on the age. In a two-year-old
child, I would ask to see the child in a year’s time with repeat radiographs.
Above the age of three, I would investigate for pathological causes and
then consider intervention if the deformity was persistent or progressive.
Children's orthopaedics 137

This could include bracing, guided partial growth arrest at an older age, or
osteotomy.

105A. What is this condition, how would you manage it and what is the
likely outcome?
There is a calcaneovalgus deformity due to congenital postero-medial bowing
of the tibia and fibula. The condition should be treated by gentle stretching and
simple splintage during infancy.
The bowing will correct spontaneously during childhood but there will be mild
to moderate residual shortening of the order of 1–2 cm, which may need to be
addressed towards the end of growth.

105B. Describe the foot deformities in the photographs. How should


they be treated?
In the part a photograph, the feet rest in positions of forefoot varus and
hindfoot equinovarus.
In the part b image, the hindfoot on the baby’s left is correctible, indicating
residual forefoot varus, but not on the right, which indicates resistant congenital
talipes equinovarus.
The left foot does not require surgery. The hindfoot will respond to simple
stretching. The metatarsus varus is likely to resolve spontaneously over the
next few years. If it does not it can be helped by a period of casting or release of
abductor hallucis. Major surgery on the asymptomatic mid-foot is rarely justified
The right foot requires operative release, either a simple tendo achillis
lengthening or a more extensive release. Thereafter, any residual forefoot varus
should be addressed as described above.

106A. What is the condition and how would you manage the case?
The condition is a benign subungual exostosis. It is causing minor distortion of
the nail bed but there is no angular deformity and the growth plate is intact.
The exostosis will grow in proportion to the rest of the phalnx.
Because of the risk of damage to the growth plate with subsequent angulation
it would be wise to postpone removal of the exostosis for several years until the
physisis is closed or closing. At that time the nail bed could be carefully raised
intact to allow excision of the exostosis and replacement of the nail bed in order
to prevent distortion of the nail.

106B. What is the diagnosis and how would you manage the condition?
The condition is Caffey’s disease (Infantile cortical hyperostosis). There are
no laboratory tests to confirm the diagnosis which is usually reached by
exclusion of other possible conditions, such as osteomyelitis, neoplasm, scurvy,
hypervitaminosis A and child abuse.
138 Chapter 10  Viva Answers

The age at presentation is typical and, although the swelling may increase
alarmingly, the condition is self-limiting and will usually resolve spontaneously
over 6 to 9 months and leave no long-term sequelae.

107A. What is the likely diagnosis and what are the manifestations and
aetiology of the condition?
The likely diagnosis is Morquio-Brailsford disease (mucopolysaccharidosis Type
IVA), in which deficiency of lyzosomal enzymes required for the degradation
of polysaccharides leads to accumulation of keratan sulphate and chondroitin
6 sulphate.
The clinical and radiological manifestations include dwarfism, multiple
dysostoses, lumbar lordosis, hip dysplasia, odontoid hypoplasia (Fig. 9.15b),
joint laxity, large elbows and knees (genu valgum is common) and flat feet.
This 12-year-old boy is about to undergo an operation because of
progressive valgus of the left elbow and early symptoms of ulnar neuritis
(Figs 10.3a and b).

a b

Figure 10.3 a and b

107B. What is the condition and what operation would you undertake?
The diagnosis is nonunion of a minimally displaced fracture of the lateral
condyle of the humerus sustained seven years earlier.
Flynn4 outlined 3 criteria for surgical treatment of an established nonunion:
1. A large metaphyseal fragment
2. Displacement of less than 1 cm from the joint surface
3. An open, viable lateral condylar physis
It was thought that reconstruction of the elbow was inadvisable because it was
unlikely that joint congruency could be restored and major stiffness would ensue.
The operation undertaken was a supracondylar closing-wedge osteotomy with
transposition of the ulnar nerve.
Children's orthopaedics 139

One year later the boy was symptom-free with full movements and the
osteotomy had united (Figs 10.4a to c).

c
Figure 10.4a to c

108A. What is the diagnosis?


The diagnosis is arthrogryposis multiplex congenita (AMC), causing severe
weakness and stiffness.

108B. How would you manage the case?


The child was managed as follows:
• Full functional assessment with a physiotherapist
• Clear goals of treatment. The aim was to achieve independent walking with
orthoses and crutches. This required correction of his feet and knees. His wrists
and hips were not a block to this. Indeed, with AMC, flexion is often the best
position for wrists and hands to work together.
• Skilled anaesthesia to cope with positioning and access to veins and airway
• Radical surgery. This boy required soft-tissue releases and talectomies for his
feet and soft tissue releases,5 including posterior capsulotomies along with
lower femoral extension osteotomies, for his knees
• Rehabilitation with a physiotherapist
• Prolonged splintage
In this case the goals of treatment were achieved (Figs 10.5a and b)
140 Chapter 10  Viva Answers

a b
Figure 10.5a and b

108C. What is the likely diagnosis and how would you manage the
case?
The diagnosis is a benign sternomastoid tumour. The aetiology is likely to be
traumatic. The condition is right-sided in 75% of cases and a higher proportion
of babies were breech deliveries, as in this case.
One in seven cases progress to a muscular torticollis. The remainder resolve
spontaneously. This was explained to the parents, no treatment was given and
the condition resolved spontaneously.6

109A. Describe the radiological features.


There is general osteopaenia. The upper and lower vertebrae are H-shaped,
due to step-like depression of the end-plates, with sparing of their peripheral
portions.
There is vertebral collapse due to avascular necrosis.

109B. What is the likely diagnosis?


In a patient of Afro-Caribbean descent it is likely this represents sickle cell
disease. Bone infarcts secondary to sickling of haemoglobin cause the
characteristic pattern of spinal collapse known as fish-tail vertebrae.

109C. What is the condition in general terms?


The general appearances are of a syndromic talipes equinovarus.
Children's orthopaedics 141

109D. What is the diagnosis?


The diagnosis is Freeman-Sheldon syndrome (distal arthrogryposis type 2A,
craniocarpotarsal dysplasia, “whistling face syndrome”).
The child shows the typical facial features of microstomia, naso-labial folds,
long philtrum, a chin dimple (typically H-shaped) and low-set ears. There is also
drooping of the lower eyelids.

109E. How would you treat his feet?


The feet are likely to resist treatment and even with extensive soft-tissue and
bony surgery, correction is difficult and long-term splintage essential. There
are special anaesthetic problems with airway and venous access and a risk of
malignant hyperpyrexia.

110A. Describe the original injury, the current radiological features


and how you would manage the case.
Figure 9.21a and b show Salter-Harris Type II injury of the lower femur, which,
with its location and mechanism, has a high risk of growth plate arrest.
Figure 9.21c shows a posterior growth plate arrest and corresponding angular
deformity. This is likely to progress and, along with the angulation, leave a limb
length discrepancy in the order of 5 cm to 6 cm.
MRI would be useful in defining the extent of the growth plate tether. However,
an attempt to resect any tether would be technically difficult and the results
unreliable
The best option would be to ablate the remaining growth plate and correct the
angulation and shortening using a circular frame.
If such facilities were not available or the child was unsuitable for the
method, the angulation could be corrected by supracondylar osteotomy and
epiphysiodesis of the contralateral knee.

110B. What is the condition, what are the differential diagnoses and,
of these, which is the most likely in this case?
The diagnosis is winged scapula.
The aetiology may be in bone (e.g. congenital malformation or subscapular
exostosis), joint (e.g. post-inflammatory or traumatic fusion), nerve (neuropraxia
of the long thoracic nerve) or muscle.
In this case, although the history suggested a nerve injury, the diagnosis proved
to be facio-scapular-humeral (F-S-H) dystrophy. With hindsight, there was
already early winging of the left scapula.
142 Chapter 10  Viva Answers

111A. How would you manage the case?


There must be a full history and examination to exclude other conditions,
sometimes hereditary, associated with polydactyly.
Confirm that child and family want the extra digit removed. In some cases or
societies the condition is accepted and even considered lucky.
In this case the most normal of the lesser toes in each foot is the outermost,
although all the toes are abnormal to some degree. It is important to explain
pre-operatively that the toes cannot be restored to normal.7
At operation, a wedge resection of the fifth toe and its metatarsal component
was undertaken. This allowed the outermost metatarsal to lie easily against the
fourth, to which it was sutured. Three weeks in a bandage helped to maintain
the position and allow healing.

111B. What is the condition?


The diagnosis is a syndromic type of arthrogryposis, namely multiple pterygium
(Escobar) syndrome.

111C. What are the clinical features?


As with arthrogryposis generally, the children are of normal intelligence and
there is amyoplasia, so they are weak (note the wheelchair).
Particular features of Escobar syndrome may include:
1. Multiple webs, crossing not only major joints but also involving the neck,
eyelids and perineum, where there may be genital abnormalities and
cryptorchism.
2. Joint contractures in hands and feet, where major foot deformities are
possible.
3. Decreased lung capacity.
4. Cleft palate.
5. Facial features such as ptosis, sloping eyelids, low-set ears, long filtrum and
retrognathia.
6. Scoliosis.
111D. What are the principles of management?
The management of the deformities is dominated by weakness and anaesthetic
considerations. There must be a full functional assessment and clearly defined
goals of treatment.
In the upper limbs early corrective splintage can be useful but surgery is only
indicated occasionally as it would not improve function.
In the lower limbs, operative correction of the deformities should be considered
in order to allow walking or at least standing for transfers. The operations can be
complex and require good anaesthetic support and prolonged splintage post-
operatively.
Children's orthopaedics 143

References
1. Almquist EE, Gordon LH, Blue AI. Congenital dislocation of the head of the radius. J Bone Joint
Surg [Am] 1969;51-A:1118-27.
2. Miller MD. Review of orthopaedics. Fifth ed. Philadelphia: Saunders Elsevier, 2008.
3. Canale ST, Beaty JH, eds. Campbell’s Operative Orthopaedics. 11th ed. Philadelphia: Mosby
Elsevier; 2007.
4. Flynn JC. Nonunion of slightly displaced fractures of the lateral humeral condyle in children: an
update. J Pediatr Orthop 1989;9:691-6.
5. Lourenco AF, Dias LS, Zoellick DM, Sodre H. Treatment of residual adduction deformity in
clubfoot: the double osteotomy. J Pediatr Orthop 2001;21:713-18.
6. McDonald D. Sternomastoid tumour and muscular torticollis. J Bone Joint Surg [Br] 1969;
50-B:432-43.
7. McCarthy GJ, Lindaman L, Stefan M. Pedal polydactyly: an overview with case report. J Foot Ankle
Surg 1995;34:577-82.
Chapter 11
Viva Questions:
Basic science
112. A. What is your thromboprophylaxis regime for a 72-year-old man who is
scheduled for a total hip replacement?
B. What is your thromboprophylaxis regime for a 72-year-old man who is
scheduled for a total knee replacement?
C. What is the evidence for previous two answers
D. Is there a role for routine chemical thromboprophylaxis in patients with ankle
fractures? If yes, what is your regime for the same?
E. What is the mechanism of action of aspirin, low-molecular-weight heparin
and warfarin?
F. Do you know of any oral anticoagulant that can be used as a
thromboprophylactic agent in joint replacement surgery? What is the
mechanism of action of this agent?
G. What are the contraindications to the use of chemical thromboprophylaxis in
joint replacement surgery?

113.  A. What are the types of bone graft that you are aware of?
B. What are the different types of synthetic grafts that you are aware of?
C. Compare and contrast the properties of synthetic bone grafts.
D. Explain the term creeping substitution.
E. Explain the process of collection and storage of donor femoral heads to be
used as bone graft in future.

114.  A. What do you understand by the term osteoporosis?


B. What is the WHO definition for osteoporosis?
C. What are the risk factors for the development of osteoporosis?
D. What is the pathophysiology of osteoporosis?
E. How would you investigate a patient with suspected osteoporosis?
F. What are the changes in peak bone mass with respect to age?
G. How do you classify osteoporosis?

115.  A. How would you differentiate osteoporosis from osteomalacia?


B. What advice would you give a patient in terms of prevention of osteoporosis?
C. Is having a national screening programme for osteoporosis worthwhile? Why?
146 Chapter 11  Viva Questions

D. How would you treat an established case of osteoporosis?


E. What are the complications/side-effects of therapy with bisphosphonates?

116.  A. What is the pathophysiology of Paget’s disease?


B. What is the aetiology of Paget’s disease?
C. What is the mode of presentation in patients with Paget’s disease?
D. What are the potential metabolic complications in patients with Paget’s
disease?
E. How would you investigate a patient with Paget’s disease?
F. What is the treatment of established Paget’s disease?

117.  A. What is the pathophysiology of rheumatoid arthritis?


B. What is the aetiology of rheumatoid arthritis?
C. What are the ARA criteria for the diagnosis of rheumatoid arthritis?
D. What are the potential extra-articular manifestations in patients with
rheumatoid arthritis?
E. How would you investigate a patient with rheumatoid arthritis?
F. What are the poor prognostic indicators for patients with rheumatoid
arthritis?

118.  A. W
 hat are the basic principles behind the use of a tourniquet in upper and
lower limb surgery?
B. How do you decide on inflation pressures in upper and lower limb surgery?
C. When would you not use a tourniquet?
D. What are the complications of the use of a tourniquet?
E. What do you understand by the term post-tourniquet syndrome? How is it
treated?

119.  Operating theatres


A. Whilst designing a new theatre complex, what are the different zones that one
has to bear in mind and why?
B. What are the sources of contamination in an operating theatre?
C. How is the risk of this contamination reduced?
D. What are the types of ventilation system that you are aware of and describe
each in detail?
E. What do you understand by the term "laminar air flow"?
F. What is the effect of laminar air flow on the risk of infection in joint
replacements?

120.  A. H
 ow would you assess that your laminar airflow operation theatre is suitable
for performing joint replacements?
B. What are the microbiological requirements for continuing to perform joint
replacements in your operation theatre?
Basic science 147

C. What antibiotics would you administer as prophylaxis prior to performing a


lower limb joint replacement and why?
D. What is the mechanism of action of flucloxacillin and vancomycin?
E. Do pre-operative prophylactic antibiotics reduce the risk of infection
following a joint replacement?
F. What is the evidence for your answer to question 5?

121.  A
 23-year-old man presents with acute onset of pain and stiffness in his
right elbow, in particular supination and pronation of his forearm. He had
a fracture of his radial neck fixed nine years ago and the metalwork was
subsequently removed (Figs 11.1a and b). There were other features, which
suggested acute sepsis on this presentation.
A. Describe the radiographs.
B. What is the differential diagnosis?
C. What other investigations would you arrange?
D. How would you manage this patient?
E. How would you classify osteomyelitis?

a b

Figure 11.1a and b

F. Describe the gram-staining technique for identification of bacteria and how


do you differentiate between gram-positive and gram-negative bacteria on
gram staining?

122.  A
 64-year-old male who has had a total knee replacement a year ago presents
with a history of pain and swelling with some discharge from his knee (Fig. 11.2).
A. Describe the clinical photograph.
B. What is the differential diagnosis?
C. What investigations would you like to arrange?
D. How would you manage this patient?
148 Chapter 11  Viva Questions

Figure 11.2

E. What do you understand by the term biofilm?


F. Describe the constituents of a biofilm and the stages in its formation.
G. Why are antibiotics ineffective in the presence of a biofilm?

123.  A. Describe the photograph (Fig. 11.3).


B. What do you think is the mechanism of failure in this situation?
C. How are ceramics manufactured?
D. Discuss the biomechanical properties of ceramics.
E. What is the mode of failure in ceramics with respect to them being used as a
bearing surface?
F. Compare ceramic-on-polyethylene with ceramic-on-ceramic as a bearing couple.

Figure 11.3
Chapter 12
Viva Answers:
Basic science
112A. What is your thromboprophylaxis regime for a 72-year-old man
who is scheduled for a total hip replacement?
Combine mechanical prophylaxis using anti-embolism stockings with
pharmacological prophylaxis using low molecular weight heparin. This regime
should continue for 4 weeks after surgery. In addition, use an intra-operative
intermittent calf compression device on the non-operated side. This approach
should be tailored to each patient and all patients should undergo risk
assessment as part of a pre-operative work-up. This should identify patients who
require other prophylaxis.
Be aware of NICE guidelines which outline other acceptable agents, including
oral anticoagulants such as riviroxiban.

112B. What is your thromboprophylaxis regime for a 72-year-old man


who is scheduled for a total knee replacement?
For elective knee replacement surgery the same protocol as above should be
followed but need only be contiuned for 10–14 days.

112C. What is the evidence for your previous two answers.


NICE guidelines – as above.1

112D. Is there a role for routine chemical thromboprophylaxis in patients


with ankle fractures? If yes, what is your regime for the same?
Consider offering pharmacological VTE prophylaxis to patients with lower limb
plaster casts after evaluating the risks and benefits based on clinical discussion
with the patient. Offer LMWH until lower limb plaster cast removal.
Following ankle fracture surgery LMWH is generally given until the plaster is
removed. For ankle fractures treated non-operatively this is not normally the case.

112E. What is the mechanism of action of aspirin, low-molecular-


weight heparin and warfarin?
Aspirin is non-selective and irreversible COX (cyclo-oxygenase) inhibitor.
Normally COX produces prostaglandins, most of which are pro-inflammatory,
150 Chapter 12  Viva Answers

and thromboxanes, which promote clotting. Low-dose, long-term aspirin use


irreversibly blocks the formation of thromboxane A2 in platelets, producing an
inhibitory effect on platelet aggregation.
LMWHs inhibit the coagulation process through binding to antithrombin via
a pentasaccharide sequence. This binding leads to a conformational change
of antithrombin which accelerates its inhibition of thrombin (factor IIa) and
activated factor X (factor Xa). Once dissociated, the LMWH is free to bind to
another antithrombin molecule and subsequently inhibit more thrombin.
Warfarin inhibits the vitamin K-dependent synthesis of biologically active forms
of the calcium-dependent clotting factors II, VII, IX and X.

112F. Do you know of any oral anticoagulant that can be used as a


thromboprophylactic agent in joint replacement surgery? What
is the mechanism of action of this agent?
Dabigatran acts by directly inhibiting the enzyme thrombin, the enzyme which
converts fibrinogen into insoluble strands of fibrin (part of the clotting cascade).

112G. What are the contraindications to the use of chemical


thromboprophylaxis in joint replacement surgery?
Contraindications to chemical thrmoboprophylaxis include:
• Active bleeding
• Acquired bleeding disorders (such as acute liver failure)
• Concurrent use of anticoagulants known to increase the risk of bleeding
(such as warfarin with international normalised ratio [INR] higher than 2)
• Lumbar puncture/epidural/spinal anaesthesia expected within the next
12 hours
• Lumbar puncture/epidural/spinal anaesthesia within the previous 4 hours
• Acute stroke
• Thrombocytopenia (platelets less than 75 x 109/l)
• Uncontrolled systolic hypertension (230/120 mmHg or higher)
• Untreated inherited bleeding disorders (such as haemophilia and von
Willebrand’s disease)
113A. What are the types of bone graft that you are aware of?
Bone grafts may be autografts or allografts and may consist of cancellous or
cortical bone.
Autografts involve utilising bone obtained from the same individual receiving
the graft. Bone can be harvested from a myriad of sites. An additional surgical
site is often required, in effect adding another potential location for post-
operative pain and complications. Autografts are osteoinductive (able to recruit
host mesenchymal cells to differentiate into osteoblasts to make new bone),
osteoconductive (form a scaffold enabling blood vessel ingrowth and new bone
formation) and osteogenic (the ability of cellular elements within the graft to make
new bone).
Basic science 151

Allograft is harvested from an individual other than the one receiving the
graft. Allograft bone is taken from cadavers. There are three types of bone
allograft available, fresh or fresh-frozen bone, freeze-dried bone allograft and
demineralized freeze-dried bone allograft. Allografts are osteoconductive and
may be osteoinductive.
Cancellous bone graft is used for osteogenesis. It is commonly used for
treating non-unions and cavitary defects as it is quickly remodelled and
incorporated.
Cortical bone graft is used for structural support and is slowly incorporated.
Heterogenous bone from another species was trialled but with poor results thus
resulting in its withdrawal.

113B. What are the different types of synthetic grafts that you are
aware of?
Synthetic bone grafts or graft substitutes are commercially available. The main
constituents of these are either calcium triphosphate, hydroxyapatite, calcium
carbonate and calcium sulphate. These often contain bone morphogenic
proteins (BMPs) and demineralised bone matrix. The composition varies between
commercial products.

113C. Compare and contrast the properties of synthetic bone grafts.


Osteoconductive grafts include calcium sulphate, hydroxyapatite, ceramics,
calcium phosphate, collagen and other polymers. Osteoinductive grafts include
demineralised bone matrix, bone morphogenic proteins and growth factors.
Composite grafts may combine these properties.

113D. Explain the term creeping substitution.


Creeping substitution is the process by which cancellous bone graft is
incorporated. It refers to osteoblasts laying down new bone over the old grafted
bone, which is subsequently resorbed.

113E. Explain the process of collection and storage of donor femoral


heads to be used as bone graft in future.
A femoral head from a living donor undergoing a total hip replacement is
packaged and frozen within 24 hours of donation. Chips from the bone taken
by the retrieving surgeon are placed into aerobic and anaerobic broths for
culture. The product is only used if free from microbial growth. The sample is
quarantined for 180 days to retest the donor for serological markers or tested
at time of donation by PCR technology for Hepatitis and HIV in addition to
the routine serology. It is stored at less than -40°C. This product is stored in
two sterile plastic containers within an outer bag. Delivery is in a disposable
transport box containing dry ice (solid carbon dioxide) validated to keep the
graft frozen until the date and time provided on the box. It is delivered direct
to the point of use e.g. theatre.2,3
152 Chapter 12  Viva Answers

114A. What do you understand by the term osteoporosis?


Osteoporosis means there reduced bone mineral density (BMD) but there is no
problem with bone mineralisation.

114B. What is the WHO definition for osteoporosis?


Osteoporosis is defined by the World Health Organisation (WHO) as a bone
mineral density that is 2.5 standard deviations or more below the mean peak
bone mass of an average young, healthy adult as measured by the DEXA scan
(dual energy X-ray absorptiometry).

114C. What are the risk factors for the development of osteoporosis?
Risk factors can be non-modifiable and modifiable:
Non-modifiable risk factors:
• Age - BMD decreases, and consequently the risk of osteoporosis increases with
age.
• Gender - Women are at greater risk of osteoporosis as they have smaller bones
and hence lower total bone mass. Additionally, women lose bone more quickly
following the menopause, and typically live.
• Ethnicity - Afro-Caribbean women have a higher BMD than white women at all
ages due to a higher peak bone mass and slower rate of loss.
• Reproduction factors - A late menopause or short time from menopause
to BMD measurement are associated with higher BMD. BMD decreases
most rapidly in the early postmenopausal years. Current use of oestrogen
replacement therapy is associated with a higher BMD.
• Family history of osteoporosis
Modifiable risk factors:
• Weight - Higher risk with low weight.
• Smoking
• Alcohol
• Exercise
• Diet
• Medications - Steroids, etc.
114D. What is the pathophysiology of osteoporosis?
Bone resorption is always followed by bone formation, a phenomenon
referred to as coupling. In osteoporosis, this coupling mechanism is thought
to be unable to keep up with the constant microtrauma to trabecular bone.
Osteoblasts not only secrete and mineralise osteoid but also appear to control
the bone resorption carried out by osteoclasts. Osteoclasts require weeks
to resorb bone, whereas osteoblasts need months to produce new bone.
Therefore, any process that increases the rate of bone remodeling results in
net bone loss over time.4
Basic science 153

114E. How would you investigate a patient with suspected


osteoporosis?
Initial work up would include a full history including family history,
gynaecological history if appropriate, medications taken, history of previous
fractures. Full examination would and blood tests to rule out other bone
pathologies would be undertaken (calcium, phosphate, alkaline phosphatase
levels) and possible urinary tests. Once other pathologies had been excluded a
DEXA scan would confirm the diagnosis of osteoporosis.

114F. What are the changes in peak bone mass with respect to age?
BMD increases until 25 then remains high until 45 to 50 years of age. BMD
then reduces more rapidly after 50 in women (due to menopause) and steadily
decrease in men.

114G. How do you classify osteoporosis?


The disease may be classified as primary type 1, primary type 2, or secondary.
The form of osteoporosis most common in women after menopause is
referred to as primary type 1 or postmenopausal osteoporosis. Primary type
2 osteoporosis or senile osteoporosis occurs after age 75 and is seen in both
females and males at a ratio of 2:1. Finally, secondary osteoporosis may arise at
any age and affect men and women equally. This form of osteoporosis results
from chronic predisposing medical problems or disease, or prolonged use of
medications such as steroids.

115A. How would you differentiate osteoporosis from osteomalacia?


Both osteoporosis and osteomalacia may present with bone fractures. Typically,
osteoporosis is painless and insidious until a fracture develops. It is commoner
with advancing age. Characteristically, osteomalacia is a painful bone disorder
at onset, which can present at any age.
Osteomalacia patients may report a history of renal failure, anticonvulsant use,
or malabsorption. Osteoporosis typically presents with a normal serum calcium,
phosphorus, alkaline phosphatase, vitamin D, and PTH. In contrast, osteomalacia
is characterised by hypophosphataemia, hypocalcaemia, increased alkaline
phosphatase levels, low levels of vitamin D metabolites, and secondary
hyperparathyroidism. Urinary calcium levels may be normal in osteoporosis
but are often low in osteomalacia. Both conditions appear as low bone mass on
radiographs and DEXA scan. However, specific radiological findings unique to
osteomalacia include Looser pseudofractures. On x-ray, the coarseness of the
trabeculae in osteomalacia may differentiate the two diagnoses.5

115B. What advice would you give a patient in terms of prevention of


osteoporosis?
In both osteoporosis and osteomalacia bone mass may be decreased, but in
osteoporosis mineralisation is normal, whereas it is deficient in osteomalacia.
154 Chapter 12  Viva Answers

Prevention of osteoporosis includes reduction of risk factors. Important


messages would be to avoid smoking, excess alcohol and drug abuse. A healthy
balanced diet and encouraging some load-bearing exercise is also important.
Prevention of falls in at-risk patients may also minimise osteoporotic fragility
fractures.

115C. Is having a national screening programme for osteoporosis


worthwhile? Why?
Osteoporosis is an important condition that causes more than 200 000 fractures
each year at a cost to the NHS of more than £940m. It would therefore seem
that a national screening programme would be of benefit. However, according
to the Wilson-Jugner criteria,6 screening the entire population would not be
worthwhile. This is because there is not really an early stage of osteoporosis
that would require treatment to prevent a late stage. The use of bone density
assessment in selected individual patients is however important in reducing
the prevalence of osteoporosis, prevention and therefore reduction of fragility
fractures. Those who would benefit from screening would be women with a
premature menopause; people on steroids for an extended period; women who
have suffered hip fractures and elderly people with a stoop or loss of height.

115D. How would you treat an established case of osteoporosis?


Treating osteoporosis includes advising patients to stop smoking, excess
alcohol and to ensure that they have a healthy balanced diet with moderate
load-bearing exercise. Options for preventing bone loss include calcium and
Vitamin D supplementation, oestrogen therapy (HRT), bisphosphonates
(inhibits osteoclasts), calcitonin and selective oestrogen receptor modulators
(e.g. raloxifene). Stimulation of bone formation could include sodium
fluoride (stimulate osteoblasts), recombinant PTH and strontium. The specific
combination of therapies would vary from patient to patient.

115E. What are the complications/side-effects of therapy with


bisphosphonates?
Oral bisphosphonates are associated with gastric irritation and oesophageal
ulceration. It is recommended that they are taken after food and that
the patient should remain upright for 30 to 60 minutes after taking the
medication. Osteonecrosis of the jaw is a complication associated with
intravenous bisphosphonates and is commoner in patients having dental
surgery involving the jaw. There have been reports of patients having severe
muscle, joint, and/or bone pain after taking bisphosphonate medications.
This complication may arise days, months, or even years after starting
bisphosphonate therapy. Atypical femoral fractures have been reported in
patients receiving long-term bisphosphonates and any significant thigh pain
should be investigated in these patients. In the USA, the FDA has commented
on an association between a higher prevalence of atrial fibrillation in patients
taking bisphosphonates.
Basic science 155

116A. What is the pathophysiology of Paget’s disease?


The pathophysiology of Paget’s includes increased osteoclast size and number.
This results in increased bone resorption with haphazard osteoblastic bone
formation. Bone is enlarged, hypervascular, deformed and biomechanically
weak.

116B. What is the aetiology of Paget’s disease?


Aetiology is largely unknown, but one theory includes a viral origin as Pagetic
osteoclasts have been shown to contain mRNA from paramyxoviruses and
canine distemper virus.

116C. What is the mode of presentation in patients’ with Paget’s


disease?
Paget’s disease is usually diagnosed in the fifth decade of life and manifests with
pain. They may progress to degenerative joint disease, long bone deformity,
cranial nerve compression, high output cardiac failure or a pathological fracture.

116D. What are the potential metabolic complications in patients with


Paget’s disease?
The potential metabolic complications include a raised alkaline phosphatase,
raised serum acid phosphatase and raise urinary hydroxyproline.

116E. How would you investigate a patient with Paget’s disease?


Patients with suspected Paget’s are investigated with a full history and
examination, radiographs and possibly an isotope bone scan to determine the
extent and activity of the Paget’s disease process. Blood tests including a full
bone and liver profile should be performed. Urinary tests for hydroxyproline
should also be undertaken.

116F. What is the treatment of established Paget’s disease?


Most patients with Paget’s disease require no treatment. Medical management
is directed at lowering osteoclast activity and numbers. This can be done with
calcitonin or bisphosphonates. Pathological fractures of the lower limbs can
be extremely hypervascular and targeting of osteoclast function can reduce
peri-operative blood loss. Otherwise treatment is directed with dealing with the
subsequent complications associated with Paget’s disease.

117A. What is the pathophysiology of rheumatoid arthritis?


Rheumatoid arthritis (RA) is an auto-immune systemic inflammatory disorder.
There is a synovial inflammatory response (pannus) around the joint capsule.
This inflammatory tissue leads to erosive arthritic changes within joints due to
destruction of the articular cartilage.
156 Chapter 12  Viva Answers

117B. What is the aetiology of rheumatoid arthritis?


The aetiology of disease is obscure; auto-immunity has a large role in
development of RA. The disease may arise as a result of:
• Age: The average age of onset of disease are about 40, but it may occur at all
ages. It is less common before puberty.
• Gender: Females are affected three times more than males.
• Genetic: Individuals with HLA-DR1 or HLA-DR4 serotypes are at risk of
developing RA. Concordance of RA in identical twins is approximately 15%
117C. What are the ARA criteria for the diagnosis of rheumatoid
arthritis?
Revised ARA Criteria for the Classification of Rheumatoid Arthritis (RA).7
For classification purposes, a patient is said to have RA if he or she has satisfied
at least four of the following seven criteria. Criteria 1 through 4 must have
been present for at least six weeks. Patients with two clinical diagnoses are not
excluded. Designation as classic, definite, or probable RA is not to be made.
1. Morning stiffness: Morning stiffness in and around the joints, lasting at least
one hour before maximal improvement.
2. Arthritis of three or more joint areas: At least three joint areas simultaneously
have had soft tissue swelling or fluid (not bony overgrowth alone) observed
by a physician; the 14 possible joint areas are right or left proximal
interphalangeal (PIP) joints, metacarpophalangeal (MCP) joints, wrist, elbow,
knee, ankle, and metatarsophalangeal (MPT) joints.
3. Arthritis of hand joints: At least one area swollen (as defined above) in a wrist,
MCP or PIP joint.
4. Symmetric arthritis: Simultaneous involvement of the same joint areas (see
2 above) on both sides of the body (bilateral involvement of PIPJs, MCPJs, or
MTPJs is acceptable without absolute symmetry).
5. Rheumatoid nodules: Subcutaneous nodules, over bony prominences, or
extensor surfaces, or in juxta-articular regions, observed by a physician.
6. Serum rheumatoid factor: Demonstration of abnormal amounts of serum
rheumatoid factor by any method for which the result has been positive in <
5% of normal control subjects.
7. Radiographic changes: Radiographic changes typical of RA on posteroanterior
hand and wrist radiographs, which must include erosions or unequivocal
bony decalcification localized to or most marked adjacent to the involved
joints (osteoarthritis changes alone do not qualify).
117D. What are the potential extra-articular manifestations in patients
with rheumatoid arthritis?
• Cardiovascular – atherosclerosis, pericarditis, IHD
• Pulmonary – pulmonary fibrosis, pleural effusions
• Abdominal – splenomegaly
• Skin – vasculitis, Raynauds syndrome, nodules, Sjögrens syndrome
• Eye – scleritis, episcleritis
Basic science 157

117E. How would you investigate a patient with rheumatoid arthritis?


A blood test for rheumatoid factor (RF) is undertaken. A negative RF does not
rule out RA. Other blood tests are usually done to allow for other causes of
arthritis, such as lupus erythematosus. The ESR, CRP, full blood count, renal
function, liver enzymes and other immunological tests (e.g. antinuclear
antibody/ANA) are all performed.
If there is small joint disease then radiographs of the hands or feet may be
taken. There may be no radiographic changes early on. Radiographs of other
symptomatic joints would be obtained as well.

117F. What are the poor prognostic indicators for patients with
rheumatoid arthritis?
Poor prognostic factors include:
• Persistent synovitis
• Early erosive disease
• Extra-articular findings
• Positive rheumatoid factor
• Family history of RA
• Poor functional status
• Socioeconomic factors
• Elevated ESR/CRP
118A. What are the basic principles behind the use of a tourniquet in
upper and lower limb surgery?
Tourniquets help provide a bloodless field during surgery, by eliminating
arterial flow distal to the tourniquet. They can be non-pneumatic (used for
digits) or pneumatic which can be automatic (operate from an air line or
electric pump) or non-automatic (hand-operated pump). They should be well
padded, of appropriate size and shielded from the surgical prep, which could
lead to a burn.

118B. How do you decide on inflation pressures in upper and lower


limb surgery?
There is no absolute value, but a combination of patient age, soft-tissue
condition, circumference of limb and comorbid medical conditions should
be used to guide inflation pressures. As a rule of thumb in the upper limb, a
pressure of 50 mmHg more than systolic is used and in lower limb surgery a
value double the systolic is used.

118C. When would you not use a tourniquet?


A tourniquet is contraindicated in severe crushing injuries or poor skin
quality, sickle cell disease and in significant peripheral vascular disease. Also
the use of tourniquets during the treatment of infection, open fractures and
intramedulllary nailing is often avoided.
158 Chapter 12  Viva Answers

118D. What are the complications of the use of a tourniquet?


Complications can be divided into local and systemic:
Local
• Compression neurapraxia
• Bone and soft-tissue necrosis
• Direct vascular injury
• Post-operative swelling/stiffness
• Wound haematoma/infection
Systemic
• Cardiorespiratory decompensation
• Increased CVP
• Deep-vein thrombosis
• Cerebral infarction
• Altered acid-base balance
118E. What do you understand by the term post-tourniquet syndrome?
How is it treated?
This is a tourniquet-induced skeletal muscle ischaemia. Oedema, stiffness,
pallor, weakness and numbness can be noted. It is treated with removal of the
tourniquet and supportive measures such as ensuring that the patient is well
hydrated and haemodynamically stable.

119A. While designing a new theatre complex, what are the different
zones that one has to bear in mind and why?
Operating theatre zones:
• An outer, or general access zone for patient reception area and general office.
• A clean, or limited access zone between the reception bay and theatre suite,
and dispersal areas, corridors and staff rest room.
• Restricted access zone, for those properly clothed personnel engaged in
operating theatre activities, including anaesthetic room, utility and “scrub up”
rooms.
• An aseptic operating zone – the operating theatre. This keeps the number
of people moving through the operating zone to a minimum, as the
bacteriological count is related to the number of persons and their movement.
This also allows the operating zone to be separate and to enable control of
ventilation, air filtration and temperature, humidity and light.
• Disposal zone.
119B. What are the sources of contamination in an operating theatre?
Sources of contamination:
1. Internal: from the patient themselves e.g. the skin of the patient and
bacteraemia.
2. External:
a. Airborne pathogens – airborne particles mainly from theatre personnel.
Basic science 159

b. Human – theatre personnel, other than airborne pathogens e.g. direct


contact or transfer via inanimate object.
c. Instruments/equipment/protheses – both disposable/single use and re-
usable.
d. Theatre fixtures and fittings.
119C. How is the risk of this contamination reduced?
In order to reduce the risk of contamination British Orthopaedic Association
guidelines recommend:8
1. The level of sterile precautions required to perform orthopaedic surgery safely
are higher than those for surgery involving the bowel, infected body cavities,
contaminated wounds and other soft-tissue surgery.
2. The following measures must be enforced when material is implanted in
bone, major joints are opened or bone is exposed.
3. The use of clean air theatres, installed, maintained and checked according to
NHS standards is considered essential for orthopaedic surgery.
4. All staff in the operating theatre suite, including the anaesthetic room and
corridor, must adhere to existing high standards of theatre discipline and
follow established procedures which include:
a. All hair to be kept covered at all times.
b. Masks to be worn at all times within the operating theatre and lay-up room.
c. Street clothes and clothes worn outside the operating theatre suite,
including shoes, must not be worn within the theatre suite.
d. Staff may only enter or leave the operating theatre through clearly
identified doors so that air within the operating theatre is not disturbed
needlessly.
e. The number of people within the operating theatre must be kept to the
minimum required to function safely.
f. Traffic from dirty areas and within the lay-up room must be rigidly
controlled.
g. Drapes and gowns must be made of impervious material. Thin cotton
drapes and gowns have no place in orthopaedic surgery.
In addition to these guidelines, the use of ultraclean air and prophylactic
antibiotics greatly reduces infection rates. Lidwell et al9 in the MRC trial reported
the lowest incidence of deep infection (0.06%) was achieved using ultra-clean
air, body exhaust systems and prophylactic antibiotics.
Further measures to reduce contamination include:10
• Screening and eradication of MRSA
• Day of surgery admission onto specific wards
• Body exhaust systems
• Double-gloving
• Effective surgeon scrubbing and use of skin antiseptics e.g. chlorhexidine
gluconate 4%
• Patient skin preparation with antiseptics
• If the operative site is to be shaved, this should be performed as close to the
operation as possible
160 Chapter 12  Viva Answers

• Equipment cleaning, disinfection and sterilisation


• Surgical technique, e.g. shorter operative time
119D. What are the types of ventilation system that you are aware of?
Describe each in detail.
Types of ventilation system:
Source of air for ventilation; air is usually taken in at the roof level of the theatre
suite. It is drawn by a series of fans through filters capable of removing bacteria
carrying particles. It is also humidified and warmed or cooled. High-efficiency
particulate air (HEPA) filters are employed. These are capable of filtering particles
of 0.5 microns in size with 99.97% efficiency.
Generally operating theatres are equipped with positive pressure (plenum)
ventilation systems. In this system the pressure inside the theatre is greater than
outside. Clean air is fed via wall or ceiling diffusers and let out of vents placed
just above floor level. Air also passes out around doors and other openings.
The opening of doors and the movement of personnel make this system less
efficient. Standard positive pressure ventilated operating theatres deliver
around 15 to 25 air changes per hour.
Laminar airflow: see the next answer.

119E. What do you understand by the term “laminar air flow”?


Laminar airflow: this involves the entire body of air within a designated space
moving with uniform velocity in a single direction along parallel flow lines.
There are three main types of theatre airflow:
• Horizontal laminar flow
• Vertical laminar flow
• Ex-flow or exponential flow (Howorth enclosures)
Theatres are usually designed with a vertical downward airflow concept. The
flow of air is around 0.3 m/s.
Horizontal laminar flow; HEPA filters form a wall, or part of a wall. The
positioning of potential objects to the laminar flow is important in order to
avoid obstructions (e.g. theatre personnel, image intensifiers).
Vertical laminar flow; air is passed through HEPA filters in the ceiling and
directed downwards towards the operative field in a vertical direction. The area
of HEPA filters on the ceiling is enclosed by panels extending towards the floor
(should extend to within 2 m of the floor). Objects such as theatre lights form
obstacles and create turbulence. Also, personnel standing under the edge of
the enclosure deflect the vertical airflow inwards (peripheral entrainment), also
deflecting contaminated air inwards towards the operative site.
Ex-flow or exponential flow (Howorth enclosures); this describes the flow of
clean air downwards and outwards in the shape of an inverted trumpet. This
eliminates the problem of peripheral entrainment. This system is in theory more
efficient than laminar flow, and requires fewer changes of air per hour.
Basic science 161

119F. What is the effect of laminar air flow on the risk of infection in
joint replacements?
In the MRC trial, Lidwell et al8 found that ultra-clean air reduced the risk of deep
joint sepsis in arthroplasty by a factor of 2.6 compared with controls. When all
groups in the trial were considered together the analysis showed deep sepsis
after 63 out of 4133 operations in the control group (1.5%) and after 23 out of
3922 operations in the ultraclean-air groups (0.6%) (ratio 2.6, 95% confidence
limits 1.6-4.2; p < 0.001).
Fitzgerald et al11 found the incidence of deep sepsis after 5,865 total hip
arthroplasties performed in four centres varied from 0.5% to 2.3%.Procedures
performed in a conventional operating room were associated with the highest
incidence of deep sepsis (1.3%). The use of a vertical, unidirectional airflow
system with a helmet aspirator suite was associated with the lowest incidence of
deep sepsis (0.6%).

120A. How would you assess that your laminar airflow operation
theatre is suitable for performing joint replacements?
I would ensure that the operating theatre complies with the Health Technical
Memorandum 2025 (HTM)12 with respect to ventilation in healthcare premises.

120B. What are the microbiological requirements for continuing to


perform joint replacements in your operation theatre?
When commissioning an ultraclean theatre by microbiological sampling 13
sampling points reproduced one at each corner of the unidirectional airflow
zone perimeter, halfway along each side of the perimeter, one at each corner of
the inner zone and one in the centre. The air should contain less than 0.5 CFU/
m3, or one colony for every 2 m3 sampled (HTM 2025).12

120C. What antibiotics would you administer as prophylaxis prior to


performing a lower limb joint replacement and why?
I would administer third-generation cephalosporins intravenously as they are
effective against staphylococcal and uropathogens. For revision cases, specific
antibiotics may been given in cases of confirmed infection, vancomycin and
teicoplanin are often used in cases of presumed aseptic revision surgery.13

120D. What is the mechanism of action of flucloxacillin and


vancomycin?
Like other β-lactam antibiotics, flucloxacillin acts by inhibiting the synthesis of
bacterial cell walls. It inhibits cross-linkage between the linear peptidoglycan
polymer chains that make up a major component of the cell wall of Gram-positive
bacteria.
Vancomycin acts by inhibiting proper cell wall synthesis in Gram-positive
bacteria. Due to the different mechanism by which Gram-negative bacteria
162 Chapter 12  Viva Answers

produce their cell walls and the various factors related to entering the outer
membrane of Gram-negative organisms, vancomycin is not active against Gram-
negative bacteria (except some non-gonococcal species of Neisseria).

120E. Do pre-operative prophylactic antibiotics reduce the risk of


infection following a joint replacement?
Yes.

120F. What is the evidence for your answer?


Fogelberg et al,14 Pavel et al,15 Boxma et al16 and Gillespie and Walenkamp17
have all demonstrated that prophylactic antibiotics reduce the incidence of
infection after orthopaedic surgery in patients without known infection and
their use is considered routine for primary total joint arthroplasty.

121A. Describe the radiographs


There is sclerosis and lysis in the proximal radius and periosteal scalloping,
consistent with osteomyelitis.

121B. What is the differential diagnosis?


There is a broad differential for these lytic lesions. Osteomyelitis, tumour and
avascular necrosis must all be considered.

121C. What other investigations would you arrange?


Blood tests including WCC, CRP, ESR and blood cultures. MRI scan for bone
marrow and soft-tissue changes. Bone scan including WCC labelled scan would
be arranged as well.

121D. How would you manage this patient?


History and examination and appropriate antibiotics if blood cultures yield an
organism. Aspiration of the bone may be attempted in the area of maximal
tenderness and swelling. The sample is sent to the laboratory for Gram stain,
culture and sensitivities. Surgery and antibiotics are complementary and
surgery may be required in cases refractory to antibiotic treatment. Sequestered
abscesses need surgical drainage.

121E. How would you classify osteomyelitis?


Chronic osteomyelitis classified by Cierny and Mader:18
• Medullary (endosteal disease).
• Superficial (cortical surface infected due to coverage defect).
• Localised (cortical sequestrum that can be excised without compromising
stability).
• Diffuse (Features of I, II and III with instability before or after debridement).
Basic science 163

This is further classified by assessing the physiologic status of the host:


• Class A denotes a normal host.
• Class B denotes a host with systemic compromise, local compromise, or
both.
• Class C denotes a host for whom the morbidity of treatment is worse than that
imposed by the disease itself.
121F. Describe the gram-staining technique for identification of
bacteria and how do you differentiate between gram-positive
and gram-negative bacteria on gram staining?
Gram-positive bacteria have a thick mesh-like cell wall made of peptidoglycan
(50% to 90% of cell envelope), which are stained purple by crystal violet,
whereas Gram-negative bacteria have a thinner layer (10% of cell envelope),
which are stained pink by the counter-stain. There are four basic steps of the
Gram stain:
1. Applying a primary stain (crystal violet) to a heat-fixed smear of a bacterial
culture. Heat fixing kills some bacteria but is mostly used to affix the bacteria
to the slide so that they don’t rinse out during the staining procedure.
2. The addition of a mordant, which binds to crystal violet and traps it in the cell
(Gram’s iodine).
3. Rapid decolorisation with alcohol.
4. Counterstaining with safranin. Carbol fuchsin is sometimes substituted for
safranin since it will stain the anaerobic bacteria more intensely but it is much
less commonly employed as a counterstain.
Crystal violet dissociates in aqueous solutions into CV+ and chloride (Cl−) ions.
These ions penetrate through the cell wall and cell membrane of both Gram-
positive and Gram-negative cells. The CV+ ion interacts with negatively charged
components of bacterial cells and stains the cells purple.

122A. Describe the clinical photograph.


There is a sinus in the central portion of midline scar over the knee. There
appears to be a significant joint effusion.

122B. What is the differential diagnosis?


Infection, either superficial or deep. Chronic discharging sinus.

122C. What investigations would you like to arrange?


Blood tests – WCC, CRP and ESR. Microbiological assessment of discharged fluid.
Plain radiographs to assess components for loosening.

122D. How would you manage this patient?


I would have a high clinical suspicion for infection. I would like to aspirate fluid
and undertake a synovial biopsy from the knee to confirm the suspicion. This
164 Chapter 12  Viva Answers

should be done under sterile conditions in the operating theatre. Combined


aspiration and biopsy confirms infection in 90% of cases. Ideal treatment then
involves a two-stage surgical procedure with extensive debridement and
washout, followed by antibiotics. An articulating/non-articulating cement
spacer (with antibiotics) is used prior to implantation of the revision prosthesis.
The timing of this is guided by inflammatory markers. The use of a single-stage
revision is gaining popularity has been advocated in certain patients where
the causative organism is known, no sinuses are present, the patient is not
immunocompromised, and there is no radiological evidence of component
loosening or osteitis.19

122E. What do you understand by the term biofilm?


Biofilm is a complex aggregation of microorganisms in which cells adhere to
each other on a solid substrate.

122F. Describe the constituents of a biofilm and the stages in its


formation.
Constituents of biofilm include the complex association of microorganisms
and a matrix of extracellular polymeric substance (EPS) which is produced by
the microorganisms themselves. This EPS is a polymeric conglomeration of
extracellular DNA, proteins and polysaccharides. There are five stages in biofilm
formation;
1. Initial attachment
2. Irreversible attachment
3. Initial maturation
4. Further maturation
5. Dispersion
122G. Why are antibiotics ineffective in the presence of a biofilm?
Biofilm cannot be treated by antibiotics as the dense extracellular matrix and
outer layer of cells protect the interior microorganisms. This acts as a penetration
barrier and in some cases antibiotic resistance can be increased a thousandfold.20

123A. Describe the photograph.


The photograph shows the acetabular component from a total hip arthroplasty.
This consists of a ceramic liner inside a metal shell. The ceramic liner is damaged
and may be incompletely seated in the shell. There is a fracture of the rim
and the inner edge of the liner is worn away in parts, with some evidence of
scratching in other areas. There is also some evidence of damage to the rim of
the metal shell.

123B. What do you think is the mechanism of failure in this situation?


The most likely mechanism of failure is dislocation of the femoral head after
impingement of the femoral neck on the anterior lip of the liner. This had
Basic science 165

fractured the anterior and posterior edges of the liner and damaged the metal
shell posteriorly. This may have been contributed to by incorrect seating of the
liner in the shell. There may be some wear to the inner edge of the posterior part
of the liner, which represents posterior edge-loading. Posterior edge-loading in
itself may be seen in well-positioned acetabular components.22
Improved materials and hot isostatic pressing during manufacture has
reduced the grain size and increased the density of the ceramic, improving
its mechanical properties. Further improvement in has been made with the
introduction of alumina/zirconia composite materials.

123C. How are ceramics manufactured?


Ceramics are manufactured by mixing powdered ceramic and water into a slurry
(mixing) and pressing them into a prefabricated casts (forming). The casts are
removed before the ceramic is heated to a high temperature (sintered/firing)
and undergoes hot isostatic pressing. In this process, gas pressure is applied
isostatically at high temperatures to enhance sintering and produce dense
bodies. After materials are pre-sintered, and their density is increased to almost
95% of the theoretical density, they are placed in a pressure container equipped
with a furnace. Gas pressure is then applied isostatically at 1000 to 2000
atmospheres while being heated. Sintering and hot isostatic pressing increases
the density of the granular structure. The component shrinks by between 10%
and 20% during sintering. The ceramic then undergoes finishing which involves
grinding and polishing, which is generally performed using a diamond wheel.
The final stage is quality control, which involves stress testing the components
to eliminate flawed components.

123D. Discuss the biomechanical properties of ceramics.


Ceramics are compounds of metallic elements such as aluminium or zirconium,
bound ionically and/or covalently with non-metallic elements.
They are hard materials and therefore very resistant to wear and are scratch
resistant. They have a high elastic modulus (high stiffness/Young’s modulus).
They are similar in strength to cold-worked stainless steel. They have a high
compressive strength, so not deform under physiological loads. Their down
side is that they are very brittle, meaning that they display almost no plastic
deformation before failure. They are therefore too brittle to be used for the
production of femoral stems, but are used to produce bearing surfaces do to
their hardness.
Ceramics do not corrode.
Polishing gives ceramic components the lowest surface roughness of all implant
materials. The mean roughness of is only around 2 nanometers (0.002 μm). The
strong hydrogen bonds between the ceramic surfaces and synovial fluid give
ceramic materials excellent wetting properties that are far superior to those
of metal and polyethylene and ensure that an effective lubricant film forms.
Three states of lubrication are observed in total hip arthroplasty: fluid film
166 Chapter 12  Viva Answers

lubrication (A), mixed lubrication (B) and boundary lubrication (C). The smooth
and hydrophilic surfaces of ceramic components help to ensure that the wear-
reducing lubrication states A and B are achieved more often than with other
bearing couples. Ceramic-on-ceramic (COC) bearing surfaces therefore produce
low friction and low wear volumes. Linear wear rates are a fraction of metal-on-
polyethylene (MOP) bearings (0.025 µm/year).
The introduction of alumina/zirconia composite materials has improved
the mechanical properties. Zirconia has a monoclinic crystalline structure at
room temperature, but changes to a smaller volume tetragonal structure at
temperatures greater than about 1100°C. It is possible to maintain this structure
at room temperature within the alumina matrix by stabilising with yttria. In
the presence of a crack, the restraint on the crystalline structure of zirconia is
removed and it transforms back to the larger volume monoclinic structure that
generates compressive stress and retards extension of the crack.
Another mechanism to dissipate the energy of a crack in the contemporary
material is the addition of strontium oxide that forms long crystals (platelets)
of strontium aluminate in the matrix of the alumina. These deflect the crack
and increase the distance it must travel to progress, thus increasing the energy
required for it to propagate.

123E. What is the mode of failure in ceramics with respect to them


being used as a bearing surface?
In a review of ceramic-on-ceramic bearings in hip arthroplasty in JBJS (Br),21
looking at all studies of COC until May 2011, the following complications were
reported: asceptic loosening, dislocation, fracture, liner chipping on insertion,
liner canting and dissociation, edge-loading and squeaking. Overall, the most
common complications changed over time, with the evolution of ceramic
implants.
The complications reported with regard to the actual ceramic bearing surface
were fracture, liner chipping on insertion, liner canting and dissociation, edge-
loading and squeaking. Some of these are design failures or complications that
can be minimised by better engineering of the prosthesis (i.e. not directly related
to the properties of ceramics).
Ceramics are susceptible to edge-loading and abrasive wear, particularly if the
acetabular cup position is too open. Wear debris in ceramic implants are usually
small volume and produce little to no biological reaction. Furthermore, wear
debris are thought to be an effect of implant failure, not a cause (as compared
with metal-on-polyethylene (MOP) bearings).
Poor implant design or incorrect surgical technique can lead to implant
instability, which may result in wear and damage to the ceramic.
Ceramics are also susceptible to brittle fracture, leading to catastrophic failure,
although the improved properties of modern ceramics have made this an
extremely rare occurrence (between 0% and 1.6% in studies that took place
between 1999 and 2011).21
Basic science 167

123F. Compare ceramic-on-polyethylene with ceramic-on-ceramic as a


bearing couple.
Ceramic-on-polyethylene (COP) couples have the theoretical advantage of lower
wear rates compared with traditional metal-on-polyethylene (MOP) couples. In
theory this is due to the smoother, more wettable, scratch resistant properties of
ceramic. However, despite these tribological advantages, the COP bearing has
not consistently shown lower rates of osteolysis than MOP.
The liner wear from the combination of a 22.225 alumina ceramic head and cross-
linked UHMWPE is only 0.022 mm/year after bedding in. This is compared with 0.07
mm/year for standard metal-on-UHMWPE. Although there is a lower wear rate,
the reduction is small in comparison with the wear rate of COC couples (0.025 µm/
year).
The disadvantages of COC couples are that they are more expensive and they
have the very small, but present risk of fracture. Additionally, COC couples suffer
from squeaking.
In the 9th annual report of the National Joint Registry (NJR), 2012, for all-
cemented prostheses COP has a revision rate of 2% at eight years. In all-
uncemented prostheses COC has an eight year revision rate of 3.89%, compared
with 2.46% for COP. In hybrid prostheses, the eight year revision rate for COC
was 1.95%, compared with 2.26% for COP.

References
1. National Institute for Health and Clinical Excellence. NICE clinical guideline 92. Venous
thromboembolism: reducing the risk. London: NICE, 2010.
2. Harkness JW, Crockarell JR. Campbell’s operative orthopaedics. 11th ed. Philadelphia: Mosby,
2008.
3. NHS Blood and Tissue. Tissue Services: Femoral head. http://www.nhsbt.nhs.uk/tissueservices/
products/bone/femoralhead
4. Miller MD. Review of orthopaedics. Fifth ed. Philadelphia: Saunders Elsevier, 2008.
5. Bingham CT, Fitzpatrick LA. Noninvasive testing in the diagnosis of osteomalacia. Am J Med
1993;95:519-23.
6. Wilson JMG, Jungner G. World Health Organization. Principles and practice of screening for
disease, 1968. http://whqlibdoc.who.int/php/WHO_PHP_34.pdf (date last accessed1 November
2012).
7. No authors listed. American College of Rheumatology. The 2010 ACR-EULAR classification criteria
for rheumatoid arthritis, 2010. http://www.rheumatology.org/practice/clinical/classification/ra/
ra_2010.asp (date last accessed 5 November 2012).
8. No authors listed. British Orthopaedic Association. Recommendations on sterile procedures in
operating theatres, 1999.
9. Lidwell OM, Lowbury EJ, Whyte W, et al. Effect of ultraclean air in operating rooms on deep
sepsis in the joint after total hip or knee replacement: a randomised study. Br Med J (Clin Res Ed)
1982;285:10-14.
10. Hughes SP, Anderson FM. Infection in the operating room. J Bone Joint Surg [Br] 1999;81-B:754-5.
11. Fitzgerald RH Jr, Bechtol CO, Eftekhar N, Nelson JP. Reduction of deep sepsis after total hip
arthroplasty. Arch Surg 1979;114:803-4.
12. No authors listed. NHS Estates. Ventilation in healthcare premises: design considerations.
Health Technical Memorandum 2025. http://www.mintie.com/assets/img/education/NHS%20
Estates%20-%20HVAC.pdf (date last accessed 20 February 2013).
168 Chapter 12  Viva Answers

13. Mangram AJ, Horan TC, Pearson ML, Silver LC, Jarvis WR. Guideline for prevention of surgical
site infection, 1999. Centers for Disease Control and Prevention (CDC) Hospital Infection Control
Practices Advisory Committee. Am J Infect Control 1999;27:97-132.
14. Fogelberg EV, Zitzmann EK, Stinchfield FE. Prophylactic penicillin in orthopaedic surgery. J Bone
Joint Surg [Am] 1970;52-A:95-8.
15. Pavel A, Smith RL, Ballard A, Larsen IJ. Prophylactic antibiotics in clean orthopaedic surgery. J
Bone Joint Surg 1974;56-A:777-82.
16. Boxma H, Broekhuizen T, Patka P, Oosting H. Randomised controlled trial of single-dose
antibiotic prophylaxis in surgical treatment of closed fractures: the Dutch Trauma Trial. Lancet
1996;347:1133-7.
17. Gillespie WJ, Walenkamp G. Antibiotic prophylaxis for proximal femoral and other closed long
bone fractures. Cochrane Database Syst Rev 2001;1:CD000244.
18. Cierny G 3rd, Mader JT. Approach to adult osteomyelitis. Orthop Rev 1987;16:259-70
19. Vanhegan IS, Morgan-Jones R, Barrett DS, Haddad FS. Developing a strategy to treat established
infection in total knee replacement: a review of the latest evidence and clinical practice. J Bone
Joint Surg [Br] 2012;94:875-81.
20. Stewart PS, Costerton JW. Antibiotic resistance of bacteria in biofilms. Lancet 2001;358;135-8.
21. Jeffers JR, Walter WL. Ceramic-on-ceramic bearings in hip arthroplasty: state of the art and the
future. J Bone Joint Surg [Br] 2012;94-B:735-45.
22. van Arkel RJ, Modenese L, Phillips AT, Jeffers JR. Hip abduction can prevent posterior edge
loading of hip replacements. J Orthop Res 2013;31:1172 -9.
Index

Note: Page numbers in bold or italic refer to tables or figures, respectively.

A Calcaneovalgus deformity 137


Aα (A alpha) fibre 16 Camptodactyly, infantile type 22
Aβ (A beta) fibre 16 Cancellous bone graft 151
Abductor pollicis longus 24 Capitellum fracture 85
Achondroplasia 26 classification of 85
Acromioclavicular ligament 74 complications of treatment 85
Aδ (A delta) fibre 16 in females 85
Aγ (A gamma) fibre 16 Kocher’s lateral approach in 85
Allograft 151 Cement fracture, in Gruen zones 2 and 6 26
Alpha angle 17 Cephalosporins 161
Ankle arthrodesis 19 Ceramic-on-ceramic (COC) bearing surfaces 166
Ankle fracture 149 Ceramic-on-polyethylene (COP) couples 167
Anterior cruciate ligament (ACL) Ceramics 164–167
injury 76 biomechanical properties of 165–166
reconstruction 25 complications with 166
Anterior humeral line 84 manufacturing of 165
Antibiotics, pre-operative use of, joint replacement Chemical thromboprophylaxis, contraindications to
and 161–162 150
Arthrogryposis multiplex congenita (AMC) 22, 22, Clavicle fracture, midshaft 81
139, 140 deforming forces in 81
Aspirin 149–150 malunions in, problems with 82–83
Autografts 150 non-union in 82
Autotransfusion 26 Cobb angle 44
Avulsion fracture of tibial spine 76 Coccygectomy, for refractory coccygodynia 18
Axillary nerve 17 Comminuted fracture, of midshaft of right humerus
71
Complex dislocation, of MCPJ of index finger 114–115
B Compression test 19
Bearing surfaces, in artificial joints 19 Conoid ligament 74
Biceps tendon rupture 106–107 Constriction band syndrome, of lower leg 136
cause of 106 Coracoclavicular ligaments 74
treatment options for 106–107 Cortical bone graft 151
Biofilm 164 Cozen’s fracture, of proximal tibial metaphysis
antibiotics for 164 134–135
formation of 164 Creeping substitution 151
Bisphosphonates 73, 154 Cubital tunnel 99
side-effects of 154 Cubitus varus, supracondylar fracture and 134
Bone grafts 150–151
Bone mineral density (BMD) 152, 153
Bone morphogenic proteins (BMPs) 21, 151
D
Bone tumours 19 Dabigatran 150
Brachialis tenodesis 24 De Quervain’s stenosing tenosynovitis 24
Buford complex 21 Developmental dysplasia of the hip 136
Bunnell–Littler test 22 DEXA (dual energy X-ray absorptiometry) scan 43
Dexamethasone 54
Disc degeneration disease 26
C Distraction osteogenesis 20
Caffey’s disease 137–138 Dorsal intercalated segmental instability (DISI)
Calcaneo-cavus foot 135 deformity 101–103
170 Index

Doxycycline 24 H
Dupuytren’s disease 110 Hawkin’s sign 49
pain in 110–111 Heterotopic ossification (HO) 45
surgical complications 111 Brooker classification for 45
treatment for 111 conditions needed for 45
Woodruff classification system for 111 prevention of 45–46
High-efficiency particulate air (HEPA) filters 160
E Hindfoot equinovarus 137
Hip arthrodesis, and back pain 15
Elbow injury, in children 15
Hip dislocation, right-sided 135–136
Endobutton 107
Hip dysplasia 41–42
Endothelial derived growth factor 21
Crowe classification of 42
Escobar syndrome 141–142
Hip pain, and measurement of angles
Exostosis 137
acetabular inclination 41
Extensor carpi radialis longus (ECRL) 112
anterior centre edge angle of Lequesne 42
Extensor indicis proprius (EIP), transfer of 111–112
lateral centre-edge angle of Wiberg 41
Extensor pollicis brevis (EPB) 112
Sharp’s angle 41
Extensor pollicis longus (EPL), rupture of 111–112
Humerus, undisplaced spiral fracture of 133, 133
Extracellular polymeric substance (EPS) 164
Hyaluronic acid 23–24
Hyperbaric oxygen therapy 25
F Hypophosphataemic rickets 26
Facet joint ablation 18
Femoral fracture, peri-prosthetic 51 I
treatment and outcomes 52–53
Infantile cortical hyperostosis see Caffey’s disease
Vancouver classification of 51, 51–52
Insulin-like growth factor II 21
Femoral head
Intramedullary nails 72, 73
blood supply to 43
Isometric muscle contraction 21
from living donor 151
Femur, fracture of 42–43
Fibrolipomatous hamartoma (FLH) 113 J
Fibrous dysplasia, malignant transformation in 20 Joint effusion 163
Fish-tail vertebrae 140
Flexor digitorum proundus (FDP) avulsion 100
classification system for 101 K
diagnosis of 100–101 Kaplan’s cardinal line 16
ring finger and 101 Key pinch grip 20
treatment of 101 Klippel–Trenaunay–Weber syndrome 19
Flucloxacillin 161 Knee
Forefoot varus 137 amputation 79
Forest plots 23 and foot orthoses 134
Fracture healing 15–16 lateral compartment osteoarthritis of 49–51
Freeman-Sheldon syndrome 141 Kniest syndrome 20
Kocher–Langenbeck approach 28
Kruskal–Wallis test 27
G
Gauchers disease 26
Genu varum 136–137 L
Glenohumeral joint 71 Laminar airflow 160
Glucocorticoids, effects of, on bone mineral ex-flow/exponential 160
metabolism 16 horizontal 160
Gram-negative bacteria 163 infection risk in joint replacements and 161
Gram-positive bacteria 163 vertical 160
Gram-staining technique 163 Larsen’s syndrome 134, 134
Growth plate arrest 141–142 Lateral clavicle fractures 73–74
Guyon’s canal 99 displacement of 74
Index 171

risk of non-union in 74 sources of contamination in 158–159


Rockwood and Green classification of 73 ventilation system in 160
treatment of 74 zones of 158
Lateral collateral ligament (LCL) 50 Osteoarthritis 16, 23–24
Latissimus dorsi transfer, male gender and 17 glenohumeral 71
Leg length discrepancy 19, 46 knee 49–51
Leri -Weill dyschondrosteosis 26 Osteoconductive grafts 151
Ligament of Struthers 27 Osteoinductive grafts 151
Limb salvage index (LSI) 80, 81 Osteomalacia 17, 153
Limb-threatening injury, lower extremity 78 Osteomyelitis, chronic 162–163
limb salvage scoring systems 79–81 Osteonecrosis of jaw 154
management of 79 Osteopaenia 140
soft-tissue injuries with closed fractures 79 Osteoporosis 16, 17, 152
Lipohaemarthrosis 76 bisphosphonates in 154
Low molecular weight heparin (LMWH) 149, 150 classification of 153
Lumbar facet joint injections 18 investigations in 153
Lumbosacral transitional vertebra (LSTV) 47–48 national screening programme 154
and osteomalacia 153
pathophysiology of 152
M prevention of 153–154
Macrodactyly 112–113 risk factors for 152
Macrodystrophia lipomatosa (ML) 112–113 treatment of 154
Mangled extremity severity score 79–80, 81 WHO definition for 152
Mean AOFAS Ankle-Hindfoot Scale score 19
Medial collateral ligament (MCL) 50
Medial stem pivot 26 P
Metacarpophalangeal joint (MCPJ), complex Paget’s disease 155
dislocation of 114–115 Patellofemoral arthroplasty 58
Metal-on-polyethylene (MOP) couples 166, 167 Patellofemoral osteoarthritis 55–58
Morquio-Brailsford disease 138, 138 aetiology of 55–56
MRI scan, in hip pain 42 management of 57
Mucopolysaccharidosis type IVA see Morquio- patellofemoral arthroplasty in 58
Brailsford disease total knee arthroplasty in 58
Mucous cyst 105 Pelvic avulsion fractures 86
causes of 105 Pelvic external fixator 78
complications of 106 Pelvic fractures 77
excision of 105–106, 106 management of 78
Multiple pterygium syndrome see Escobar syndrome risk of urological injury or impotence in 77
Mumford procedure 25 rotational and vertical instability in, radiological
Muscle contraction, types of 21–22 signs of 78
Muscular dystrophy 26 Tile classification of 77
Muscular torticollis 140 Young and Burgess system of 77
Myelomeningocele 136 Perrin’s strain theory 15–16
Pes cavus deformity 28
Pilon fracture 74
N CT scan in 75
Nail–patella syndrome 135 management of 75
Nerve recovery, first sign of 27 Ruedi–Allgower principles of operative fixation
Non-steroidal anti-inflammatory drugs (NSAIDs), in for 75–76
heterotopic ossification 45 Piriformis fossa 72
Nonunion, surgical treatment of 138–139, 139 Platelet derived growth factor 21
Polydactyly 142
Polydactyly of thumb with duplication of phalanges
O 104
Operating theatre inheritance patterns of 105
laminar airflow 160–161 surgical treatment of 105
reducing risk of contamination in 159–160 Wassel classification of 104
172 Index

Predictive salvage index (PSI) 80, 81 Segond fracture 76


Pre-ganglionic plexus injury 17 Seinsheimer type V fracture 72
Prosthetic joint infection, scanning method for 15 Septic arthritis 23
Proteoglycan 27 Silicone prosthesis 25
Proximal interphalangeal (PIP) fracture/dislocations Speeds test 27
107–109 Spinal cord compression, metastatic 54–55, 55
injury, assessment of 107 Spinal tumours 19
treatment for 108, 108–109, 110 Spine, embryology of 48
Pseudohypoparathyroidism 21 Spondyloepiphyseal dysplasia see Kniest syndrome
Psoriatic arthritis 103–104 Sprengel deformity 136
natural history of 104 Stener lesion 20
radiographic features of 103–104 Sternomastoid tumour 140
Stress fracture 42
Subtrochanteric fracture 72
R bisphosphonates in 73
Radial head dislocation and fracture of ulna 83–84, Fielding’s classification system 72
135 implant for 73
chronic post-traumatic dislocation of radial head management of 72–73
84 Russell-Taylor system 72
management of 84 Seinsheimer classification system 72
Monteggia equivalent fractures 84 Subungual exostosis 137
Monteggia fracture classification 84 Supraclavicular nerve, medial 17
Radiocapitellar line 83–84 Supracondylar osteotomy 134
Resection arthroplasty 54 Suprascapular nerve 17
Retropulsion 111 Synovial biopsy from knee 163–164
Rheumatoid arthritis (RA) 155 Synthetic bone grafts 151
aetiology of 156
ARA criteria for classification of 156
extra-articular manifestations in 156 T
investigations in 157 Talar fractures 48–49
poor prognostic indicators for 157 blood supply to talus and 49
Rheumatoid factor (RF) 157 Hawkins classification of 49
Riviroxiban 149 Talipes equinovarus 140
Tarsal tunnel syndrome 18–19
Tendons 18
S Tendon transfers, principles of 112
Sacro-iliac (SI) joint 77, 78 Tetracycline-labelled bone biopsy 17
Salter-Harris Type II injury, of lower femur 141 Thinner distal femoral augmentation wedge, use of
Scapholunate advanced collapse (SLAC) wrist 102 21
Scapholunate (SL) injury Thoracic curves 44–45
classification system for 102, 102 Tibial bowing deformity 135
with DISI deformity 101–103 Tibialis posterior dysfunction 25
natural history of 102 Tibial malunion 25
staging of 102 Tibio-femoral alignment, in children 18
treatment of 103 Tinel’s sign 19
Scapho-trapezio-trapezoid (STT) joint arthritis 46 Total ankle replacement 19
causes of 46 Total hip arthroplasty 164
complications of surgical treatment 47 ceramic liner in, damage to 164–165
treatment for 46–47 Total hip replacement
Scapular neck fracture, displaced 83 dislocation after 53–54
Ideberg classification system for 83 thromboprophylaxis regime for 149
injuries associated with 83 Total knee arthroplasty 58
surgical treatment of 83 Total knee replacement, thromboprophylaxis regime
Scoliosis, adult 44–45 for 149
Screws, orthopaedic 26 Tourniquets, in upper and lower limb surgery 157
Secondary hyperparathyroidism 16 complications of 158
Index 173

contraindications to use of 157 V


inflation pressures in 157 Valgus knee 50
post-tourniquet syndrome 158 Vancomycin 161–162
principles for use of 157 Ventilation system 160
Tranexamic acid 24 Vertical scapular osteotomy 136
Transforming growth factor-β (TGF-β) 21 Volar plate arthroplasty 108, 109, 110
Trapezoid ligament 74 Volar surgical approach, in complex dorsal MCPJ
dislocations 114
U
Ulnar collateral ligament of thumb, open repair of 20 W
Ulnar nerve entrapment 99 Warfarin 150
diagnosis of 99 Wartenberg’s syndrome 27
investigation of 100 Water and hyaline cartilage, interaction between 16
prognosis for 100 Weber C fractures, mechanism of injury for 28
sites of 99 Winged scapula 142
surgical intervention in 100 Wrist arthrodesis 24

Вам также может понравиться